You are on page 1of 77

Exam preparation: First do full outline with all the facts, reasoning, and class discussion.

Second, go over the first outline and summarize them. Third, do the step 2 again and make it shorter with short-hand. Last, make the most precise and simple outline that just reminds you things that are already in your head, i.g., AP: statutes, cases, main points. Exam grading: 1. Issue spotting; 2. Developing sub-issues (w/o this, everything is conclusory); 3. Organization (conclusion is not important, but developing an organized answer to argue the side you choose carefully is) Exam: 5 hours; Word Limits to organize your answer instead of spitting out everything you want to say; Citation not necessary but nice to have it; Make cases for both sides effectively; Completely open book but no communication with outside; Do not bring your cell phone; 2-3 questions I. Acquisition by Capture a. Pierson v. Post Supreme Court of New York, 1805 (p. 17) i. -respondent (Post/hunter) was hunting on uninhabited waste land and was pursuing a fox with his dogs and hounds. -appellant (Pierson/shooter), with knowledge that the fox was being hunted by , caught, killed, and carried off the fox. The trial court found for (Post) and the NYSC reversed and no property right is established. 1. Arguments a. claims he acquired title to the fox because he was the first to hunt it b. maintains that he acquired title to the fox because he was the one to kill it 2. Question of Facts v. Question of Law: If its question of law, the appellate court will look closely at what the trial court judge did. For question of fact, the appellate court will give greater deference to the trial court judges judgment because trial court got more information in person than appellate court in which the judge only reviews the briefs and transcripts. ii. Because a fox is a wild animal, a property right can be acquired in it only if the hunter occupies it. Some authorities feel that a hunter can occupy a wild animal only if he traps it. Others feel a hunter can occupy a wild animal if he mortally wounds it and remains in pursuit, because a hunter in pursuit of a mortally wounded animal has effectively captured the animal. All authorities agree that mere pursuit of a wild animal is insufficient to vest title in the hunter. This is to prevent litigation. If mere pursuit were to vest title, it would be very difficult to determine who the first to pursue was. Since was in mere pursuit of the fox, he acquired no title to it. iii. A hunter must either trap or mortally wound a wild animal in order to acquire title to it iv. Dissent: Since foxes are a public nuisance, the killing of foxes is in the public interest. Therefore, the rule that this court should adopt would encourage the destruction of these animals. The following rule accomplishes this: A pursuer acquires title to a wild animal if he is in reach of the animal or if he has a reasonable prospect of capturing the animal. The sportsmens rules and norms/customs should be looked at instead of rules made by the majority. (Poses the question when should a court consider the customs of a culture in which it operates?) v. Notes on Pierson 1. The majority creates a rule that capture = possession whereas the dissent sets a standard (p. 20, 3rd paragraph) that will require a consideration of the facts and contexts of each case to determine whether a party was in pursuit/in reach of the animal to conclude which party maintains possession 2. Rules versus Standards a. Advantage to having clear rules as opposed to fuzzy standards a. Increased efficiency, certainty b. Minimizes expenditure of resources fewer litigations 1. After this decision, someone in Posts position is not likely to litigate because theres a rule established that he would lose or decided by SJ 2. Encourage or discourage settlement c. peace and order in society b. Advantages to standards a. Allow for the investigation of facts of individual cases b. Sports-custom c. Encourages killing foxes due to social benefit Livingstons assumption is that law should be helping chicken farmers by encouraging killing of foxes and by rewarding effort to hunt them. But, even if we follow the majority opinion, are they discouraging hunters to kill foxes? Not necessarily SO LONG AS the hunters objective is to help chicken farmers. But Livingston is probably really assuming that hunters do hunt for sports, so if we follow the majority opinion, they will hunt less, so we dont get the result he wants. Understand the assumptions judges have to make in order to get to a point. Then, as a litigator, we can try to persuade judges to decide on our favor by suggesting the way people would react to the rule/decision. d. Rewarding effort c. If you have a party in Posts position who has put all of his energy into pursuit and some random person comes in and takes the prey, it doesnt seem fair d. The next Post knows the system wont help him and it might encourage Post to take action of his own; or, it could potentially encourage future Piersons who let the Posts do the work and take advantage in the end e. If courts fail to recognize peoples general sense of fairness or justice, peoples respect for the courts go down. But at the same time, whose standard of fairness or justice should be respected? b. Smith v. Jones- HO, classroom hypothetical i. Facts: Smith, an artist who uses pigeon feathers and other materials to make miniature doll clothing, aims and shoots at a pigeon while strolling along 5th Avenue. The pigeon falls to the ground on 12th street. Jones, a rival artist, scoops up the pigeon before Smith can capture it. ii. Procedural Posture: Smith sued Jones for the return of the pigeon and trial court held for Jones based on Pierson v. Post. Smith Appealed. iii. Construct the arguments for Smith/Artist that would persuade the judges who wrote the Pierson opinion. 1. The rule of the Pierson was acquisition = possession based on the reasoning that law requires actual possession of the animal with its freedom restricted before it will grant right to the pursuer. IN other words, the court was saying that there is no reason to object to a person who mortally wounds or seizes the animal to acquire possession of it. 2. In this case, the facts demonstrate that the person who actually limited the freedom of the animal, a bird in this case, was Smith and not Jones, although Jones scooped up the pigeon before Smith could. 3. In Pierson, three objectives were 1. Consistency, 2. Less litigation, 3. Peace in society. Finding for Smith will not undermine these objectives. This case fits the mortal wound standard clearly and still following the precedent. 4. Jones did not restrict the freedom of the bird from flying away but only took advantage by taking the bird away which could not fly away due to the injury caused by Smith, this case is different from Pierson. In Pierson, the person who acquired the fox also shot the

iv.

v.

fox and killed him. By applying the same reasoning of Pierson, this case should be decided for Smith because he was the actual capturer in a sense that he restricted the birds freedom by shooting, and Jones was a mere physical acquirer of Smiths alreadypossessed bird. Construct the argument for Jones/Scooper that would persuade the judges who wrote Pierson majority opinion. 1. The question asked in Pierson was whether one could obtain possession of a wild animal by pursuit and the court decided that pursuit was not occupancy, thus here the court has an opportunity to decide whether mortal wound/injury is enough. But keeping in mind why Pierson was decided the way it did, the argument on deciding what injury is and how to distinguish it, is a murky ground and court doesnt want to go there. The rule is a good rule and the courts must follow the precedent. 2. Even if what Jones did to Smith was mean and rude, that does not give Smith a right to the pigeon, thus following the Pierson rule, capture = possession, Jones should be entitled to the pigeon. Now, suppose instead that Livingstons opinion had been the majority opinion. How would this change the arguments that you ma ke for Jones and Smith? 1. Livingstons argument is that pursuit with reasonable prospect of taking and intent to take should be sufficient because it serves a useful purpose of encouraging hunters to kill foxes and possession can be seen in relative terms, and should reward effort. 2. Argument change for Smith/Artist: Smith was not only pursuing the pigeon but actually aimed and shot the bird and the bird fell to the ground because of the shot. Although killing a pigeon may not have the same socially useful purpose as killing a fox, here Smith demonstrated the control of the bird from the moment he shot it and the bird fell to the ground, thus it can be argued that Smith possessed the bird if Jones did not rudely took the actual pigeon body away. We should not encourage people who take advantage of other peoples effort and intent to capture an animal. Jones was cheating and behaved with an intent to screw Smith. 3. Argument change for Jones/Scooper: Jones did not steal the pigeon away while it was in the physical possession of Smith. Smith may have shot the bird, but he had not taken it into his possession yet and just left it there. Here pursuit itself also has no socially useful purpose because Smiths pursuit of killing pigeons was for his own benefit. Pierson had a strong concern about the sports, but here court doesnt need to worry about sportsman or foxes. Affirmative argument: actual capture is enough.

II.

Acquisition by Adverse Possession a. The Theory and Elements of Adverse Possession i. Powell on Real Property 91.01 (112) 1. Every American jurisdiction has one or more statutes of limitations that fix the period of time beyond which the owner of land can no longer bring an action, or undertake self-help, for the recovery of land from another person in possession 2. Case law elaborates on the kind of possession by another that is sufficient to cause the statutory period to begin to run, and to continue running, against the true owner. 3. AP functions as a method of transferring interests in land without the consent of and even in spite of the dissent of the prior owner. 4. Social judgment is that after a reasonable time period, the person claiming to be an owner should assure security of the possession. 5. Theory: adverse possessor may acquire title at such time as an action in ejectment by the record owner would be barred by the statute of limitation. ii. Oliver Wendell Holmes, The Path of the Law (113) 1. A thing which you have enjoyed and used as your own for a long time cannot be torn away without your resenting the act and trying to defend yourself, however you came by it 2. If the owner knows that another is doing acts which on their face show that he is on the way toward establishing such an association, I should argue that in justice to that other, the owner was bound at his peril to find out whether the other was acting under the owners permission, to see that he was warned, and, if necessary, stopped 3. AP is motivated by economic, psychological, and/or moral concerns iii. Notes (114-115) 1. The running of the statute of limitations not only bars an action by the erstwhile/previous owner, but also vests a new title, created by operation of law, in the adverse possessor a. Once acquired, this new title relates back to the date of the event that started the statute of limitations running. The law treats the adverse possessor as the owner from that date 2. Adverse possession applies to personal property as well as land with some exceptions 3. Regarding the length of the statute of limitations for adverse possession, 20 years used to be common, but the modern trend is to shorten this to something around 6 10 years iv. Class notes Date Grantor Grantee Date Grantee Grantor 1910 Leslie Hoffman 1910 Hoffman Leslie 1920 Hoffman Serebro 1920 Serebro Hoffman 1930 Hoffman Whitingham 1930 Whitingham Hoffman 1955 Whit. Tievsky 1955 TIevsky Whit. 1980 Tiev. Grange 1980 Grange Tiev. 1. Every time you transfer the ownership of the land, you make a new deed listing a new grantee and grantor. 2. If you are wanting to buy the land, you first look at the Grantee-Grantor index, day by day, looking for the entry on when Grange purchased the land. Tedious job without internet. Assume you found it. 3. Then, you have to make sure Tievsky had a right to sell the land to Grange. So, look for that record. 1955 Whitt Tievsky. Do the same thing for 1930 and 1910 record. 4. If you want to be really sure, you can do it all the way to the beginning of the country. 5. Suppose, in 1920, Hoffman sold the property to Serebro. If you just look at Grantee-Grantor index, you dont see Serebro. Only when you look at the Grantor-Grantee index, you see Serebros name that you didnt see in Grantee -Grantor index. 6. Point, go back in Grantee-Grantor index then go forward with Grantor-Grantee index [the burden is on the purchaser not on the clerk at the office]. 7. AP facilitates clearing up the clouds in the sky when you have doubts about how to solve a problem; given how little it takes for you to visit your own property from time to time to check on it v. how much it takes for someone who lives on the property to loose 8. Without AP, its really risky to buy a property especially if you dont have records that go far enough back. You can still argue that a person lived on that land for the last 20 years and no other records are found (search is super expensive and takes up a lot of time and energy). It allows people to buy property at fair market price. 9. Reward people to develop land/encouraging cultivation. Leslie doesnt think its compelling, but maybe the study guides say so and people write that in the exam. v. Van Valkenburgh v. Lutz Court of Appeals of New York, 1952 (115) 1. In 1912, (Lutz) bought lots 14 & 15 in a large subdivision. Instead of climbing a steep grade to get to his lot, crossed lots 19 -22, which he did not own, to get to his lot. With regard to lots 19-22, cleared it, built a one-bedroom shack on it, and grew vegetables on it. In 1928, the city broke the private water line leading to s main home. left work to fix it and as a result, lost his job. developed a farm on the disputed land and grew and sold vegetables to neighbors, the proceeds of which were his livelihood. In 1937, purchased a lot near . In 1947, purchased lots 19-22 at a sale by the City of Yonkers who had taken the land from the previous owner for failure to pay taxes. was not notified of the sale, although the City should have provided notice because adverse possession gave a property interest in the land. Soon after, sent a letter to informing that now owned the propert y. The letter also requested that remove his property from the land. Later, erected a fence across the land, preventing from crossing. sued , admitting that owned the land, but claiming a right -of-way across the land. prevailed on this issue. sue here to gain possession of the lots. contends that he has acquired title to the lots by adverse possession. Judgment for (reversed) 2. NY Civil Practice Act 39 (requirements to acquire title by possession) clearly states that land must be actually possessed. This requirement is not met here for 2 reasons: (1) the garden occupied only a small portion of lots 19-22 and (2) the s garage encroached on the land by only a few inches. NY Civil Practice Act 40 (requirements to acquire title by adverse possession) clearly states that where a person claims title not founded upon written instrument, he must show that the land was protected by a substantial enclosure or that he cultivated or improved the land. This requirement was not met here. Concededly, the land was not enclosed. The land was not sufficiently improved for 3 reasons: (1) the shack built on the land was too small, (2) the garden was insubstantial and (3) placing rubbish on the land is not an improvement. New York Civil Practice Act 39 clearly states that land must be possessed under claim of title (be hostile). Since testified that he knew the land belonged to , fails to prove the claim of title element of the adverse possession statute 3. In order to acquire title by adverse possession, possession must be actual, it must be under claim of title, and the land must be either enclosed or sufficiently improved 4. There is substantial evidence to indicate that the land was substantially improved. Second, the land need not be completely occupied. It needs to be occupied only to the extent necessary to put the true owner on notice. There was enough evidence to

vi.

vii.

support this element. Finally, in order to satisfy the claim of right element of the statute, it is not necessary that b elieved that the property belonged to him. It is sufficient only that intended to acquire and use the land as his own. The fact that admitted that he did not have title is only evidence of whether or not they intended to acquire and use the land as their own 5. Notes on Van Valkenburgh a. NY statutes was required to meet because he was not claiming title under a written instrument - 39 and 40 a. 39 Where there has been an actual continued occupation of premises under a claim of title, exclusive of any other right, but not founded upon a written instrument or 1a judgment or decree, the premises so actually occupied, and no others, are deemed to have been held adversely 1. How do we define claim of title? b. 40 For the purpose of constituting an adverse possession by a person claiming title not founded upon a written instrument or a judgment or decree, land is deemed to have been possessed and occupied in either of the following cases, and no others: 1. where it has been protected by substantial enclosure 2. where it has been usually cultivated or improved b. Not occupied, because 1) did not cultivate ENTIRE parcel, 2) not sufficiently improved. If you make that as question of fact, you can argue that no reasonable fact finder could not have come out that way and lower courts holding must be upheld as long as there is evidence in the record to support that. Then argue that there is evidence in the argument to support referees determination. For example, notice to the true owner and future purchaser is important to determine whether the property owner acted as a owner. Another reason could be reliance that it is not fair for the owner to know but never stopped cultivating the land and thus did not prevent improvement of the land. Caveat: Interpreting a reasoning of a court is perfectly legitimate if the reasons are not listed out. c. Claim of title/right (Hostile): Court said that Luz admitted that he did not have the claim of title in a different suit. Under the courts reasoning, there is no way to prove claim of title. Its really murky what claim of title/right means here. d. Possible justification for the doctrine of adverse possession there are people who reasonably rely on property and may even think that the property belongs to them. All of a sudden, years later, the owner shows up and claims the land. It wouldnt be fair to remove the adverse possessor who reasonably relied on the land a. Applies to the adverse possessor who is mistaken about what they own b. In the case of the adverse possessor who trespasses and relies anyway, the persuasiveness of reliance loses its force e. Not persuasive argument we want to encourage/reward development of dormant land a. However, this isnt good because we cant force people to develop their land and the owners of the property should be able to do with the land whatever they want b. Immediate exploitation of the land isnt always efficient c. Even if we did want to encourage cultivation, adverse possession is not effectual for that purpose most people are not dumb enough to invest money in land they dont own in hopes that the true owner wont show up before the statute runs White v. Tilly (Angel) 1. -respondent owned lands bounded on the west by the shore of a lake and on the easy by a highway. In 1959, -appellant acquired a tract of land on the east side of the highway and the conveyance included a parcel of land on the west side of the highway which adjoined on the northern side of respondents land. In 1925, constructed a breakwater along the westerly shoreline of his property and the wall extended along s property as well as s and an additional 12 or 13 ft. In 1936, another breakwater erected by extended across the 10-ft strip owned by . In 1963, Pope constructed an extension of one of the breakwater walls and filled i n part of the bay. established that the triangular parcel which included the westerly strip of s land was enclosed from 1936-1963 and that the and members of her family exclusively occupied the parcel for more than 25 years. In 1939, a concrete shuffleboard was constructed partially on the s land and the remainder on s land. The remainder of the property was made into a lawn and shrubs were planted. The grass was regularly cut by . asserts that s occupation of the land was not notorious because the 10-ft. strip was wild, overgrown and little used; that it was not until Pope cleared out and improved his lakefront property that s occu pation was apparent from the highway. testified that when building the wall, she only thought to be enc losing her own property; and that since 1935, she had treated the land as though she was the owner. The court held that the -respondent was the owner by adverse possession Notes on the classroom exercise a. Court held for best arguments (as per Van Valkenburgh) was acting under a claim of title, protected the land by substantial enclosure and did cultivate the land within the walls by building the shuffleboard court and maintaining the grass and the shrubs; owner clearly had notice and the minute the wall went up, the owner should have said something (Like Lutz, began to rely on the inaction of the and treat the land as his own if another person drove by and looked at the land, that prospective purchaser would have thought the land belonge d to , the adverse possessor); cultivation needs to be what a normal would do b. Best arguments for was not acting under a claim of title; if hostility was necessary, the s actions were not hostile (like Lutz, asserts at first that she was only trying to enclose her own property like Lutzs garage and because the court held Lutz believed it was his property (good-faith), there was no hostility) Notes (124) 1. In most, if not all jurisdictions, the courts have developed a series of requirements of their own. These are stated in various ways, but their essence is that there must be (1) an actual entry giving exclusive possession that is (2) open and notorious, (3) adverse (or hostile) and under a claim of right, and (4) continuous for the statutory period 2. If the adverse possessor abandons the property leaves with no intention to return before the statute has run, the statute stops, a new entry is required, and the whole process begins anew 3. The sort of entry and exclusive possession that will ripen into title by adverse possession is use of the property in the manner that an average true owner would use it under he circumstances, such at neighbors and other observers would regard the occupant as a person exercising exclusive dominion 4. Claim of Title what does this mean? Obviously unclear in Van Valkenburgh a. Approach the inquiry in terms of the state of mind required of the adverse possessor and in this respect existing doctrine reflects 3 different views: a. Objective Standard State of mind is irrelevant, but your actions are like the true owner 1. Majority position in the US b. Good-Faith Standard The required state of mind is, I thought I owned it 1. Voiced from time to time in US decisions

b.

Aggressive Trespass Standard The required state of mind is, I thought I didnt own it, but I intended to make it mine viii. Color of Title and Constructive Adverse Possession (129) 1. Color of Title refers to a claim founded on a written instrument (a deed, a will) or a judgment or decree that is for some reason defective and invalid (as when the owner did not own the land conveyed by deed or is incompetent to convey, or the deed is improperly executed) a. Not a prerequisite for adverse possession in most states b. Has advantages for the adverse possessor a. In some states a shorter statute of limitations is applicable to adverse possessors with color of title than to those without b. In all states entry with color of title may have an advantage where the adverse possessor enters into possession of only a part of the property c. Actual possession under color of title of only a part of the land covered by the defective writing is constructive possession of all that the writing describes City of Tonawanda v. Ellicott Creek Homeowners Assoc. (Angel) i. This case concerns the ownership a use of certain real property lying between Ellicott Creek and Ellicott Creek Road. The City has record title to the property, but certain individuals who own residential property across Ellicott Creek Road from the contested strip of creek-front property, as well as 10 boat owners who own no adjacent property, have used the creek-front property for years. In February 1977, the City commenced an action for ejectment against all 41 named s, including the Ellicott Creek Homeowners Association, Inc. The act ion has been discontinued against certain individuals and others are in default. The Association and 24 individuals answered and asserted affirmative defenses and counterclaims for adverse possession and easements by prescription. The City moved for summary judgment dismissing the defenses and counterclaims for adverse possession. The City has appealed and the s have cross -appealed from that order. ii. The s make no claim of title under a written instrument and do not deny that the City is the record owner. The City would be entitled to summary judgment unless the s can demonstrate their rights by competent evidence their claim to title through adverse posses sion. With respect to the claim of adverse possession, 5 elements must be established: (1) possession was hostile and under claim of right, (2) actual, (3) open and notorious, (4) exclusive, and (5) continuous for the statutory period. The majority of the claims for adverse possession must be dismissed because the s have failed to demonstrate that their po ssession was continuous for the requisite statutory period (applicable period is 15 years if prior to 9/1/1963 and 10 years thereafter) 1. Branns and Eckerts maintained the creek-front land by mowing the grass and planting trees a. Court grants summary judgment one seeking to obtain title by adverse possession on a claim not based upon a written instrument must show actual occupation of the premises which requires that the parcel be usually cultivated or improved or that it have been protected by a substantial enclosure (Van Valkenburgh) The creek-side property was suitable for more extensive use than that asserted by these s 2. Maurer summary judgment should not have been granted affidavit sets forth acts which are sufficient to establish usual cultivation or improvement (replacing a dock, erecting a light pole, installing a retaining wall along the bank, granting another permission to moor his boat at the). Question of fact as to whether Maurer ever admitted or recognized title rested with the City, and, if so, whether such acknowledgement occurred prior to the completion of the 10-year statutory period (Van Valkenburgh) 3. Anderson evidence of admission during the statutory period that title rested in another; his adverse possession claim must fail (Van Valkenburgh). Claim for prescriptive easement is not dismissed 4. Melick also fails for the same reason as Anderson recognized title in another during the statutory period. Sought permission to use the creek-side property which defeats claim to prescriptive easement iii. To defeat the record title owners claim for summary judgment, s must show that their possession was hostile and under claim of right; actual; open and notorious; exclusive; and continuous for the statutory period. In addition, may not have acknowledged that title was held by another 1. What does it really mean to be under the claim of title? City needs to show that people had Permissive Use. Whats relevant about admission is that it will induce the true owner to be in sleep, so it should defeat the claim of title. Its subjective and permissive. If you use a property WITHOUT acknowledging the other has the ownership, you have the claim of title and that acknowledgement destroys hostility. c. iv. Notes on City of Tonawanda 1. Question of whether the City had designated the land for public use if so, there can be no adverse possession Why not allow adverse possession on public property? a. It would create chaos! Very difficult for the government to regulate and very expensive, too! No notice requirement would ever be met because the government is allowing the public to use the land possession might be open and notorious, but would never be adverse or hostile 2. There is a presumption that the record owner had the possession of the premises (although the owner did not have possession until he purchased the property and got record, it is presumed that he has been having the possession). So if someone wants to claim adverse possession, the burden of proof is transferred to that person claiming adverse possession by rebutting the presumption. 3. The element of hostility is presumed from the proof that the possession was actual, open, continuous, and exclusive.

c.

Tubolino v. Drake (Angel) i. s commenced this action to quiet title(they are already in possession and just trying to secure the title by the judgment of the court) under RPAPL article 15, claiming ownership of a disputed 10-acre parcel of land. s contended that their chain of title is superior to that of s, and that, in any event, s acquired title to the property by adverse possession. Following a bench trial, judgment was entered dismissing s complaint and impliedly awarding judgment to s on their counterc laim asserting ownership of the property. Reversed and judgment is awarded to s on their claim of adverse possession, which supersedes the issue concerning which party has a superior chain of title ii. In order to establish title by adverse possession, s were required to show that for a period of 10 years s actually possessed the premises and that such possession was hostile and under a claim of right; open and notorious; and exclusive and continuous for the statutory period. RPAPL 512 provides that a person claiming adverse possession of land nominally owned by him is deemed to have possessed it where it has been usually cultivated or improved, protected by a substantial enclosure, or used for the supply of fuel or of fencing timber either for the purposes of husbandry or for the ordinary use of the occupant. The common-law and statutory elements of actual, open, exclusive, and continuous use and possession of the disputed parcel were established by s. The 10-year requirement was met by proof that s and their predecessors possessed the property since 1961 ( asserts 1st claim in 1988). Exclusivity is clearly met and s possession was actual and open. s hunted, fished, trapped, and walked on the property. s cut numerous standing trees and fallen logs (can look at 38 from Van

d.

Valkenburgh), and removed the timber over a road constructed by them for that purpose. In addition to constructing the road, s constru cted a drainage culvert, repaired a footbridge, and posted a no trespassing sign. Those activities constitute sufficient cultivation and improvement where the proof was uncontroverted that the land was unsuitable for farming or other development. Continuity is well-established. s immediately entered the property upon acquiring it and the adjoining residential parcel in 1976, and their exclusive possession was unchallenged and not interfered with until 1988, when asserted his first claim. s and their predecessors took title to th e property by written instrument and paid the property taxes on the separately addressed parcel for over 20 years iii. Where the necessary common law and statutory elements of adverse possession are met, the adverse possessor will take title to the property iv. Notes 1. had the deed, paid property tax. Then what is claiming? also had deed. There is a competing chains of title. Somebody dee ded this property twice. Then, why just figure out who did that, and try to figure out whose chain is superior? Its easier to deal with who has the title now instead of going back the chain and determining whose chain is superior. Most recent 10-15 years is much easier to deal with than going back so many years back. Clean example how adverse possession facilitates peoples sense of who owns the property and fulfilled the expectations. 2. Isnt it too harsh to give possession to adverse possessor and not compensate the party who paid for it? In this case, did not even know he had the land until recently. So whats there to loose really? 3. In trial court, by dismissing the complaint, it implied to uphold s right. 4. Here, the person is claiming under Color of title; He has a deed. Its easier to claim adverse possession. Look at page 118-119. For cases without written document, 39-40 was looked at. But for this case, look at 38. Where a known farm or single lot..improved and cultivated. This means that as long as you cultivate a part of the land as would have done normally, you are at a bett er position to claim adverse possession. 5. Reliance when someone has a deed is the purchase and what they did to the land while reliance without a written document can be only viewed by what they did to the land. 6. Think about the purposes of adverse possession and why having a deed makes it easier to uphold those purposes. Maybe by having the deed, the parameter of whats sufficient cultivation is more clear. As a prospective purchaser, if you look at the deed and how the owner treats the land, you get better reliance on their ownership and feel more comfortable. 7. Think about how we want to promote prospective purchases and how we can do that. Owner would argue that regular entrance to my land is not enough notice. But opponent will say that cultivation here was enough to give notice. Moreover, having the deed and cultivating the land strengthens the validity of the ownership when he is trying to sell the land to another. The Mechanics of Adverse Possession i. Mannillo v. Gorski Supreme Court of NJ, 1969 (130) 1. (true owner) filed a complaint for an injunction against an alleged trespass upon their lands. counterclaimed for judgment by adverse possession. 2. Facts: and are next door neighbors. Since moved in 1946, made changes to the house and built steps and in 1953 raised the house and extended steps and concrete walk which admits to encroach upon s lands by 15 inches. 3. s argument: To establish adverse possession, the encroachment is required to accompany an intention to invade the rights of another in the lands. But in this case did not have such required intention but encroached s land by mistaken belief that she owned the land. 4. Courts reasoning: 1) Requiring intention to claim the ownership of land that is not his rewards the possessor who entered wi th premeditated and predesigned hostility & disfavors an honest, 5. Notoriety/Notice: in the case of minor encroachments, the owner of the land must have actual knowledge thereof in order for the adverse claimant to satisfy the notoriety requirement. If true owner had no actual notice, opponent cant claim adverse possession. 6. Notes: No disputed facts but is claiming that law is wrong so the case is at state supreme court. In NJ, to be hostile, you cant ask for permission because then youre acknowledging their right. Now, without permission, you have to go to objective standard and look at what they did with the land. This court doesnt want to deal with notice argument, but deal with open and notorious requir ement. By doing so, it still puts the responsibility to do survey on encroacher/cultivator. ii. Howard v. Kunto Court of Appeals of Washington, 1970 (136) 1. Dating back to 1932, a 50-ft tract of land bordering the Hood Canal in Washington and the summerhouse upon it had passed through several owners, until (Kunto) took possession by deed in 1959. In each transaction, the same legal description was used on the deed and surveys were conducted on the property. The Moyers occupied the parcel of land immediately east of s land. (Howard) sought to convey a one-half interest in his property to the Yearlys. Upon commissioning a survey of his property, learned that the legal description of his deed did not coincide with his property and that he was in fact the record owner of the Moyers property. Likewise, the Moyers were the record owners of the property owned by , who owned the property immediately to the west of the land upon which his house was constructed. After making this discovery, agreed with Moyer to convey record title of his property that which Moyer previously believed to belong to him in exchange for Moyers interest in the s property. Because neither Moyer nor any of his predecessors in interest had ever asserted ownership of the s property, he agreed to t he conveyance. and Yearly filed an action to quiet title. When considering s claim that he had acquired title to the prope rty by adverse possession, the trial court concluded that had not continuously possessed the property because it was used only for summer occupancy, and that he was not permitted to tack (add his years of adverse possession to that of his predecessors) his predecessors occupancy of the property to establish the 10-year statutory period required for adverse possession. appealed. The SC of WA reversed 2. To constitute adverse possession, there must be actual possession which is uninterrupted, open and notorious, hostile and exclusive, and under a claim of right made in good faith for the statutory period. Uninterrupted possession does not require constant yearround occupancy, but rather such control over the property as ordinarily marks the conduct of owners in general in holding, managing, and caring for property of a similar nature and condition. Accordingly, occupancy of the summerhouse during summer months over the 1-year period sufficiently established adverse possession. , however, has not personally occupied the property for the required 10-year period. Accordingly to establish continuity for the statutory period, tacking of s predecessors possession must be allowed. In the ordinary situation in which one seeks more than is described in a deed, a purchaser may tack the adverse use of its predecessor in interest to that of his own where the land was intended to be inc luded in the deed between them, but was mistakenly omitted from the description. Such tacking is permitted because the current possessor is in privity of estate with each successive possessor by operation of the deed running between them. The notion of privity ensures that continuous trespassers, seeking ownership of property without a valid claim of tight, will not defeat the property interests of true owners. In reality, however, privity is a judicial recognition of a reasonable connection between successive occupants who have a good faith claim of right. Whether they seek more property than represented in their deed or a different parcel altogether, their successive interests are in privity and tacking is permissible to establish adverse possession.

Because and his successors continuously occupied the parcel for more than the statutory period (see statute on p. 137), is entitled to quiet title to the disputed property 3. Tacking on successive possessions of property is permitted for purposes of establishing adverse possession if the successive owners are in privity 4. Why didnt the lower court allow tacking? There was no problem with privity (the successive relationship between owners). The re was no description on the deed. 5. This court does what weve been doing: Look in to the purpose of the rule of privity. Why do we require privity? You want to promote that purpose. If the previous owners were just all trespassers, then privity doesnt apply. But the owners here are n ot trespassers. 6. How to define privity? Whats the purpose of privity? Then you get an answer. 7. Another issue was continuity. Whats the objective of continuity? Part of it is notice and part of it is about control. Over the course, think about ownership as a legal right to possess land 1. Over time/For a period of time as in Lease, 2. Concurrent use rights by separate owners to use the same property, 3. Concurrent use rights but to use in different characteristic ways. In property law, enterprise is different from contracts law. Its always about what c ategory best defines the type of contracts parties made, and once thats decided, court can decide what liabilities and rights each party has under that. Later, contracts principles began to come in to property law.

III.

Possessory Estates a. Up from Feudalism (175) i. A central feature of the social system was land tenure, according to which ones position was defined in terms of ones relat ionship to land. Everyone (except the King) was made subservient to someone else, literally, his land lord. All were subservient to the crown, from whom all land titles derived ii. According to the status in land tenure, a feudal pyramid was built up, with services flowing to the king at the top and protection extending downward to the actual occupants of the land at the bottom. iii. Feudal Tenures: 3 major tenures- Military, economic and religious- each with accompanying services iv. Tenure was so closely tied to grant of land that even symbolic service were to be due. v. Feudal Incidents: While services were fixed obligations first by providing knights but later by money which was affected in value by inflation, incidents were jealously guarded by the king and gave the lord the possession of the land or its equivalent which maintained value and kept pace with inflation. vi. After Quia Emptores, with free substitution permitted, the relationship between tenant and lord basically became an economic one and the personal relationship, a key feature of feudalism, began to astrophy. b. The Fee Simple (181) i. Up from feudalism developed a system of estates in land. A tenant had a status as a tenant of the fee or a tenant for life. In the course of time, status became estate ii. How the Fee/ Simple Developed 1. Rise of Heritability a. One of the key features of feudalism was that land was not owned by the possessor but was held by the possessor as tenant of someone else b. Due to highly personal nature of tenant-lord relationship, the fee or holding could not be inherited once he is dead, but customarily was transferred to the heir for same obligations c. By the beginning of the 13th century, inheritance of a fee became a matter of right, but the payment of a relief to the lord continued 2. Rise of Alienability a. As the demand for land increased, the idea that a tenant should be able to convey the fee to another during his life openly and without the lords consent gained currency b. By the end of the 13th century, the fee was freely alienable 3. Rise of the Fee Simple Estate a. Once the fee became alienable, the feudal realities behind a conveyance became meaningless b. The fee, which started out as simply a holding, became an alienable fee simple, a freehold estate not terminable at the will of the lord, with an existence on its own a. The fee could be conveyed during the owners life b. And if he owns fee simple at death, it passes under his will or descends to his heirs 1. creditors may reach the fee simple and sell it to pay debts in default c. Modern analysis estate is a bundle of rights and what rights are in the bundle involves issues of public policy, not merely an analysis of qualities of imagined objects, and rights in the bundle have varied depending on time and place d. Fee simple is as close to absolute ownership as our law recognizes a. Largest estate in terms of duration it may endure forever iii. Creation of a Fee Simple 1. At early common law a fee simple was created by the grantor conveying land to A and his heirs a. and his heirs inserted in a conveyance indicated that As interest in the land was inheritable by his heirs, but such words did not give As prospective heirs any interest in the land As heirs would inherit the fee simple at As death, but would have no interest during As lifetime b. and his heirs words of limitation indicating that A takes a fee simple c. No longer necessary to put words of inheritance (and his heirs) in a deed in any state a. Statutes and judicial decisions now provide that a grantor is presumed, in the absence of words indicating otherwise, to transfer the grantors entire estate 2. Problems on page 183 a. Question 1: What does it mean O conveys? Action O took during his life (while hes still alive) using deed. a. In 1600, A has life estate. What about B? Life estate. The presumption in 1600 was that if the language was not absolute, then life estate. So what it said was O conveys Blackacre to A for life and B for life. b. If both A and B die, then the right reverts back to O. In 2002, A has life estate and B has absolute fee simple for remainder. If A and B die, first look at Bs will. If B dies without a will (intestate), follow intestate statute and the order of preference. b. Hypo: O conveys Greenacre to A for life and remainder to B. If A is still alive, does B have right of possession? No. He has no possessory right but he has legal remainder that has value but with no right of possession until the end of life estate. So, B is a legal remainder. If B dies before A, then whos getting it upon As death? Look at As will (?) c. Question 3: O conveys Greenacre to A and her heirs. Bs the only child and Bs creditors want to attach Bs property. What legal right does B have? Do the words and her heirs give B any legal right? Fee Simple Absolute from O to A. In modern times, the words and her heirs have no meaning and they are just residual language. Its same things with O conveys Greenacre to A. So, B has no legal right because As still alive. a. If A dies with a will stating that all of her property goes to her trainer, then the trainer gets the Greeenacre and not B. b. If A dies intestate, with no spouse and only B as a child, then according to the statute, B has the property and his creditor can come after B. d. Hypo: O to A, remainder to As heirs As attorney will say that since the language is ambiguous about Os int ention, we must assume fee simple absolute. Bs attorney will say A received life estate and B has remainder because the language suggests sequence. a. If you want to give it to A, then just say conveys to A or to A fee simple absolute. b. If you want to give it to A and make sure it never goes to someone else like a trainer, then to A during the duration of his life (life estate) and at As death, the property goes to ___ people (remainder). iv. Inheritance of a Fee Simple 1. Heirs if a person dies intestate, the decedents real property descends to his heirs

2.

3.

Heirs are persons who survive the decedent and are designated as intestate successors under the states statute of descent A spouse was not an heir at common law, but in all states today, the surviving spouse is designated as an intestate successor of some share in the decedents land; the size of the share can depend on who else survives Issue if the decedent leaves issue, they take to the exclusion of all other kindred a. Issue = descendants a. Children means just one generation down, but issue (not issues) or descendants are anyone down the line that gets the property rights upon intestate. b. Until 1925, the rule of primogeniture applied to most of the land in England a. Primogeniture was followed in some US states before the Revolution, but was completely abolished by the end of the 18th century c. Child born out of wedlock was filius nullius the child of no one and could not inherit from neither mother nor father at common law a. Today a child born out of wedlock inherits from the mother, and if paternity is acknowledged or proved, from the father b. In US states, adopted children inherit from their adoptive parents and sometimes from their natural parents as well d. Ancestors by statute parents usually take as heirs if the decedent leaves no issue e. Collaterals all persons related by blood to the decedent who are neither descendants nor ancestors a. If a descendant leaves no spouse, no issue, and no parents, the decedents brothers and sisters (and their descendants) take in all jurisdictions f. Escheat if a person died intestate without any heirs, the persons real property escheated to the overlord in feudal times a. Now such property escheats to the state where the property is located Problems on page 185 a. Question 1: In England in 1800, B2 inherits because hes the only son. Under modern American law, Court will follow intestate statute which usually will split it between A and B. But B is dead and left everything to his wife. But intestate statute only gives to blood-related children. So, the split will be between A and B1, B2, and B3. So the court just created 4 ownerships. b. Why not to Bs wife? B left everything to wife. But B never had the ownership of the property. So, the transfer can be made to W only if B had it first. Because B never had it first, property goes straight from O to A and other descendants (B1-B3) c. Question 2: Yes, the escheat to the state. a. b.

c.

The Life Estate i. Judicial recognition of a life estate had 2 important consequences: 1. It meant that the grantor of a life estate could control who takes the property at the life tenants death 2. As land and stocks and bonds came to be viewed as income-producing capital, trust management for the life tenant developed a. Under modern trust management, one person (often a corporate person) manages the property for the benefit of the life tenant, paying the life tenant the income therefrom ii. White v. Brown Supreme Court of Tennessee, 1977 (190) 1. , along with her husband and daughter, lived in Lides house for 25 years. When Lide died, she left a holographic will which read in part: I wish Evelyn White (), Lides sister-in-law, to have my home to live in and not to be sold My house is not to be sold. contends that the will conveyed a fee simple interest in the house. Brown () contends that the will merely conveyed a life estate to . The trial court ruled that the will, on its face, unambiguously conveyed a life estate to . Since the trial court fo und that the will was facially unambiguous, it did not consider extrinsic evidence. The SC of TN reversed 2. At common law, there was a presumption that a testator conveyed only a life estate. However, the legislature has enacted a rule to construction which reverses this presumption: a will shall convey all the real estate belonging to the testator unless a contrary intention appears by the terms of the will and its context. Several cases demonstrate the strength of the presumption that a testator conveys his entire interest. It is clear that if the only language in Lides will was that the home was for to live in, a fee simple would pass. However the language not to be sold complicates matters. The court must look to th e context to determine if Lide intended to convey only a life estate. Where there is ambiguity, the court construes a will to dispose of the testators entire estate, which would be a fee simple in this case. In this case, it appears that Lide attempted to pass a fee simple to and further attempted to restrain from alienating the property. This attempt to restrict alienation is clear: My house is not to be sold. A restraint on alienation is void as against public policy, leaving with a fee simple 3. Unless a contrary intention appears by the terms of the will and its context, a will conveys a testa tors entire interest 4. Lides will is not ambiguous. It is clear that she intended to pass only a life estate: to have my home to live in and not to be sold. Here, Lide bequeathed to only real property. In this case, the testatrix knew how to convey all her property. She demonstrated this when she left all her personal property to her niece. As to , she merely wished that she live in her home; she inten ded to convey a life estate 5. Notes on White a. White prevails court holds that the will created in White a fee simple absolute b. Brown nieces and nephews of Lide say the will is a conveyance of a life estate only a. Brown (and others) would benefit from the courts finding of a life estate because the y are assigned, as per the intestacy laws, as heirs and so if its a life estate, Brown and the others would get fee simple to the home 1. Why do they (Brown etc.) have standing? There nothing specific in the text of the statute that makes this a life estate with a designee of the property after Whites death. They argue that they have a remainder interest because the will created a life estate and because the will didnt provide remaindermen, Brown and others would prevail 2. Argue not to be sold is a restraint on alienability thats inconsistent with fee simple absolute Lide must have wanted to give something less than fee simple by saying the house could not be sold during Whites life the use of to live in is also an indication of a life estate c. The majority assumed that Lide didnt know the legal rules surrounding her transaction a. The majority decided that there were some ambiguous language, so the court wanted to look at the two statutory presumptions giving favorable result to , but this court found for . b. Lide probably assumed she could keep the house from being sold c. Majority looks at incorrect assumptions as an explanation and says there was misunderstanding, and Lides intent was to grant a fee simple absolute

d.

a.

a. iii.

iv. v.

b. Notes (195) 1. Objections to restraints on alienation a. such restraints make property unmarketable b. restraints tend to perpetuate the concentration of wealth by making it impossible for the owner to sell property and consume the proceeds of sale c. restraints discourage improvements on land d. restraints prevent the owners creditors from reaching the property, working hardsh ip on creditors who rely on the owners enjoyment of the property in extending credit 2. Restraints on alienation have traditionally been classified as disabling restraints, forfeiture restraints, and promissory restraints a. Disabling Restraint withholds from the grantee the power of transferring his interest b. Forfeiture Restraint provides that if the grantee attempts to transfer his interest, it is forfeited to another person c. Promissory Restraint provides that the grantee promises not to transfer his interest a. If valid, this is enforceable by the contract remedies of damages or an injunction b. Rare, except in the landlord-tenant context d. Restatement (Second) of Property, Donative Transfers generally treats all these restraints alike when they are imposed on a fee simple a. Provides that an absolute restraint on a fee simple is void b. But with respect to partial restraints on a fee simple, the Restatement takes a more tolerant position than do most courts 1. Provides that a partial restraint is valid if, under all the circumstances of the case, the restraint is found to be reasonable in purpose, effect, and duration 2. An absolute disabling restraint is void, but a forfeiture restraint is valid Valuation of Life Estate and Remainder 1. Since we do not know how long the life tenant will actually live, to value the life estate we assume the life tenant will die at the time predicted by a life expectancy table Baker v. Weedon Supreme Court of Mississippi, 1972 (197) 1. John Weedons first marriage to Lula Edwards resulted in 2 children, Florence Baker and Delette Jones. Florence Baker () mothered 3 kids, Henry Baker () and 2 others. After a second marriage, John Weedon finally married Anna Weedon (). and John had no children together. In his will, John left a life estate to Anna () and left to his grandchildren a contingent remainder: I give to Anna all of my property during her life and in the event she dies without issue then I give all of my property to my grandchildren. John died in 1932. ceased to farm the land in 1955 due to her age and began to rent the property. It is undisputed that the rental income and s other income is insufficient to support . In 1964, just as the commercial value o f the land was increasing rapidly, the state sought a right-of-way through s property. At trial, the property was worth $168,500. However, the propertys estimated value in 4 years is placed at $336,000. Although now lives in a new home, she is in economic dist ress and prays that the land be sold so that she can reap her share of the property value. wishes to allow the property to increase in value before it is sold. The trial court ordered the land sold because of waste. appealed and the Supreme Court of MI reversed 2. Trial courts have jurisdiction to order the sale of land for the prevention of waste if the facts so merit. However, the trial court can order a sale only if it is in the best interest of all the parties. This rule has the necessary flexibility to meet unique situations and to yield equitable solutions. In this case, though it is true that a sale would benefit , it would bring great financial loss to and therefore judicial sale is not in the best interests of all parties. The court suggests that a sale of part of the land may be equitable under the circumstances 3. A trial court shall order a judicial sale only if it is in the best interest of both the freehold tenant and the holder of the future interest 4. Notes on Baker a. Why not give Anna a fee simple instead of giving her the life estate? a. If Anna remarried, John wouldnt want Annas new husband to get the land b. Giving Anna a life estate is giving her something no one can take away from her 1. Best guess the land was income-producing + making sure the estate is hers as long as she lives (even if she remarries, she can only give life estate to her new husband for the rest of her life) 2. PROBLEM is that John failed to think about the expansion of the city and is exerting dead hand control by making decisions for people in the future based on the facts as he knew them i. Eventually small farms are no longer lucrative and the land will be valuable for other purposes while John thinks hes creating a good situation to protect Anna, hes actually locked her into a life of poverty because she has nothing more than a life estate b. SC decision appears to try and preserve the intentions of John sell whats necessary for the life tenants needs and preserve the rest for the benefit of the remaindermen

Dissent says Lide understood exactly what she was doing the restriction on alienability was for the life estate; Lide also gave other gifts to White Why would the court reverse the presumption and say that where theres ambiguity, fee simple is granted? a. If court says this is fee simple absolute but you cannot sell it meaning the court restricts alienation, then this means that you are discouraging improvement of the home and detrimental to efficient economic activities. b. Life estate is a property interest it cannot be sold and therefore might not make any improvements because she wouldnt be likely to get a return on her investments 1. One option to still make an improvement would be to get the permission of the remaindermen to make improvements 2. The problem with this is that consent of all would be required and the transaction cost of putting this deal together is so high and there is no guarantee the deal will go through. Even if theres an efficient course of action (such as getting consent from everyone and make improvement) that makes everyone better off, the improvements might not happen c. Free Rider Problem maybe remaindermen will not pay and still reap the benefit of the improvements when you have multiple parties and you need the consent of all, some will act strategically and exploit their position, often causing the deal to fall apart d. Holdouts people may act strategically to delay or postpone Life tenants want to maximize value immediately in order to get the maximum profit possible while they still have an interest in the land Remaindermen want to maximize value over time, efficiently

1.

10

c. d. e. f. g. h. i. vi.

d.

e.

Notes (201) 1. Waste the law of waste becomes relevant whenever 2 or more persons have rights to possess property at the same time or consecutively: not generating the best value a. Central idea of the waste concept is that one should not be able to use the property in a manner that unreasonably interferes with the expectations of the other b. Law of waste is designed to avoid uses of property that fail to maximize the propertys value c. Posner the law of waste emerged to reconcile the competing interests of life tenants and remaindermen. A life tenant will have an incentive to maximize the present value of the earnings stream obtainable during his expected lifetime. The tenant and the remaindermen are locked into dealing with each other creating a bilateral monopoly which usu ally has high transaction costs d. Precise application of waste doctrine turns on a number of variables a. Nature of the property interests of the competing parties b. Conduct in question c. The remedy sought e. Conduct that amounts to waste turns in part on the nature of the interests involved and on the conduct in question - courts have created 2 general categories a. Affirmative Waste liability results from injurious acts that have more than trivial effects where injurious has meant acts that substantially reduce the value of the property in question b. Permissive Waste a question of negligence failure to take reasonable care of the property 2. Hypo: O gave A for life and his nephew, B, is a remainderman- B wants to sell the land but A doesnt. a. B, remaindermans argument- Waste doctrine: were not making the best use of the land, its not generating the most profitable and economic use of the land b. As argument- should not order sale because 1. Must respect the will of O, 2. Greater market value is not always the best way to calculate the best value of the land, 3. If you say that remainderman has the cause of action and that cause must be testified by an expert witness to be proved, and if thats adopted, that would create a power to threaten a suit. B has the power to start a suit, A might just settle instead of going through all the trouble of the lawsuit. c. Trial court probably was saying that lets prefer Anna over remainder and respect the wishes of the dead. And the deterioration of the property was not done so there is no need to stop the use of the property by the life estate. We dont have to give much meaning to trial courts judging. 3. What John could have done to preserve his wishes but not create this mass? a. Create a trust b. Assign 1st national bank as Trustee which has legal title(gets no benefits) c. Trust beneficiaries can be divided as 1. Anna for life, 2. Nothing to children, 3. Remaindermen are grandchildren d. Trustee has two duties duty of care / duty of loyalty e. How does this solve problems? Trustee can only make a decision that is only beneficial to the beneficiary Trustee can look at the wishes/intent of John Sell the property and give Anna whatever is appropriate and give the rest to remaindermen Trustee can get mortgage to support beneficieries if needed The point is that by creating trust, we eliminated the litigation Trust law is well developed Even Anna can be a trustee Leasehold Estates i. Nonfreehold Possessory Estates (fee simple and life estates are freehold estates) and leasehold tenants do not have seisin ii. Term of Years an estate ending on a fixed calendar date, such as a term of 5 years or a term of 6 months 1. Common today in commercial and residential leasing Defeasible Estates i. Defeasible Estates can last forever or are able to be revoked in the case of occurrence (or non occurrence) of a certain event or the performance (or failure) of a condition 1. Most common kind of defeasible freehold estate is a fee simple defeasible (as opposed to a fee simple absolute) ii. 2 types of defeasible fees (IMPORTANT) 1. Fee Simple Determinable -a fee so limited that it will end automatically when a stated event happens a. Created by language that the transferor is conveying a fee simple only until an event happens b. Words that merely state the motive of the transferor in making a gift do not create a determinable fee c. Accompanied by a future interest a. In the ordinary case, the future interest is retained by the transferor and called a possibility of reverter (can be expressly retained or may arise by operation of law)

Trial court relies on theory of waste says this property was deteriorated and that waste occurs when there is a better economic use for the land than the use that is employed because the land isnt being given its highest and best use, its wasteful Contingent remainder- giving the right to remaindermen who are not yet born and followed by alternate contingent remainder (v. vested remainder), so giving it to Annas children was contingent remainder & giving it to the grandchildren is also contingent remainder Why did the city contact the remaindermen, the grandchildren, who did not even know of the existence of the land? a. City had to give notice although they were contingent remaindermen. They had the right to be noticed. Problem 1: poor foresight for future Problem 2: Life estate want to maximize value now v. Remaindermen want to maximize the value in the future a. Point: two parties are locked into bi-lateral monopoly and they cant transact in the market and the only person who can sell and buy is each other, and it is inefficient. was bringing the suit by arguing that waste means something else because this case is distinguished from other cases because other cases satisfied interest of both parties while selling the land here only helps one party. Courts decision: make an agreement- this is highly equitable decision. a. How do you think about this opinion? Maybe the trial court effectuated the intent of John for leaving the land and providing what is best for Anna.

11

iii.

Fee Simple Subject to Condition Subsequent a fee simple that does not automatically terminate but may be cut short or divested at the transferors election when a stated condition happens a. Not automatically terminated when the stated event happens b. Unless and until entry is made, the fee simple continues c. Created by a conveyance of a fee simple, followed by language providing that the fee simple may be divested by the transferor if a specified event happens d. Future interest retained by the transferor to divest a fee simple subject to condition subsequent is called a right of entry a. Right of entry may be expressly retained or it may be implied if the words of the instrument are reasonably susceptible to the interpretation that this type of forfeiture estate was contemplated by the parties b. Statute of limitations starts running on the possibility of reverter as soon as the determinable fee ends 1. With respect to the right of entry, theoretically the statute of limitations should not begin to run until the grantor attempts to exercise the right and is rebuffed, giving rise to a cause of action i. This may be more theory than reality in many states the statute begins to run on the right of entry when the condition occurs ii. If the right of entry is never used to retake land, the grantee may acquire fee simple absolute by adverse possession Mahrenholz v. County Board of School Trustees Appellate Court of Illinois, 1981 (208) 1. In 1941, the Huttons conveyed 1.5 acres property to the trustees of School District No. 1. The deed stated: This land to be used for school purposes only; otherwise to revert to grantors herein. Also in 1941 the Huttons conveyed remaining 38.5 acres & their reversionary interest in this property to Jacqmain. In 1959, Jacqmain conveyed this reversionary interest & the land to . When the Huttons died, their only legal heir was Harry Hutton. In 1977 Harry Hutton conveyed any possibility of reverter or right of entry that he may have had in the land to . Later in 1977, Hutton disclaimed any interest he had in the land to , the school board. The property that the Huttons conveyed to the school was used for classes until 1973. Since then, the school has used the property for storage. contends that the deed conveying the land to the school board created a determinable fee simple followed by a possibility of reverter. contends that the deed created a fee simple followed by a right of entry. sought to quiet title to school property in themselves and the trial court dismissed the complaint. The Appellate Court of IL reversed 2. By IL statute, neither a right of entry (fee simple subject to condition subsequent) nor a possibility of reverter (fee simple determinable) can be transferred by will or inter vivos conveyance, Jacqmain could not have received the interest from the Huttons and could not have acquired the land from Jacqmain. The reversion passes to Harry, the sole heir of the holder (Huttons). Thus, the court must determine what interest, if any, received from Harry. To determine whether a fee simple on condition subsequent (School board wins) or a fee simple determinable (Mahrenholz wins) was created, the court must examine the words of the deed. Upon examination of the language in the deed, it appears that the Huttons conveyed a fee simple determinable. For school purposes only suggests that the Huttons wanted to give the land only as long as it was needed and no longer. The second phrase, oth erwise to revert to grantors, seems to trigger an automatic return of the property to the Huttons. Thus, the grant indicates a fee simple determinable. Therefore, when the land ceased to be used for school purposes, Harrys interest became a fee simple absolute (because of the reverter) and Harrys interest can pass to . 3. A fee simple determinable is created by language which connotes that the grantor is giving a fee simple only until a stated event happens. The difference between a fee simple determinable and a fee subject to a condition subsequent is a matter of judicial interpretation. 4. Notes on Mahrenholz a. The IL statute says that one cant alienate his reversionary interest and can only be transferred by inheritance (divisible by intestate descendant)/ cant transfer the right to revert (minority of states) a. The IL statute provided that Harry inherited the right because the property is not devisable, it automatically went to the child based on intestate provisions b. Created based on common law maybe thought that reversionary interest is not actually an estate in land only the possibility of an estate in land and because reversionary interest is just a possibility, theres nothing one can give away c. Is this a sound idea? 1. No. When a reversionary interest is passed on to heirs, there can be multiple heirs who all have a fraction of the interest. Even if the school board wants to get the release from holders so the board will have the fee simple absolute, contacting so many holders can be expansive or impossible. 2. No. Problem with automatic reversion is that the holder of the fee might not think that he is breaching any conditions, while a court might find that he is and that the property had automatically reverted i. If the property interest has automatically reverted, you may have owners in fee simple who arent aware theyre owners because they may not know of the breach or of the courts determination of breach d. How can the owner of a defeasible fee eliminate the reversion? 1. Would require the holders of the reversion to release the interest i. Could require huge investments of resources to find all of the heirs with reversionary interest and ask for their rights potential holdout problems and the result is an inefficient use of land b. Modern trend is that reversionary interest can be alienable and devisable. c. This land to be used for school purposes only. Otherwise to revert to grantor herein. a. Strongest argument to support school board One, Constructional preference: where ambiguous language is used the court of IL would interpret it as a fee simple subject to condition subsequent; here, the language is ambiguous, so interpret it as FSCS Why? Its better to let people know who owns what at any time (FSCS = Certainty) * Fee simple determinable = uncertainty Second, Case comparison: The case in Mahrenholz, Latham (IL SC case), parallels the language of the Will of Mr. Hutton. for purposes only + Mackelvain case - page 230 b. It seems like these are strong arguments, but the court rejects them essentially. Do you find the courts reasoning compelling?

2.

12

5.

Page 211, bottom: persuasive, binding case? These cases are not binding cases because federal court decisions are precedential only for federal statute. For state law, state court is not bound by these decisions. These are so easy to rebut based on non-precedent ground. Then why are they saying this? Page 211, we believe only as long as needed and no longer. This is different from If.. then. Page 215, #2- B wants to buy the restaurant from A and add a bar. O conveys Blackacre to A under the condition that no sale of alcohol on the premise and O remains a right to re-enter the premise. a. Court can find that opening a bar can be breach, so investing is a huge risk. b. Maybe because the statute of limitation has run out, O cant do anything, so no risk. c. But youre advising a client. You dont want to tell them all the problems but want to get a solution. a. You can contact O and ask for a disclaimer for a fee. But your deal can be totally off if O or his heirs decides to go for a fee simple determinable or claim a right to re-enter. b. You can have A figure out this matter if A wants to sell it to B. d. so long as is for determinable; then the property goes back to O automatically, that makes A a trespassor. e. right to re-enter is for condition subsequent; action usually must be filed within a reasonable time (statute of limitations) f. Cannot separate the statement so long as and right to re-enter -

13

iv.

v.

Covenants 1. Conditions imposed by the grantor in creating defeasible fees must be distinguished from covenants made by a grantee a. A condition is much more onerous than a covenant because if a condition is breached, the land is or may be forfeited to the holder of the future interest b. A covenant is a promise by the grantee that a specified act will or will not be performed a. If a covenant is breached, the promisee may sue for an injunction or damages Mountain Brow Lodge v. Toscano Court of Appeal Of California, 1967 (215) 1. Toscano (), an active member of deeded property to . The deed stated that the land was conveyed in consideration of love and affection. The deed provided that the lot shall revert back to if either failed to use the lot or attempted to sell or transfer it. contends that this language is a restraint on who may use the land. As such, contends that this language is a restraint on alienation and is void. s heirs contend that the language creates a fee simple subject to condition subsequent. sued to quiet titl e as to their interest in the land. The trial court held in favor of s and the CA Appellate Court affirmed 2. A condition which prohibits the sale of land is a restraint on who may use the land and as such is void. The remaining question is whether the condition which required that use the land created a defeasible fee or whether it is a void restraint on alienation. Considering that was an active member of the lodge and that he conveyed it in loving consideration, the clause which reads the land is restricted for the use of the second party meant that the land be used for s purposes. It did not mean or intend to restrict alienation. Thus, this clause created a fee simple subject to condition subsequent. Even though a restraint on how land may be used can possibly be a restraint on who may use the land, such restraints are still allowed in CA and in other jurisdictions. To avoid confusion, the court rewrote the paragraph to discuss only the term of use as applied to real property the court decides the deed proclaims the lands use for fraternal purposes only and that this was the condition upon which the land was conveyed. The no sale or transfer provision was an invalid restraint on alienation, but the court upheld the use restriction on the theory that m eant to convey a determinable fee to rather than merely restrict alienability. The court also specifies that if the land is not used for the fraternal purposes prescribed, the land will revert to s heirs 3. The object in constructing a deed is to ascertain the intention of the grantor from the words which have been employed and from the surrounding circumstances 4. The entire clause should have been read together because the entire clause which purports to restrict the fee simple conveyed is invalid as a restraint upon alienation. Should an organization holding property under a deed similar to the one before us be disbanded one hundred years or so after the conveyance is made, the result may well be a title fragmented into the interests of heirs of the grantors numbering in the hundreds and scattered all over the place. Then entire clause is repugnant to the grant in fee simple that precedes it 5. Notes on Odd Fellows a. There are 2 provisions in the deed, one restricting alienability (restriction on sale) and the other restricting use a. wants the provisions to be read together because the restriction on alienability is void (White v. Brown: blanket restriction over alienability is not enforceable), so the court is likely not to uphold the deed (find it entirely void) b. But the court separates the restrictions the one on alienability is void but the other, a restriction on use, is valid; and the effect is exactly the same. Odd fellows cant sell it because whos going to buy it if they cant use the property due to the restriction of use stating that only Odd fellows can use it. c. Does it make sense that the 2 parts are severable? 1. Use restrictions that are this extreme are in effect, restrictions on sale if only these people can use the property, the court has eliminated all other people who might have bought the property i. Doesnt seem to make sense the court appears to be using the use restriction to achieve the same ends as a restriction on alienability ii. If was using the property, they wouldnt be bringing the suit clearly would rather have the cash than the property and so through the use restriction, the grantor has achieved what he cant actually do restricted alienability b. In many of the cases where extreme use restrictions have been upheld, it has been to benefit the public (cases cited on p. 219) and things important for civic life a. Why? Courts understand restrictions on alienation, but on the other hand, the law might want to encourage donations of real property to public uses 1. If restricts alienability absolutely, then it is wasteful, inefficient, etc because it blocks market activity 2. If restrictions on alienability are reserved for non-profits, etc., meaning that they cant sell it, theres not a huge restriction on land i. Wont have the negative effects on efficiency and marketability and its likely to generate some public benefit (because they cant sell it due to use restriction, public can still use it) 3. But if you apply restrictions to private use, then theres dead-hand control and the marketability and efficiency problems arise b. Confining Odd Fellows to its facts makes sense just looking at the holding, there are community concerns at stake (the court enforced a use restriction on a non-profit and not on private use) 6. Questions- After this case, O conveys family home to my nephew and his heirs for the use and benefits of my nephew and his descendants only. If the land ever ceases to be used by my nephew and his descendants, the land will revert to the grantor O dies leaving 3 children. Nephew is a school teacher and it costs a lot to maintain the property. He wants to sell it and consults you about his options. a. Can he sell the property? Whose consent does he need? Need consent of all three children since they have partial interests. If everyones acting selfishly, then Nephew will have to give up the maintaining of the property and it will automatically go to the children as a fee simple absolute. If they sell it as a fee simple absolute after getting the consent of nephew, they can split up the $ minus cost of selling, then Nephew can get his portion. Quintessential situation is that there might be a hold-out problem. And the transaction cost of negotiating is so great that the deal does not often happen especially in real life. b. If he cant obtain the consent to sell, does he have any other option?

14

c.

The nephew can bring in a legal action and ask for declaratory judgment (a remedy that a party may ask when hes about to do something but doesnt want to violate someone else right, so asks the court to clarify their right) because the language says use and benefits and its unclear. Children need to be notified because they have interest in this matter. Counter argument is that if we say that sale does not violate the will, then the condition subsequent if the land ever ceases to be used.. has no meaning for reversionary interest. In other words, once the property goes out of the family by sale, it never reverts back to the family. What do you do with the odd fellows opinion since it set the precedent that alienability is valid If the court scratches out the use restriction, the nephew has a fee simple absolute. So, nephew should distinguish this case from Odd Fellows or look at the purpose and make arguments that way. Gift to a non-profit organization (court uses church, historic society, etc. in the case of Odd Fellows)v. Gift for profit. Reason for restricting alienation for non-profit was to encourage people to give property for public use and honor the wishes of the dead who wanted to provide for the public. So it makes sense to use it in Odd-fellows. But in this case, too many people involved, no regards for the public, so a different case. In determining whether restricting alienability, court balances many variables. Social benefits v. Use restrictions, etc. Its a very fuzzy standard. -

15

IV.

Future Interests a. The first level of distinguishing is to determine the difference between future interests retained by the transferor/grantor and future interests created in 3rd party (other than the grantor) i. Future Interests retained by grantor (226) 1. Reversion the interest left in an owner/grantor when he carves out of his estate a lesser estate and does not provide who to take the property when the lesser estate expires 2. Possibility of Reverter a future interest remaining in the transferor/grantor or his heirs when a fee simple determinable is created 3. Right of Entry when an owner transfers an estate subject to condition subsequent and retains the power to cut short or terminate the estate, the transferor/grantor has a right of entry 4. Excutory Limitation if the grantor gives the right of entry to 3rd party then the party is excutory limitation 5. Problems in page 227. (a) O conveys to A for life, then to B and his heirs (his heirs means nothing). A has life estate B has vested remainder No reversion Future Interests created in Transferees (227) 1. Remainders a. Vested (must satisfy 2 conditions) a remainder is vested if it is given to an ascertained (identified) person(s) and it is not subject to a condition precedent apply two standards separately a. 3 Types of Vested Remainders 1. Indefeasibly(absolute) Vested Remainder at the time of the creation of the will, we can point to the remaindermen and there is nothing that can happen that can prevent the remaindermen from getting the estate A for life, remainder to B: B has indefeasibly vested remainder. If B dies before As death, its still Bs property and it will be handled according to Bs will or by intestate statute 2. Vested Remainder Subject to Open (subject to partial divestment) remainder is given to a class of people and the class is open (i.e., to my children: if there are 3 children at the time of the will drafting, each has 1/3rd share, but if 4th child is born, then each child gets 1/4th share) 3. Vested Remainder Subject to Complete Divestment looks like a contingent remainder. If theres a grant to one party and in the next clause theres a divesting condition, its a condition subsequent (rather than a condition precedent, which would make it a contingent remainder) i. Executory Interests divests a vested interest a future interest in a 3rd party that vests upon any condition subsequent except the natural termination of the original grantees rights b. Contingent remainder a remainder is contingent if it is given to an unascertained (not identified) person or it is made contingent on some event occurring other than the natural termination of the preceding estate (if there is condition precedent) c. Litigation is with vested remainder subject to complete divestment a. The question is whether if is a part of which (before the comment or after etc.) b. Remainder to B; however, if B dies prior to reaching age 21, then to C There shouldnt be no condition for remainder to B. Then you added a condition after that. It looks like a contingent remainder. then to C is executor interests Rule #1: If the first remainder is vested remainder subject to complete divestment, the second remainder is called executory interest (no meaningful difference between executor interest and contingent remainder) Rule #2: If the first remainder is contingent, then the second remainder following is contingent too. (Remainder to B if B reaches 21, if B dies before 21 to C; Think about AnnaJohn said to Anna and her children but she didnt have children at the moment) Classification of Future Interests see Classifying Future Interests Worksheet and Hypos i. Children has life estate. Q has vested interest because 2 questions meet. If there were no grandchildren, then its only contingent intere st, so as soon as a grandchild is born, it becomes a vested interest open to a class. ii. W has life estate. A, B, & C has vested remainder. iii. W for life estate and children has remainder contingent. iv. W for life estate; B&C would say surviving means surviving W. Q would argue that surviving means surviving T, the grantor. Q can use the early vesting policy argument which facilitates alienability. But B&C would rebut that modern rules dont favor early vesting anymore due to tax issues. Can Q distinguish his case from Browning? In Browning, there are other evidence to infer and make sense of the intent of the grantor, but here there isnt much to work with. Furthermore, absent any counter evidence, its hard to believe that grandparents dont want their grandkids not to inherit if their children die. In Browning, grantor was worried about Clyde but here nothing like that. v. A for life estate; to As children- vested subject open b/c A can have more children & subject to complete divestment (then, B has executor interest) b/c if none of As children survive A, then to my nephew B. Rule: Remainder can be vested subject to complete div estment & vested subject open, while nothing else can be combined. vi. Martin owns Blackacre because the executor interest doesnt seem to limit the possession to survival. vii. L argues that Qs remainder was vested remainder subject divestment and the divestment cond ition was if A left no surviving children. Since A left children, the condition did not meet. And L&Q were married, so Q has the vested remainder. R argues about the grantor wanted the property stay in the family- not so strong. The Trust (239) i. An important and regularly used device in estate planning ii. Allow settlers to arrange their assets in ways that maximize flexibility in property management as well as transfer wealth to future generations iii. The core of the trust is the separation of legal and equitable title 1. The trustee holds legal title to the trust property and manages that property for the benefit of the beneficiaries 2. Upon termination of the trust, the trust assets as they then exist are handed over to the designated beneficiaries, free of the trust ii.

b.

c.

16

d.

e.

Although the trustee is the legal owner, it is subject to orders of an equity court, which enforces the trustees duties to t he beneficiaries, who are said to hold equitable interests (interests enforceable by courts of equity) a. In the typical trust the beneficiaries hold equitable interests that correspond to the legal possessory estates and future interests iv. The trustee is a fiduciary, and thus subject to stringent duties in managing trust property 1. Most important is the duty of loyalty the trustee must act for the exclusive benefit of the beneficiaries and is not permitted to benefit personally v. One benefit of the trust settlers can protect the beneficiaries interests by making them inalienable 1. Spendthrift Trusts are possible because US courts in the late 19th century held that the common law rules against restraints on alienation can be drafted in such a way that trust beneficiaries have no power to transfer or borrow against their trust interests, and creditors have no power to reach those interests to satisfy beneficiaries debts vi. Another important development in trust law legislation permits settlers to create perpetual trusts, meaning trusts that can continue to control the disposition of wealth forever into the future In the Matter of Krooss Court of Appeals of NY, 1951 (Angel) i. Herman Krooss died in 1932 and was survived by his wife Eliese and his 2 children, Florence and John. By his will, Herman gave his residuary estate, real and personal, to his wife, to have and to hold the same during the term of her natural live, with the power to use any part of it for her support and maintenance that she deemed necessary. The disputed part of the will reads: upon the death of my beloved wife, Eliese Krooss, I then give, devise and bequeath all the rest, residue and remainder of my estate, as well as real as personal, and wheresoever situate, to my beloved children, John and Florence, share and share alike, to and for their own use absolutely and forever . In the event that either of my children aforesaid should die prior to the death of my beloved wife, leaving descendants, then it is my wish and I so direct that such descendants shall take the share their parent would have taken if then living, share and share alike, to and for their own use absolutely and forever. Florence died before the life tenant (Eliese) and left no descendants. In her will, Florence left her residuary estate i n trust to her husband and to her brother John, as trustees, to pay the net income therefore to the husband for life; on his death, the principal was to be distributed between Florences 2 nephews, Johns children, if living, and to their issue should either die before the termination of the trust. , is Florences husband and the executor of her estate. brought suit against , John, to render and settle his estates. The Surrogate Court held that Florences interest vested when her father died, subject to be divested only in the event of her predeceasing her mother leaving no descendants, that her interest passed under her will, and that , as executor, was entitled to bring the action. The Appella te Division modified the decision. Disagreeing with the Surrogates interpretation, the Appellate Division construed the will as imposing upon each of the remaindermen a condition that he or she has to survive the life tenant. Because Florence died before Eliese, the condition was not met and as to Florences share in the remainder, Herman died intestate. The Court of Appeals reve rsed the decision of the Appellate Division and affirms the Surrogate Court ii. If Florence survived Eliese, no problem and it will go to Florence. If Florence doesnt survive Eliese but left issue, then i t will go to Eliese and still nothing to the husband, . is saying that Florence has vested interest subject to complete divestment. In the second paragraph of the will says in the event., leaving descendants, then and because that condition didnt meet, no divestment was complete and it is indefeasibly vested. a. The 2nd paragraph creates the confusion and if the will had stopped with devise and bequeath all the rest to my beloved children there would not no question that the remainders were vested. b. But the further language of the same paragraph demonstrates not that the testator was rendering the vesting of the estates in his children condition upon survival of the life beneficiary, but that the testator was willing to have those estates divested only upon the combined occurrences of 2 further events: (1) the child Florence or John must die before Eliese; and (2) the child who dies must leave descendants. Only if both of these conditions are met was the remainder to be divested. c. In cases where similar language was employed in a will, courts have held that a vested remainder is created in fee subject to be divested by the remaindermens failing to survive the life beneficiary if, and only if, such remaindermen leaves issue of descendants surviving. At the expiration of Elieses life estate, the testator then gave the remainder to his children absolutely and forever. The word then must be construed to relate solely to the time of enjoyment of the estate and not to the time of its vesting interest. Because Florence did not meet both necessary requirements (which together would have resulted in divestment), Florences absolute gift remained vested and was not defeated iii. Whats Hermans intent? He didnt think about Florences husband and wanted the property to stay in the family. Think about wh at husband really gets if he wins and what John really gets if he wins. (See footnote1 pg3)That should impact the finding. iv. How would you draft this in order to avoid litigation? a. If Herman wanted to create a dynasty so everything stays in the family, then you should say The property goes to my wife for life and then to my children. If one of the children predeceases my wife leaving issue, the descendants will take it. If one of the children predeceases my wife without leaving issue, then. If both of my children predecease my wife without leaving issue, then v. A court will assume that an interest vests if the interest conveyed in a will is somewhat ambiguous as to what conditions would cause the divestment Browning v. Sacrison Supreme Court of Oregon, 1974 (Angel) i. Webb, the maternal grandmother of Franklin Browning and Robert Sacrison, executed a will that read: I give and devise to my daughter, Ada, a life estate for the term of her natural life in and to all real property belonging to me at the time of my death, excepting only the residence property at Pilot Rock with remainder over at the death of said Ada, share and share alike, to my grandsons, Franklin and Ro bert, or, if either of them be dead, then all to the other, subject to a like condition as to the use of the same or any portion of the proceeds thereof for Clyde Browning, as mentioned In 1954, Webb died. She was survived by Ada, and grandchildren Franklin and Robert. Franklin died in 1972 without issue. He did not survive the life tenant Ada, who is still alive. is the wife of Franklin (deceased) who claims that the language in the will provided for a vesting of the interest in the grandmothers property at the death of the grandmother, rather than at the death of the life tenant. is the surviving grandson who takes the position that the remainder is contingent on surviving the life tenant. The trial court held that the grandsons remainder was contingent on the death of the life tenant. The Supreme Court of OR affirms 1. , the wife is Franklyn who died and was a remainderman, wants the remainder to be indefeasibly vested, so she can claim Franklyns portion. She argues that if either of them be dead, then all to the other refers to the death of the testatrix, Kate Webb, and not the death of their mother Ada, the life tenant. So she argues that the estate vested when Kate (grantor) died. 2. , Robert who was another remainderman, wants the remainder to be contingent, so he can own it all. Hes saying that there is a condition precedent to vesting which is that they must survive the life estate. 3. How does the court resolve this? Constructional argument a. Holding: affirmed trial courts decision for the (contingent )

3.

17

V.

VI.

Language (look at other words in the will): The court relies on the textual argument made by the trial court to affirm the holding the trial court looked at a different paragraph where the testatrix expressly designates the time for vesting as at the time of my death and because that same explicit wording isnt present in this paragraph of the will, it did not apply and the interest did not vest until the death of the life tenant. c. Testatrixs intent (what did the grantor want?): finds an even stronger argument it is apparent from the will that the testatrix did not want her son-in-law, Clyde Browning, to use or obtain anything belonging to the testatrix for his benefit (note 4). If the construction suggested by the were accurate, and the grandsons had died intestate before the life tenant, Clyde would have shared in his interest as an heir of his children, clearly contrary to the testatrixs specific wishes d. Policy argument (historical preference): there has been a preference among the courts to favor the early vesting of estates (s claim); however, modern presumption is that the remaindermen must survive the life tenant and not just the testator. Historically early vesting was favored because it was thought to help commerce by facilitating alienability. However, now, unfortunate tax consequences (if the remainder is vested, when Franklyn dies the value of his estate will be taxed even though he never benefited the estate, so the grantor does not want early vesting in order to not burden the remainderman) and transferors are inclined to postpone vesting. 4. If you were the lawyer of Kate Webb, how would you have drafted the will to limit the litigation? a. Whats the first question you ask? What was the issue in this case? What do you want to happen when one of your grandchildren dies before your daughter, Ada? b. If Kate says, my grandchildren can give it to anyone they want if they die before Ada. How would you draft the will? Ada for life and indefeasibly vested for Franklyn and Robert. c. If Kate says, my grandchildren can get it only if they survive Ada and I dont want it to go to anyone else but my grandchildren. How would you draft the will? Grandchildren are remaindermen if they survive Ada. If either one doesnt survive Ada, one of the grandchildren gets everything. If both survive Ada, .. ?? d. If Kate says, I never want Clyde to get his hands on my property. What do you do?? e. What if both of the grandchildren died? Then do whatever you want to do ii. In determining whether a future interest left in a will is vested at the death of the testator or at the death of the life tenant, the court will look to the construction of the language, the modern trend, and the intent of the testator The Rule against Perpetuities a. The rule is the culmination of the long struggle between landowners who wanted to keep land within the family and the judges who tried to stand against these efforts i. Heads of families wanted to secure family land from incompetent children 1. Court accepted the notion that fathers could assess the capabilities of living members of his family, so with respect to the fathers, the fathers informed judgment was given effect 2. However, the head of the family could not know anything of unborn persons, so court did not let the heads wishes to be in effect 3. Later, judges permitted testators to extend their control beyond living members if any of the persons in the next generation was actually a minor. 4. Finally, the judges fixed the period for living as live in being + 21 years and on ii. Effect: this permission of extensive period of dead hand control represented a considerable victory for the rich desirous of controlling their fortunes after death b. Mechanics of the Rule i. First step: determine whether the future interest in question is even subject to the rule 1. Only contingent remainders, executor interests, and class gifts are subject to the rule 2. Contingent remainders + executor interests = not vested interests ii. Second step: determine whether the given interest might not vest within the perpetuity period of lives in being plus 21 years. 1. It strikes down contingent interests that might vest too remotely 2. It is a rule of logical proof: must prove that a contingent interest is certain to vest or terminate no later than 21 years after the death of some person alive at the creation of the interest 3. Validating life: a person who will enable you to prove that the contingent interest will vest or fail within the life c. Problems and note i. O conveys to A for life, then to B if B attains the age of 30. B is now 2 years old. 1. There is validating life but that person will only be 23 years old after 21 years and wouldnt be meeting the condition. ii. O conveys to A for life, then to As children for their lives, then to B if B is then alive, and if B is not then alive, to Bs heirs. Assume that A has no children at the time of conveyance. 1. No validating life at the moment, but if As pregnant at the time of conveyance, then may be this will work.? Maybe . 2. It depends on the state. If there is a pro-life rule, there is a validating life. iii. O, a property teacher, declares that she holds in trust $1,000 for all members of my present property class who are admitted to the bar. Is the gift good? Suppose that O had said for the first child of A who is admitted to the bar. 1. Yes, its just that the gift is not vested to anyone until all members pass the bar. 2. It is not vested unless A passes the bar and As child is in existence and identified. iv. O conveys to A for life, then to As children who reach 25. A has a child, B, age 26, living at the time of the conveyance. Is the remainder valid? Yes, of course. v. O conveys to A for life, then to As widow, if any, for life, then to As issue then living. Is the gift to As issue valid? 1. It is not vested unless A dies, As widow dies, and As issue are in existence and identified and all that happens within 21 years from the death of A. vi. Co-ownership and Marital Interests a. Ownership may be divided among 2 or more persons in the sense that they have consecutive rights of possession i. The division results in possessory and future interests, not co-ownership 1. Co-ownership refers to situations when 2 or more persons have concurrent rights of present or future possession b. Common Law Concurrent Interests (275) i. 3 types of concurrent interests (there are 5, but 2 are not covered) 1. Tenancy in Common separate but undivided interests in the property a. The interests of each is descendible and may be conveyed by deed or by will b. No survivorship rights between tenants in common c. Each tenant in common owns an undivided share of the whole d. Each has independent share b.

18

2.

3.

c.

d.

e. f.

g.

Presumptions i. The English common law, disliking division of land into smaller parcels, favored joint tenancies over tenancies in common 1. If an instrument conveying property to 2+ persons were ambiguous, a joint tenancy resulted ii. Today the situation is reversed; the presumption favoring joint tenancies has been abolished in all states so theres a presumption in favor of tenancies in common (except in some states where the conveyance is to husband and wife) 1. Usually the abolition has been accomplished by statutes providing that a grant or devise to 2+ persons creates a tenancy in common unless an intent to create a joint tenancy is express a. A few states have done away with the joint tenancy entirely iii. At common law, a conveyance to husband and wife was presumed to create a tenancy by the entirety, absent some clear indication to the contrary 1. Presumption still has considerable force in those states that retain the tenancy, though in some of these a conveyance to husband and wife will be presumed to create a tenancy in common or a joint tenancy Example: To A for life, remainder to B and C in equal shares. i. A for life estate; B&C indefeasibly vested remainder ii. When A dies, B&Cs remainder becomes possessory. B&C hold the property in tenancy in common. 1. Modern presumption when the language is ambiguous is that its tenancy in common. iii. During As life, B conveys his interest to Q. What happens at As death? Who owns the property? 1. Q&C have tenancy in common because B conveyed half of his property interest to Q. iv. If B didnt convey, what happens? In tenancy in common, any party can convey even before its possessory . Example: To A and B jointly. i. This is ambiguous, because there is no language for right of survivorship (this is the key). Example: To A&B as joint tenants with right of survivorship. i. This becomes Joint Tenancy. ii. A&B in possession of Blackacre. A dies leaving a will saying her interest goes to Q. 1. A cant convey her share, cant severe. 2. At the death of a joint tenant, the surviving one owns everything. iii. If A executes her conveyance during her life to Q. 1. Then A broke joint tenancy and transforms it to tenancy in common by transferring her interest to Q. 2. How can one unilaterally break joint tenancy and create tenancy in common? a. Convey the right to a third person. b. Its based on the feudal tradition based on 4 unities. iv. Suppose that A&B were married and live in one of the 17 states recognizing tenancy by entirety. A dies and left her share to Q. 1. A cant do anything without the consent of B. It prevents a party from unilaterally transferring their right of survivorship 2. Its based on the feudal conception that married couple is one. Questions on page 278. i. O to A, B, and C as joint tenants. A conveys to D. B dies intestate, leaving H as his heir. What is the state of the title? 1. When A conveys to D, did it severe/break joint tenancy and transformed it to tenancy in common? Not exactly. The rule is that what A does cant affect the relationship between B&C. B&C still has joint tenancy. 2. C owns her 1/3 still + 1/3 from B. And D owns 1/3 from As conveyance. 3. What if B had died leaving a will devising his interest to H? a. Then, C owns her 1/3, H owns 1/3, and D owns 1/3 from A. ii. T devises Blackacre to A and B as joint tenants for their joint lives, remainder to the survivor. During her life A conveys to Q and dies. What interests are created by the devise? 1. Who owns Blackacre? Think about it from Qs and Bs perspective.

Joint Tenancy a. Right of survivorship b. Joint tenants together are regarded as a single owner a. In theory, each owns the undivided whole of the property b. This being so, when one joint tenant dies, nothing passes to the surviving joint tenant(s); Rather, the estate simply continues in survivors freed from the participation of the decedent, whose interest is extinguished c. Since the original notion was that all joint tenants were seised together as one owner, the common law insisted that their interests be equal in all respects - 4 unities essential to joint tenancy a. Time the interest of each joint tenant must be acquired or vest at the same time b. Title all joint tenants must acquire title by the same instrument or by a joint adverse possession c. Interest all must have equal undivided shares and identical interests measured by duration d. Possession each must have a right to possession of the whole e. After a joint tenancy is created, one joint tenant can voluntarily give exclusive possession to the other joint tenant 1. At common law, and in many states today, if these 4 unities do not exist, a joint tenancy is not created, a tenancy in common is created 2. If the 4 unities exist at the time the joint tenancy is created but are later severed, the joint tenancy turns into a tenancy in common when the unities cease to exist d. Either tenant can terminate the joint tenancy by conveying his interest to another (while alive), which creates a tenancy in common e. If tenants in common or joint tenants cannot solve their problems by mutual agreement, any one of them can bring an action for judicial partition a. In a partition action, a court will order the land sold and divide the proceeds among the tenants Tenancy by the Entirety can be created only in a husband and wife a. Like the joint tenancy in that the 4 unities (+ marriage) are required and the surviving tenant has the right of survivorship b. Husband and wife are considered to hold as one person at common law a. Neither husband nor wife can defeat the right of survivorship of the other by a conveyance of a moiety to a 3 rd party; only a conveyance by husband and wife together can do so b. Neither husband nor wife, acting alone, has the right to a judicial partition of property held as tenants by the entirety

19

h.

i.

Bs perspective: A& B had joint life estate, so when A conveyed, A conveyed As share of life estate. That is contingent remainder, so now B owns the whole. Emphasizing joint lives for life estate and remainder. b. Qs perspective: A& B had joint tenancy, when A conveyed, it transformed to tenancy in common. Emphasizing joint tenants that its ambiguous. c. Q can respond to B, that its just a poor drafting. If you look into the intent, you can find that conveyor meant to iii. A & B are planning to be married. 2 wks before marriage, they buy a house and take title in A&B as tenants by the entirety. Years after, A moves out of the house and conveys his interest in the house to his brother C. C brings an action to partition the property. What result? 1. Since they are not divorced yet, A cant convey his interest to C. B will argue that C has no share. a. Then C can argue that it was never tenants by the entirety because they werent married at the time of purchas e. So they held the house in tenancy in common, so A can convey his right to C. 2. In common law, they were not married so there is no tenants by the entirety. 3. In modern law, the court will probably will look at the intent of the parties and assume they were trying to create tenants by the entirety. Honor the original intention by overlooking at the fact that they were not married at the time of the purchase. 4. Advice: use strawman- take the title as joint tenancy and transform it to tenants by the entirety. a. Property law is different from contract law. You cant make it to automatically become tenants by the entirety as you can do with contract law. It is questionable why you couldnt. Severance of Joint Tenancies (280) i. Riddle v. Harmon Court of Appeal of California, 1980 (280) 1. Mr. () and Mrs. Riddle, husband and wife, purchased some real estate and took title as joint tenants. Several months before she died, Mrs. Riddle had an attorney plan her estate. This attorney told her that her property was held in joint tenancy and thus, upon her death, it would pass to her husband, . After learning this, Mrs. Riddle wished to sever the joint tenancy so she could pass on her interest in the land by the use of a will. Her attorney prepared a deed which allowed Mrs. Riddle to grant an undivided interest in the property to herself. The document also explicitly stated that its purpose was to terminate those joint tenancies formerly existing: between and Mrs. Riddle. Her attorney also prepared a will disposing of her interest in the property. Both the grant deed and will were executed on 12/8/1975. Mrs. Riddle died on 12/28/1975. The trial court denied her plan to sever the joint tenancy and quieted title in . , executrix of Mrs. Riddles will, appeals and the judgment is reversed 2. It is a universal right of a joint tenant to effect a severance and destroy the right of survivorship by conveyance of his joint tenancy interest to another person, including himself. The 4 unities are essential to the creation and existence of an estate in joint tenancy and if one unity is destroyed, a tenancy in common results. An indisputable right of each joint tenant is the power to convey his separate estate by way of gift or otherwise without the knowledge or consent of the other joint tenant and thereby terminate the joint tenancy. Historically, it has been held that one could convey an interest in joint tenancy to himself as tenant in common through the use of a strawman who would then return the interest to the joint tenant (Clark v. Carter). The court finds that it should not uphold the ancient rules, deriving from English common law (which required passing the dirt clod), especially if allowing one to convey tenancy in common to himself achieves the same goal as what was permitted in the past 3. A joint tenant can unilaterally sever a joint tenancy without the use of an intermediating 3 rd party (strawman) by conveying his property interest to himself 4. Notes on Riddle a. A joint tenant could sever the joint tenancy, and Mrs. Riddle did that. So whats the problem? Why is this being litigated? The problem was that she conveyed the interest to herself not to a third person. The feudal mind/rule says that you must have a third person (grantor and grantee, cant be the same person). The reason for that rule doesnt apply anymore. b. Why would the court of appeals reverse the ancient rule? a. Its a rule that has its origin in feudal times (required the grantor and the grantee to b e different because of livery of seisin (passing a dirt clod) b. Clash between the rule grounded in feudalism and the modern rule that people should have freedom to do what they want with their property c. The rules only exist because of a blind imitation of the past and serve no other purpose in modern times c. Why might you advocate for the retention of a strawman? a. Here, the court is allowing a party to secretly sever a joint tenancy b. If Mrs. Riddles husband died first, Mrs. Riddle is the only person who knows about the conveyance to herself if this is the case, shes likely to tear up the deed because she would get the entire property because joint tenancies have survivorship c. At least with the strawman you can still commit fraud, but its much harder d. Using a strawman gives some semblance of notice: why do you deprive someone who holds the share from the right of notice? Here, the husband didnt know about the conveyance and now is forced into sharing the tenancy with a third person whom he didnt agree or even know about. Why shouldnt he have the right of notice? d. As a result of this case, the property is now a tenancy in common between Mrs. Riddles devisees () and Mr. Riddle () a. Now Mrs. Riddle wins. Should the attorney be sued for malpractice? Malpractice is a tort action and must have damages. His client won, so no damages? There are damage. If you put your client in a position where litigation might be necessary by going against the precedent, youve not done your job and that is malpractice. Doesnt matter if you have a winning argument at the end and you won the litigation. b. You could have used a strawman and made the matter much simpler and less risky. c. Whos paying for the litigation? Mr. Riddle is paying for his attorney. Mrs. Riddles estate devisees are paying from what they could have owned. Joint Tenancy Bank Accounts (289) i. Subject of much litigation primary reason is because the joint bank account is used by depositors with different intentions and for a variety of purposes 1. invites litigation to establish the true intention of the depositor 2. During the lifetime of the parties, litigation may arise over what present rights the parties to a joint account have in the sum on deposit a. In most jurisdictions, during the lifetime of the parties the presumption is that the joint account belongs to the parties in proportion to the net contribution of each party b. In a few jurisdictions, the parties to a joint account own, during the lifetime of the parties, equal fractional shares in the account ii. In recent years, payable-on-death accounts have become legally acceptable a.

20

j.

In jurisdictions that do not permit payable-on-death accounts, depositors sometimes create a joint tenancy with the intention that it be a payable-on-death account in disguise iii. Majority of jurisdictions hold that the surviving joint tenant takes the sum remaining on deposit in a joint account unless there is a clear and convincing evidence that a convenience account was intended 1. The burden of proof is placed upon persons challenging the surviving joint tenant Relations among Concurrent Owners (291) i. When parties are concurrent owners of a piece of property what are their rights and liabilities to the property and to each other? ii. Communal ownership encourages inefficient use of common property resources 1. Presumably, thoughtfully devised legal rules can help avoid this inefficiency a. The rules governing co-ownership should distribute the benefits and burdens evenly iii. Partition 1. Concurrent owners might decide for any number of reasons to terminate a cotenancy a. If the parties agree on division of the property or the proceeds from its sale, no problems arise b. But in the event that such an arrangement is impossible, recourse to the equitable action of partition is necessary a. The action is available to any joint tenant or tenant in common; it is unavailable to tenants by the entirety c. Partition in-kind: physical division of the property d. Partition by sale: sale and division of the sale proceeds

1.

21

2.

3.

Delfino v. Vealencis Supreme Court of Connecticut, 1980 (292) a. s and own real property as tenants in common. The parcel is rectangular and s house is on the extreme western end. owns an undivided 99/144 interest in the land, and owns a 45/144 interest. and are the sole owners of the property. also runs a garbage removal business on a portion of the land. None of the people involved is in actual possession of the remainder of the property. wants to develop the property into 45 residential lots. In 1978, brought an action in the trial court seeking a partition of the property by sale. also wanted the proceeds of the sale divided according to the respective interests of and . moved for a judgment of in-kind partition. After a hearing, the trial court concluded that a partition in-kind would result in material injury to both the s and . The court ordered the property sold at auction by a committee and the proceeds paid into the court and redistributed to the tenants. The SC of CT determined that the trial court erred in issuing a partition by sale and that a partition in-kind is possible. The judgment is set aside and the case remanded b. Under CT General Statutes 520500, a court is not required to order a physical partition even in cases of extreme hardship. It has long been the policy of the court to favor partition in-kind over partition by sale and a sale is permitted if a division cannot be made. Because of the presumption that in-kind partition is preferred, the burden of proof is on the party insisting on the partition by sale to show that the following conditions are met and therefore a sale should be forced: (1) the physical attributes of the land are such that a partition in-kind is impracticable or inequitable and (2) the interests of the owners would better be promoted by a partition in sale. In this case, there are only 2 competing ownership interests and a physical division of the property is entirely feasible. In considering whether the best interests of both parties are met by partition in sale, the trial court erred when it held only that the sale would be in the best interests of the (reasons for benefit on p. 295). s contention that was using the land illegally is incorrect and should be permitted to maintain her business there. The interests of both parties must be considered and not just the economic gain of one of the parties that would result from the sale . has been in actual and exclusive possession of part of the property and has made it her home and the source of her livelihood. A sale would take the s home away an d could potentially deprive her of a livelihood. A partition for sale would violate law, logic, reason and fairness, and is therefore not necessary in this situation c. A partition by sale should only be ordered if the physical attributes of the land in question are such that a partition is impracticable or inequitable, and the interests of the owners would not be promoted by a partition by sale d. Notes on Delfino a. Why is that when one tenant asks for a partition it is always granted despite the other tenant(s) s objections? 1. Bilateral monopoly (when 2 disagree, neither party gets what they want) when 2 parties are locked into dealing with each other, each can only get what he needs from the other and it creates transaction cost problems (its not the free market, so theres no pressure that forces one party to change his position. Sometimes a party will start positioning himself in such a way that theres no negotiation possible and the parties are stuck) i. Posner there may even be a situation that makes both of the parties better off, but because of the standstill created by the bilateral monopoly, the best scenario is not an option ii. If one party is asking for a partition, the land isnt being used to its maximum economic potential b. If the rule is that the land should be used for its highest and best purpose and here the highest and best interest is building a subdivision, why does the court reverse the holding that the s cannot have a partition by sale? 1. Trial court looked at the situation and objectively determined the highest and best use for the land because was standing in the way of the development and the trial court ordered the sale 2. The appellate court comes at this on a more subjective level believes the trial court disregarded other important factors like s emotion attachment, s livelihood, etc. : value could mean what each party attaches i. However, its very difficult for a court to determine whether a person is being honest about their subjective value of the property by sentimental attachment or if the person is using sentimental attachment just for the purpose of prevailing in the case ii. Do the above mean its wrong for the court to take emotional attachment to property into account? Probably not. Here, theres evidence that supports a true atta chment to the property iii. The court cant know for sure that someone places a value on the property greater than the market share, but its possible and here, the recognition of that risk is small because theres no possible exploitation of the other party iv. Also, if gets the partition in-kind she has requested, the partition will not leave with more than she is entitled to and the property will be partitioned no matter what v. If there is partition sale, she can buy out the property? Her subjective value of the property may not be the same with the market value and she may not really have the ability to finance the purchase. Its disturbing to force her to buy out and to come up with the money when there is no fault of her own except that she disagrees with the concurrent owner. c. What happened after this case? She really came out of a very bad shape. d. The point: When there is a dispute, the court will intervene and will rule for partition. The question is what kind of partition! e. What other way did the appellate court say the trial court erred? 1. The trial court made a determination not based on the facts. i. The s attorneys should have gone directly to the planning board to find out if s use of the land was permissible. Instead, the trial court allows for speculation that it was not legal use of the land Notes a. Although it is usually said that partition in-kind is preferred, the modern practice is to decree a sale in partition actions in a great majority of the cases, either because the parties all wish it or because courts are convinced that sale is the fairest method of resolving conflict b. Ark Land Co. v. Harper (298)- favoring partition in-kind

22

c.

d.

Partition-by-sale, the court noted, can work hardship on owners unwilling to sell because they have emotional attachments to the land, as in the case of a homestead; money alone cannot compensate for the sentimental losses that the sale would entail b. The fact that the economic value of the land as a whole would be less if it were partitioned in kind is relevant but not dispositive, especially in cases of longstanding ownership coupled with emotional ties to the land. Johnson v. Hendrickson (298)- favoring partition by-sale a. The court ordered partition by sale under a statute authorizing sale if physical partition cannot be made without great prejudice to the owners 1. a sale may be ordered if it appears to the satisfaction of the court that the value of the share of each cotenant, in case of partition, would be materially less than his share of the money equivalent that could probably be obtained for the whole 2. It is a matter of common knowledge in this state that the division of this quarter section of land, located as it is, into 4 or more separate tracts would materially depreciate its value, both as to its stability and as to its use for agricultural purposes dividing the land would make it useable 3. The fact that it would be an advantage to appellants to have the farm partitioned according to their demands because of their ownership of adjoining land is immaterial i. The court gave no weight to the family staying in the homestead Example: 10 acre property. A development wants to buy the property and willing to pay 1.5 million dollars. The house is worth $300k. If sold as a whole the land is worth $500k. A brother wants to sell. A sister doesnt want to sell. a. If brother goes to court, hes going to ask for a partition by sale- sell the whole 10 acres and divide the proceeds. 1. Will argue that that is the most profitable and beneficiary way to the use of the land since the developer is willing to pay 1.5 million for the purchase of the whole lot 2. Johnson v. Hendrickson will support- what one will get by selling it by piece is materially less than the share of money equivalent that could probably be obtained for the whole/whole is greater than addition of parts (he can only get 300k when he could get 500k or even 1.5 M if sold as a whole) 3. Hendrickson case also considered sentimental value, but notwithstanding that, undermining economical value is not fair for others who want to sell 4. Delfino is distinguished- it said selling must be in the best interest of all parties; here, selling the land and with the money, the sister can buy another house. + in Delfino, the was running a family business and it was their livelihood and court cared about that, but here even if sale, it doesnt hurt the sister & if she really likes the house, then let her buy the house back later. 5. Ark land is distinguished- the family wanting partition in kind has been living there for almost a century. The extent of emotional attachment is different. + Ark land had a large corporation who wanted to buy out personal property but here its two siblings. 6. Questioning the validity of the emotional attachment- there is a split of authority from courts and its not ok to say anybody with the emotional attachment should get it. Because parties are locked into a bilateral monopoly, parties are locked into dealing with each other and market value here doesnt work because what is worth to the sister is not the s ame with the market value. Cant really tell whos really claiming it or whos faking it. 7. Partition by sale is easier for the court-judicial economy and efficiency b. In court, the sister will ask for a partition in kind divide the property physically according to the share. 1. Will argue that as was in Delfino and Ark land, there are more things to consider when valuing the best use of the property than just money. Emotional attachment is also important and selling the entire land to the developer is not in the best interest of all parties. 2. Delfino standard: 1) must be practical to divide it up & 2) all parties interested must be best represented-not only mean financial interest 3. Johnson is distinguished: the land the Hendrickson sons lived and asked the court to rule on partition in kind was not directly given to them by the original owner. They were merely enjoying the house and the land because that was given to their wife/mother. Johnson court would have been more favorable to consider emotional attachment or partition in kind if the mother/wife was claiming that. 4. In Hendrickson, it was divided to 6 people, so according to Delfino standard, it is less practical to do partition in kind 5. Then the court can appoint a committee to divide up who much share each party would get. this also has unexpected risk 6. Maybe since the sister is wealthy, she can do partition in sale and buy out the land and that may be less risky a.

23

iv.

Sharing the Benefits and Burdens of Co-ownership 1. Spiller v. Mackereth Supreme Court of Alabama, 1976 (300) a. (Mackreth) and (Spiller) owned a building as tenants in common. A lessee, Auto -Rite, was renting the building but later vacated, and when moving out, Auto-Rite removed the locks. began using the building as a warehouse and acquired new locks to secure the merchandise stored inside. wrote a letter to (dated 11/15/1973) demanding that either vacate the building or pay the rental value. did neither and brought suit. The trial court awarded $2,100 in rent. appealed. The SC of AL reversed b. The general (majority) rule is that in absence of an agreement to pay rent or ouster a cotenant, a cotenant in possession is not liable to cotenants for the value of use and occupation of the property. Each cotenant has the right to occupy and unless the one in actual possession denies the other the right to enter, or agrees to pay rent, nothing can be claimed for such occupation. In addition to the letter, relied on changing the locks, however, there is no evidence that changed the locks to keep out or that was ever denied access to the building. The key here lies in the definition of the word ouster. Ouster can refer to either 1) the beginning of the running of the statute of limitations for adverse possession or 2) the liability of an occupying cotenant for rent ($) to the other cotenant. The adverse possession aspect of the word is precluded here because has acknowledged the existence of the cotenant relationship by filing a bill for partition. With regard to liability for rent, ouster is evidenced when an occupying cotenant refuses a demand of other cotenants to be allowed into use and enjoyment of the land. Here, s letter only demanded that vacate or pay rent, and did not state any desire on the part of to enter the building. The letter was insufficient because , as an occupying cotenant holds title to the whole property and may rightfully occupy it until , another cotenant, asserts possessory rights. Without evidence that intended to exclude the other cotenants, is not liable for rent c. In the absence of an agreement to pay rent, a cotenant in possession is not liable to his cotenants for the value of his use and occupation of the property unless there is ouster (when cotenant out of possession tries to enter the property and cotenant in possession denies it) 2. Notes on Spiller a. According to this court, how does one cotenant establish ouster? a. Physical attempt to enter the property b. Case-by-case analysis in front of the court to determine whether ouster has occurred b. Ouster is very difficult to establish a. Encourages confrontation b. Seems inefficient c. Expansive c. For example; Mackereth writes a letter Dear Spiller, I have Crunch fitness center expressing interest to rent the premise. Please vacate or move out. Is this ouster? What if the letter says vacate and I want to list the premise with the realtor? d. Advantages to the majority rule (that absent an agreement to pay rent, when a cotenant is in exclusive possession of concurrently owned property, he does not have to pay an appropriate share of the rental value to the cotenants out of possession) a. Most cotenancies are inherited residential properties and it seems to make sense when youre talking about residential cotenancies 1. Ex. If a house is owned by cotenant-siblings, and one of them is in the home taking care of the ailing mother, we wouldnt want this sibling to have to pay rent to the others would completely go against the grantors intentions 2. Most people do not think that they owe rent to their siblings by using the residential inheritance b. On a commercial level, it doesnt seem to make sense. The majority rule probably came from cases after cases dealing with residential use so applying this directly in commercial case may not fit quite right. It just goes against a common notion of fairness that a cotenant should not gain anything valuable while the other is getting something out of the property ( is basically storing a junk in the building when could rent it out and make some money. would have to pay some amount anyway i f he wants to store it to somewhere else, and by paying only of the rental value, isnt still getting a good deal or at least paying less than what he would pay if he rents another premise of the same size and condition and pay rent there?) c. Likely to result in more partitions the cotenant out of possession is getting nothing while the cotenant in possession uses the building, therefore, the cotenant out of possession is likely to request partition so that he receives something from his ownership e. Minority rule where the cotenant in possession is using the property in full, the cotenant not in possession is entitled to the rent without establishing ouster, but only after a demand by the cotenant not in possession to vacate or pay rent is made a. Notice by having to physically arrive and try to kick out the tenant in possession, there needs to be a moment where the cotenant out of possession comes in and is denied f. Ouster for adverse possession according to Spiller, the cotenant in possession would have to make the claim of complete ownership, giving notice to the cotenant out of possession that the occupant is denying outright, the out of possession cotenants ownership of the property a. Without this notice, theres no adverse possession the cotenant in possession can enjoy the property because he has a right to do so until the cotenant out of possession decides not to let the occupant reap the benefits anymore g. Hypo: A&B are cotenants. In 1930, A leases to X for 40 years. In 1945 A dies and X con tinues to pay rent to As heirs. In 1965, B tries to enter/tries the possession. Bs asking for A to vacate. If A oust B, whats gonna happen? A will have to pa y. And how much? If cotenants wants to share the rent income, you just have to bring accountant. But if cotenants oust another cotenants, he will have to half of the market value. Now, As heirs will have to give B half of the market value which is much higher now that what they got before. A will argue that B is not cotenants using adverse possession argument. A has been acting as a sole owner well beyond a statutory limitation. B will argue that he was never on notice of the hostile claim but just agreed with A to do what he wanted to do. A will argue that A paid property taxes, the amount was negligent, etc. But still a weak case for A. What if in 1956, B attempted to oust. Then ???

24

v.

Swartzbaugh v. Sampson Court of Appeal of California, 1936 (303) a. Mrs. Swartzbaugh () and her husband owned, as joint tenants with the right of survivorship, 60 acr es of land planted to bearing walnuts. In December 1933, Sampson started negotiations with s husband for the lease of a small fraction of the 60 acres for a boxing pavilion. At all times, objected to the lease. An option for a lease, dates January 5, 1934, was executed by Mr. Swartzbaugh and Sampson (). was injured in February, 1934, and was confined to her bed for some time. The lease was executed by Mr. Swartzbaugh and on February 2, 1934, and a second lease for property adjoining the boxing pavilion site was also signed after this date. refused to sign these documents and her name does not appear on any of them. The walnut trees were removed from the leased premises against s wishes. then went into possession and built the pavilion. filed suit on June 20, 1934. At trial, testified that she received no part of the $15 monthly rent, a rate which she considered too low when compared to the $10,000 needed to build the arena. also claimed that she did not want the arena on her land because women and liquor followed. filed for nonsuit and the motion was granted. appealed and the CA Appellate Court affirmed b. It is a general rule that the act of one joint tenant without express or implied authority from the consent of his cotenant cannot bind or prejudicially affect the rights of the latter. If so, one cotenant may recover from the tenant of his cotenant the reasonable value of the use and enjoyment of his share of the estate, if the tenant under lease refuses him the right to enjoy his moiety of the estate. In a joint tenancy, each joint tenant owns an equal interest in, and has an equal right to possession of, the whole property. This court has held that each joint tenant has the right to convey or mortgage an equal share of the property, and can also pledge his interest in the property to another person. In addition, a joint tenant can make a lease to his share of the property, and such a lease would be valid as to his share. Even a lease to all of the joint property by one joint tenant would be a valid and supportable contract as far as the lessor in the joint property is concerned (Lee Chuck v. Quan Wo Chong & Co). Thus, the leases from Mr. Swartzbaugh to are valid and existing contracts which give the same right of possession of the leased property that Mr. Swartzbaugh had. A joint tenant can recover exclusive possession of joint property from a cotenant. Because has effectively the same rights to the leased property as had prior to the transaction, cannot cancel the leases. may fear that she may lose her interest in the premises by prescription. This fear is unfounded because a lessee ca nnot generally dispute a landlords title or claim adverse possession while holding the property under the conditions of a lease. There is no evidence that is holding adversely to , nor that ever demanded entry into the premises c. A joint tenant, during the existence of a joint estate, has the right to convey or mortgage his interest in the property, even if the other join tenant objects d. Notes on Swartzbaugh a. Other options available to Mrs. Swartzbaugh 1. She could bring a partition action for the entire 60-acre tract, or just the portion being leased by 2. She could extract money and get of s contractual obligations, but only if she can establish ouster i. The lease doesnt seem to reflect fair market value, so Mrs. Swartzbaugh has a good argument for bringing up the amount is paying as unreasonable 3. If this had been a tenancy by the entirety, the lease would not be valid because it forbids conveyance or lease by one party without the express permission of the other party (CA doesnt have tenancy by the entirety) b. It may seem counter-intuitive that should go along with the decision of her husband, the cotenant of the joint tenancy. But remember that each cotenant has equal right to convey or mortgage his interest in the property with or without the consent of other joint tenant. Accounting for Benefits, Recovering Costs (308) 1. Rents and Profits a. In all states, a cotenant who collects from 3rd parties rents and other payments arising from the co-owned land must account to cotenants for the amounts received b. Absent ouster, however, the accounting is usually based only on actual receipts, not fair market value c. In some jurisdictions extraction of minerals or cutting of timber is deemed waste, even though such operations are permitted to the owners of a fee simple a. One cotenant is entitled to an injunction forbidding the extraction of minerals, even though a majority of cotenants favor extraction 2. Taxes, Mortgage Payments, and Other Carrying Charges a. A Cotenant paying more than his share of taxes, mortgage payments, and other necessary carrying charges generally has a right to contribution from other cotenants, at least up to the amount of the value of their share in the property b. The principle behind this the protection if the interest of each cotenant from extinction by a tax or foreclosure sale imposes on each the duty to contribute to the extent of his proportionate share the money required to make such payments this qualification is not uniformly applied 3. Repairs and Improvements a. Necessary Repairs in most jurisdictions a cotenant making or paying for this has no affirmative right to contribution from the other cotenants in the absence of an agreement a. General rule interests of someone making necessary repairs are to be protected if physical partition is impossible, a sale will be ordered and if the cotenant has made necessary repairs, that cotenant will get the cost of repairs and the value of the improvement 1. Policy reason behind this we want to encourage necessary repairs and cotenants to keep-up their properties. If a cotenant knows that hell get his money back in a sale, theres an incentive to make necessary changes b. Improvements as with repairs, a cotenant has no right to contribution from other cotenants for expenditures for improvements; beyond this, and unlike the case of repairs, no credit for the cost of the improvements is given such in an accounting or partition action a. The general rule is that the interests of the improver are to be protected if this can be accomplished without detriment to the interests of the other cotenants (get money back for the increased value). 1. If property is physically divided pursuant to a partition action, the improved portion is awarded to the improving cotenant if such a distribution would not diminish the interests of the other cotenants as they stood prior to the making of the improvements 2. Alternative remedy where physical partition is possible but would jeopardize the interests of the improver by awarding improvements to cotenants who did not contribute to their costs is to divide

3.

25

3. 4. c.

vi.

Marital Interests a. 2 different systems of marital property 1. English (common law) System husband and wife have separate property; ownership is given to the spouse who acquires property i. accepted in the large majority of US states ii. Has come under pressure to reform itself so that the results resemble those reached under a community property system 2. Communal System husband and wife are a marital partnership and should share assets equally i. Adopted in states influenced by French or Spanish law ii. Idea of treating husband and wife as an economic unit has more or less triumphed when spousal property is divided upon divorce iii. Has less effect on the division of property at the death of a spouse The Common Law Marital Property System 1. During Marriage (The Fiction that the Husband and Wife are One) a. The English property system, feudal in origin, mirrored the need of the patriarchal landed to class to keep their estates in tact and under control of a single male a. At the instant of marriage, a woman moved under her husbands protection or cover and ceased to be a legal person for the duration of the marriage b. Husband and wife were regarded as one, and that one was the husband 1. Except for clothes and ornaments, all personal property owned by the wife at the time of the marriage or acquired thereafter, including earnings, became the property of the husband 2. The husband had the right to possession of all the wifes lands during marriage, including land acquired after marriage i. Right was alienable by the husband and reachable by his creditors 2. Beginning with Mississippi in 1839, all common law property states had, by the end of the 19 th century, enacted Married Womens Property Acts a. These statutes removed the disabilities of covertures and gave a married woman, like a single woman, control over all her property b. First push for equal rights if you cant own property you cant be an autonomous person (thus the right to vote was meaningless) 3. Sawada v. Endo Supreme Court of Hawaii, 1977 (313) a. D and his wife owned a parcel of real property as tenants by the entirety. On 11/30/1968, Ps were injured when struck by a car driven by D. On 6/17/1969, one of the Ps filed her complaint against D for damages. Mrs. and D conveyed the property to their sons by a deed dated 7/26/1969. The other P filed her suit against D on 8/13/1969. The complaint and summons in each suit was served on D on 10/29/1969. The deed from D and his wife to their sons was recorded on 12/17/1969. The sons paid no consideration for the conveyance. Both sons were aware at the time of conveyance that D had been in an auto accident and carried no liability insurance. D and wife continued to reside on the premises they conveyed to their sons. On 1/19/1971, judgment was entered in favor of the first P against D for $8,846.46. Judgment was entered for the other P against D for the sum on $16,199.28. The wife of the D died 10 days later and was survived by D. Ps brought suit to set aside the conveyance of the marital property (invalidate it) by deeming it a fraudulent transfer (a transfer that is made with the intent to defraud creditors) and satisfy the judgment. The trial court refused to set aside the conveyance and the Ps appealed. The SC of HI affirms the decision of the trial court b. Under the Married Womens Property Act, the interest of a husband or wife in an estate by the entireties is not subject to the claims of his or her individual creditors during the joint lives of the spouses. This jurisdiction has long recognized the tenancy by the entirety and its basis on the legal unity of husband and wife and their single ownership of the estate. Because the spouses are considered to take the whole estate and each have all the privileges that ownership confers, neither spouse has a separate divisible interest in the property that can be conveyed or reached by execution of a judgment. The spouses can convey the property jointly and neither husband nor wife has a separate divisible interest in the life property held by the entirety that can be conveyed or reached by execution. Hurd v. Hughes (DE) held that each spouse owns the whole while both lives the creditors of either spouse cannot during their joint lives reach by execution any interest which the debtor had in land so held. It is not unfair to the creditors of either spouse to hold that the estate by the entirety may not, without the consent of both spouses, be levied upon for the separate debts of either spouse. And if the debt arises subsequent to the creation of the tenancy by the entirety, the creditors had notice of the characteristics of the estate which limited the creditors right to reach the property. From a standpoint of public policy, the surviving spouse must be protected from the others improvident debts. The property held by spouses in a tenancy by the entirety is often the single-most important asset of the family unity. Property is crucial to private owners who can use if for loans, etc. and as long as it remains whole during the joint lives of the spouses, it is always available in its entirety for the benefit and use of the entire family. To allow 3rd parties to become a joint tenant would severely hinder the use of this asset, as it would put a cloud on the title and make it virtually impossible for the spouse to utilize the estate for other advantageous purposes (as security for loans for family expenses such as on education, etc) c. An estate by the entirety is not subject to the claims of creditors of only one of the spouses because neither spouse acting alone can transfer his interest d. If the wife takes equal rights with her husband in the estate, she must also take equal disabilities and the judgment that creditors of either spouse may levy and execute upon their separate rights of survivorship. The separate interest of the husband in entireties property, at least to the extent of his right of survivorship, is alienable by him and subject to attachment by his separate creditors, so that a voluntary conveyance of the husbands interest should be set aside where it is fraudulent as to such creditors e. Notes on Sawada

the property but order payment (owelty) from noncontributing cotenants to the improver in an amount equal to the formers share of the enhanced value of the property resulting from the improvements Where improvements cost more than they yield in terms of increased sale or rental value (or when they actually diminish the value), the improver bears the full downside risk When improvements increase value beyond their cost, the improver gets the full upside the total increase in value (rather than being required to share it with the other cotenants)

26

a.

b.

c.

d. e.

vii.

Notes (318) a. Divergent views have been taken in various jurisdictions as to the effect of the Married Womens Property Acts on the tenancy by the entirety a. Interpretive variations regarding the effect of the Married Womens Property Acts on the tenancy by the entirety have important consequences for general creditors because a creditor can reach only such property as the debtor can voluntarily assign 1. This exception from creditors is one of the main reasons for the survival of the tenancy by the entirety it serves to protect the family home as well as other property from transfer by one spouse and from creditors of one spouse i. There is one creditor that can reach the interest of a debtor spouse in tenancy by the entirety property: the IRS ii. Or, upon proof that property is used for an illegal purposes, property itself is forfeited, except for any interest in an innocent owner 2. If the property in Endo case was not family home but was an investment property, should the outcome different? What about it under Group 3 states? In HI, if youre smart, you will put everything in property in entirety as soon as you are married, so creditors cant take anything. Isnt this holding a slippery slope? Termination of Marriage by Divorce 1. At common law, upon divorce, property of the spouses remained with the spouse holding title. Property held by the spouses as tenants in common or as joint tenants remained in such co-ownership a. Because the unity of marriage was severed by divorce, property held in tenancy by the entirety was converted to a tenancy in common

4.

Fraudulent transfers: give away something less than full consideration- the issue in this case was whether the conveyance of the property to the children was fraudulent transfer 1. Hard to prove: Looking for transfers with no consideration that are made when the transferor may face liability thats greater than their assets, or its a transfer that leaves them insolvent, transfer of substantially all of their assets Before the Married Womens Property Act, wife had no ability to hold a property in her own name during the married time. 1. Is that good? Women would never be responsible for husbands fault then? 2. Whats the problem? You dont have any credit and autonomy. You cant be autonomous. 3. Even after the enactment of the Act, most women didnt have money or resources to go out and buy property. This was mainly protecting wealthy women. This is an issue of 1st impression before the court has to determine if creditors in HI can assign the interest of one of the tenants who is part of a tenancy by the entirety 1. If the creditors could have attached s interest in the house, then s transfer of the house is a fraudulent conveyance 2. Court looks to what other states are doing and determines that the transfer is legitimate and the creditors cannot attach i. Group 1 the interest of could have been attached, but his wifes interest was not attachable: doesnt exist anymore ii. Group 2 if the wife is the tortfeasor, the wifes creditors can attach the right to possession and the right of survivorship, but cannot get in the way of the husbands right of survivorship. Under this group, there can be no interference with the husbands rights: best approach for creditors, creditors can right to possess everything from the spouse, if the court followed this law, creditors clearly would have won. iii. Group 3 cannot attach possession or right of survival of either party adopted by this court, so creditors have no right. iv. Group 4 creditors can attach the rights of survivorship: If two s were not spouses but brother and sister holding the property as a joint tenant, creditors could attach and take the interest of one cotenant. Logic of the court doesnt seem to make much sense here only talking about voluntary creditors not being able to attach the interest of the tortfeasor because voluntary creditors would have notice of the tenancy by the entirety 1. But the s here were not voluntary creditors, they were victims of a tort inflicted by the shouldnt they be entitled to something? i. Courts reasoning is that they should think about it before they extend credit. They were talking about voluntary creditors. ii. In HI at the time there was very little personal property ownership and possession was something the state was trying to encourage, so at the time it made sense to prevent creditors access iii. This is preferring married relationship and protecting that against non-married, etc. relationship. So the courts saying that it matters who hits the victim. If married couple, the injured person should pay, but if anyone else, then get money from the reckless hitter. iv. Most states (NY, NJ) take Group2 rule and prefer creditors right, and they are doing fine. So why do you think some states follow Group 3? Maybe the injured person was a breadwinner and now cant make living for the family. But under group 3 law, the court will say that if the injuring party owns the property by entirety, then injured person cant get the property and be paid. Does that make sense? v. In HI, legislators had to intervene to this extent in order to encourage establishing property ownership because Polynesian culture did not have individual ownership and a few people owned massive land and leased it out to others. vi. Why not make Group 3 as an exception? The court can say its attachable by tort victims but not attachable by non-voluntary creditors. So find a mid-point. This creates fairness and protects both families.

27

2.

3.

The wife was usually entitled to a continuation of support (alimony), though it might be denied her if she was at fault Common law largely ignored the wifes contribution of services in the home Class discussion: Whoever had the title of the deed or account was the owner of the property. Husband held title of property. Womens responsibility was obedience and household duties. Mens responsibility was duty of financial support. So, at divorce, people keep their own property. It sounds good, but in reality, husband kept property and had to pay alimony after divorce as well as child support. In the last 30 years, dramatic changes have taken place in divorce law a. No-fault divorce and equitable distribution brought with it changes in property division upon divorce a. Even if women dont work outside, women at home also does her part. SO judge should split it up in 2 equal parts first wave of feminism b. Under the rule of equitable distribution, property is divided by the court, in its discretion, on equitable principles (doesnt matter whose fault the divorce is resulted or who wants the divorce) 1. Many equitable division statutes authorize a court to divide all property owned by the spouses , regardless of the time and manner of the acquisition 2. Other statutes authorize a court to divide only marital property i. This approach is based on the principle underlying community property that marriage is a partnership and property acquired from earnings of the spouses during marriage should be equally divided upon dissolution of the partnership ii. In equitably dividing property, there is a movement toward equal distribution of marital property upon divorce b. The idea that marriage involves a lifelong obligation of the husband to support the wife after divorce has also been discarded in modern divorce legislation Why the change of the law of dissolution if it ends up leaving some wives worse off than the common law system? a. Change spurred by feminism when divorce law changed, the attempt was for women to be seen as equals, so the law provided a 50/50 split of all property, looking at equal rights alone a. Problem with the 50/50 split the women arent always prepared to go out on their own, having devoted most of their time to raising kids and maintaining the household. Giving women 50% of the property isnt actually fair because many of the women dont have the skills to go out and make money all the time they could have spent developing their skills was spent in the home and their earning capacity is gone 1. As a result, in the 1970s and 1980s, women (with children) were plunged into poverty and barely getting by b. Class discussion: in community property states (such as CA, all property coming to marriage during the marriage is divided up into 2 50% parts), this was never an issue. b. c. d.

28

4.

5.

6.

In re Marriage of Graham Supreme Court of Colorado, 1978 (322) a. and were married in 1968. Throughout their 6-year marriage, was employed full-time as a flight attendant, and continues in that profession. worked part-time for most of the marriage, and devoted his time to his education. attended school for 3 years of the marriage, acquiring a BA and an MBA. After graduation, obtained a corporate position at a starting salary of $14,000/year. During the marriage, contributed roughly 70% of the total income, which was used for family expenses and for s education. The couple accumulated no marital assets during the marriage. also did the majority of the cooking and other housework. Both persons filed jointly for divorce in 1974. made no claim for maintenance or for attorney fees. The trial court found that, as a matter of law, an education obtained by one spouse during a marriage is jointly-owned property to which the other spouse has a property right. was awarded $33,134 of s future earnings. The court of appeals reversed and the Supreme Court of CO affirmed the court of appeals b. The purpose of the division of marital property is to give each spouse the property that fairly belongs to him or her. There is no rigid formula that the court must follow in doing so. However, there are limits on what actually constitutes property (The issue is is advanced degree a property?). An educational degree is not encompassed even by very broad views of the concept of property. A degree is of personal value to the holder, terminates on the death of the holder, and is not inheritable. It cannot be assigned, sold, transferred, conveyed, or pledged. The degree is an intellectual achievement that may assist in the future acquisition of property, but it is not property in and of itself. A spouse who provides financial support while the other spouse acquires an education will have that contribution taken into consideration when marital property is to be divided. Here, though, no marital property has been accumulated by the couple c. An educational degree is not property, and therefore is not subject to division upon divorce d. In a practical sense, the most valuable asset acquired during the Grahams marriage was s increased earning capacity; undeniably the result of s obtaining the 2 degrees while married. s earning mounted to an investment in that she took on the brunt of financial responsibilities so would have the time and funds necessary to get a degree. In cases such as this, where the wife works to educate her husband but is rewarded with a divorce after the husband attains a degree, the court should go beyond narrow concepts of property in order to promote equity. The law of torts recognizes that the deprivation of future earnings is something that can be compensated for when impaired or destroyed. Thus, should be compensated for the loss of future earnings of her husband, which were made possible by her investment in his education e. Notes on Graham a. Under common law before the divorce law revolution, the (wife) probably would have gotten alimony and been better off. b. s argument: what I did was putting my money into husbands education and thats my investment, so I should get a portion of his future income. c. Trial courts mistake: 1. Didnt take the domestic work into account when calculating, 2. Had husband pay wife over time although discounted for present value d. A number of other states have agreed with the NJ court in Mahoney v. Mahoney, in which the court held that a professional degree is too speculative in value to be considered marital property. The court held that the working spouse should be awarded reimbursement alimony. With this ruling, the working spouse would be repaid for covering the costs of the spouses education, travel expenses to and from school, household expenses, and any other means of support he or she provided while the other spouse earned a degree. 1. Some states have rejected this decision by the enactment of statutes specifically mandating a division of educational benefits. Problem is whether or not to reimburse working spouses for the income the other spouses might have contributed if they had not devoted all their time to education e. Argument that Graham was decided correctly (favorable to husband) Uniform Dissolution of Marriage Act property is tangible and must be alienable (with a market value) to be divided; earning potential is not property; future value of a degree is too speculative; no fault marriage means theres no wrong thats compensated (as opposed to tort where a pays because he did something wrong) because the husband hasnt done anything wrong, theres nothing to be redressed; marriage and business should be kept separate (this might devalue marriage because its dangerous to talk about marriage the same way you would talk about buying a new coat); she can also work and has earning power, why pay alimony? f. Argument that Graham was decided incorrectly (favorable to wife) tangible is too narrow of a term; the degree will generate money and advanced earning potential is recognized as having value; in tort damages, courts look at earning potential; there was an implied promise that would benefit from her hard work (dissent says maybe she can recover in quasi contract) and there was an implicit understanding that the wife would support the husband so that he could get a degree and both would be better off and the wife should not be punished for not getting the agreement in writing; even giving reimbursement alimony cant make her whole because that cant account for opportunity cost; if the interest was not degree but a land, she would get some from the appreciation of value, so should in this case; g. Valuation problem: how do you put number on his future earning potential? What if he doesnt want to get a corporate job but want to work for a non-profit? Notes (327) a. Almost all courts that have ruled on the issue agree with either Graham (degree is not marital property, supporting spouse not entitled to anything) or Mahoney (reimbursement alimony). NY is an exception a. OBrien v. OBrien the court had before it the question of whether the husbands medical license constituted marital property within the meaning of the states equitable distribution law 1. The court held that an interest in a profession or professional career potential is marital property which may be represented by direct or indirect contributions of the non-title holding spouse, including financial and non-financial contributions made by caring for the home and family the professional license should be considered marital property i. The court rejected the argument that reimbursement (Mahoney) was an adequate remedy the court viewed reimbursement of financial contributions as inequitable to the supporting spouse Elkus v. Elkus Supreme Court of New York, Appellate Division, 1991 (328) a. and her husband were married in 1973. During the marriage, achieved tremendous success. s income went from $2,250 in 1973 to $621,878 in 1989. became an internationally known recording artist and performer, won numerous awards, and performed for POTUS. During the marriage, traveled with her, attending and critiquing her performances

29

and rehearsals throughout the world. photographed for album covers and magazine articles. served as s voice coach and teacher for 10 years of the marriage and took care of their 2 children. claimed that he sacrificed his own career as a singer and teacher to devote his time to s career and the children. claimed her career and celebrity status is not licensed, but rather is the product of her talent, and thus not property . However, because s celebrity status and career level increased in value in part because of s efforts, claimed he was entitled to equitable distribution of this property (s career and celebrity status). The Supreme Court found for and appealed. The Appellate Division reversed b. Things of value that are acquired during marriage are marital property even if they do not fit the traditional definitions of property. Moreover, the property may be tangible or intangible ( OBrien there the court also relied on NY Domestic Relations Law 236 in its decision. That law states that when making an equitable distribution of marital property, the court shall consider any direct or indirect contribution a spouse makes to the career or career potential of the other spouse.) Thus, an interest in ones career counts as marital property, and this may be repre sented by contributions of the other spouse, including financial and non-financial contributions, such as caring for the home and family. The fact that s career is not a licensed profession is irrelevant. It is not the license or degree, but rather the increased earning capacity that constitutes marital property. Here, it is clear that made direct and concrete contributions to s career, and gave support through his time with the children. Though came into the marriage with her own genuine talent , and with a position with the Metropolitan Opera, her career had barely begun at that time. was actively involved in s career through his critiquing, teaching, and coaching of . Thus, to the extent the appreciation in s career was due to s efforts, this appreciation constitutes marital property c. An increase in the value of one spouses career, when it is the result of the efforts of the other spouse, constitutes marita l property and is thus subject to equitable distribution d. Notes on Elkus a. In NY to this day, degree is property, and your spouse share the benefit of the property, the degree. b. Critique was already a teacher when they married and therefore, in essence, was able to keep his career and enjoy a comfortable lifestyle as he accompanied his only student all over the world for concerts and other performances. It would seem that has already enjoyed considerable acclaim and other benefits, not only as s husband, but as the teacher of an internationally known opera singer. The courts decision allowed to continue reaping those benefits even though he no longer filled the role of husband or teacher. Other courts have held that an entertainer has good will that is a marital asset, regardless of the contributions of the other spouse VII. Leaseholds: The Law of Landlord and Tenant a. The Leasehold Estates (363) i. Nonfreehold estates 1. When any of the leasehold estates is created, a future interest in the landlord or in a 3rd party necessarily arises a. If the landlord has retained the right to possession at the end of the leasehold, the future interest is a reversion b. If a provision is made for some 3rd party to take possession, ordinarily the future interest will be a remainder ii. The Term of Years an estate that lasts for some fixed period of time or for a period computable by a formula that results in fixing calendar dates for beginning and ending, once the term is created or becomes possessory 1. At common law there was no limit on the number of years permitted, but some US state statutes limit the duration of terms of years 2. Term must be for a fixed period, but can terminate earlier upon happening of some event or condition 3. Notice of termination a. Because a term of years states from the outset when it will terminate, no notice of termination is necessary to bring the estate to an end iii. The Periodic Tenancy a lease for a period of some fixed duration that continues for succeeding periods until either the landlord or tenant gives notice of termination 1. If notice is not given the period is automatically extended for another period 2. Notice of termination a. For any periodic tenancy less than a year, notice of termination must be given equal to the length of the period, but not to exceed 6 months b. The notice must terminate the tenancy on the final day of the period, not in the middle of the tenancy c. In many states, statutes have shortened the length of notice required to terminate periodic tenancies and have permitted a month-to-month tenancy to be terminated at any time following 30-days notice 3. The death of the landlord or tenant has no effect on the duration of a term of years or periodic tenancy, but it does on the tenancy at will 4. Problems on page 364 a. Question 1- The terms of Years; Ts moving out and current market value is $21,000/yr. The best position for the landlord is going after the tenant for the whole year. If he leases it to someone else, he will make less money. Point: the positions each party will take always depend on what the market is doing. The landlord will be better off arguing for a year periodic tenancy by focusing on the fact that they never had a notice of termination. The tenant will argue month-to-month language and argue against the notice problem by saying that the act that gives notice (instead of a letter of notice) was his moving out of one month notice so he only owes October rent but thats it. Landlords counter argument can be (?) b. Question 2 The owner will probably argue that the rules are rules and the notice must be given on the final day of the period and not in the middle. Following the rules straight gives out a message that tenants and landlords must follow the rules and cant cheat. But most likely the court would rule that notice was given and may-by allow December rent to be collected for giving a middle of the month notice but nothing more than that. iv. The Tenancy at Will a tenancy of no fixed period that endures so long as both landlord and tenant desire 1. If the lease provides that it can be terminated by one party, it is necessarily at the will of the other as well if a tenancy at will has been created a. A unilateral power to terminate a lease can be engrafted on a term of years or a periodic tenancy 2. Tenancy at will ends, among other ways, when one of the parties terminates it

30

v.

Garner v. Gerrish Court of Appeals of New York, 1984 (365) a. In 1977, Donovan leased to for $100/month. The lease stated that had the option of terminating the lease at a date of his choice. lived there without incident until 1981. In 1981, Donovan died, and became the executor of his estate. requested that quit the premises. When refused, commenced his action. contends that the lease created a tenancy at will since the length of the lease was indefinite. contends that the lease, by its language, created a determinable life tenancy. The lower court held for (a tenancy at will) and the Court of Appeals reversed b. At common law, the rule was that a lease for so long as the lessee shall please is a lease at will of both the lessee and the lessor. This rule had its origin in the ancient ritual of livery of seisin. Livery of seisin, the transfer of a clod of dirt, was required to create a life estate. Therefore, the common law did not allow the creation of a determinable life tenancy livery of seisin. Hence the old rule: a lease for so long as the lessee shall please is a lease at the will of both lessee and lessor. In modern times, livery of seisin has been abandoned because it is out of step with the realities of modern life. In modern times, the courts look to the terms of the agreement to determine what type of lease has been created. In this case, the terms of the agreement clearly indicate that Donovan and agreed that would have a deter minable life tenancy c. If a lessee has the option of terminating a lease when he pleases, a determinable life tenancy is created d. Notes on Garner a. Lower court thought about it as a contract law. Cant be term of years because there is no term. Cant be periodic tenancy because there is no ending period. So it must be a tenancy at will. b. The appellate court is trying to find a category to stick this in because they are looking at the intent of the parties and look at it like a contract law. The court was resisting the property law approach and wanted to look at the intent and treat it like a contract law. Thats how they said the agreement was a determinable life tenancy. c. Under the old common law, life estate must accompany livery of seisin (ceremony by transferring a clod of dirt). In modern law, that ceremony is abandoned. Livery of seisin was telling the people what you were doing and here the court was basically departing from property approach and wanted to look at the words of the agreement and find its type. So the court made up this rule to meet the intent of the people. d. What is a better approach? How should the modern court look at problems? 1. Problems with treating it like a contract or justifications with sticking with the property format Merilyn Smiths article: property rights often impact third parties while contracts only impact the two parties. So, maybe third parites might need to know whats going on with the agreement to arrange their own matters, and having the rigid property law helps people to be in notice and keep the ones who might be impacted. e. Tension between contract law and property law present in almost all leasehold cases, which are a mixed bad of property law and contract law 1. At early common law, a leasehold was viewed as an interest in property. Since it was a property interest, all of the rules of property law applied i. (the tenant) is giving a contract argument its a contract between Donovan and that says lessee and lessee only has the right to terminate at the time of his choosing ii. is making a property argument theres no definite period, so this must be a tenancy at will, terminable by either party at anytime. There cant be a landlord -tenant relationship that revolves around a life estate. The label of the lease is important, not the words used The Tenancy at Sufferance: Holdovers 1. Tenancy at Sufferance arises when a tenant remains in possession (holds over) after termination of the tenancy a. Common law rules give the landlord confronted with a holdover essentially 2 options to the creation of a new tenancy eviction or consent 2. Crechale & Polles, Inc. v. Smith Supreme Court of Mississippi, 1974 (369) a. In 1964, entered into a lease with . The term of the lease was for 5 years. As the end of the lease approached, learned that the new building he intended to occupy would not be immediately available. notified that he wished to holdover on a month-to-month basis. refused this offer and told that he would have to vacate at the end of the lease. maintains that he told that since he was trying to sell the property, he did not want to get involved in any month -tomonth rental. asserts that informed him that he was trying to sell the building, but that could stay until it was sold or until s new building was ready. a. 2/4/1969 sent a letter confirming their oral agreement to extend the lease on a monthly basis. b. 2/6/1969 wrote denying the existence of any oral agreement concerning extension of the lease and requesting that quit and vacate the premises upon expiration of the term at midnight on 2/6/1969. The letter also advised that he was liable for double rent for any holdover. c. 3/3/1969 paid rent for the period of February to March. The check was accepted and cashed by . d. 4/6/1969 paid rent for March to April, but the check was not accepted by because if was for final payment e. 4/7/1969 sent a telegram to stating that he was tendering the premises for purposes of lessors inventory f. 4/19/1969 approximately 2 months after the expiration of the lease, s attorney wrote stating that since the lessee had held over beyond the normal term, the lessor was treating this as a renewal of the lease for a new term expiring 2/6/1974 g. 4/24/1969 again tendered the check for the final months occupancy and it was rejected by h. 4/29/1969 s attorney wrote again stating the lessors intention to consider the lessees holdover as a renewal of the terms of the lease i. There was no further communication between the parties until a letter dated 5/15/1970 from to requesting that pay the past-due rent or vacate the premises j. 5/27/1970 s attorney tendered the keys to the premises to k. filed suit to recover back rent and damages beyond ordinary wear and tear to the leasehold premises. The Chancery Court awarded the $1,750 in back rent payment and $760 for damages to the leasehold premises, as well as costs incurred in the proceeding. appealed and the SC of Mississippi affirmed b. The landlord may elect one of 2 options when a tenant holds over. He may either (1) evict him (2) treat him as a trespasser or (3) hold him over for a new term. The letter on 2/6/1969 was an effective election on the part of to terminate the lease and treat the as a trespasser. After having elected not to accept the s as tenants, could not at a later date, after failing to pursue his remedy to evict the tenants, change t he election so as to hold the s as tenants for a

3.

31

c. d. e.

f.

g.

new term. Although expressly refused to extend the lease on a month-to-month basis, nevertheless accepted and cashed the rent check for the month of February. The normal effect of such action by the landlord is tantamount to extension of the lease for the period of time for which the check was accepted unless, of course, the landlord had elected to treat the tenant as a holdover tenant. It is the rule that, absent evidence to show a contrary intent on the part of the landlord, a landlord who accepts rent from his holding-over tenant will be held to have consented to a renewal or extension of the leasing. After cashing the check, the landlord refused to accept further tender and rejected the final payment check. On 4/19, informed that he would be considered a holdover tenant so as to extend the lease for another term. Once a landlord elects to treat a tenant as a trespasser and refuses to extend the lease on a month-to-month basis, but fails to pursue his remedy of ejecting the tenant, and accepts monthly checks for tent due, he in effect agrees to an extension of the lease on a month-to-month basis. The issue of damages was a question of fact for the Chancellor, and from an examination of the record, the court cannot say the Chancellor was manifestly wrong (and the decision is therefore affirmed) Once a landlord elects either to treat a holdover as a trespasser or to hold him to a new term, the landlord may not change his mind (landlord) won the lower court but still appealed because wanted more money. He wanted to get damages for the whole year not for the month and a half. Was it a mistake of the s lawyer advising his client to write a letter on 2/6/1969 to write that letter? If that letter didnt go out, then could just hold the tenant for a new year term. It probably wasnt a mistake because the wanted the tenant out for sure so writing that letter asking him to leave was a good thing. over time realized that he probably cant sell the property so now he doesnt want the tenant to leave and wants to keep the tenant for another year because that will be better for . It is clear the landlords position is evolving. But, why are we making it so easy for the tenant? T his is not a criminal law but is business. Once the letter was sent to move out by certain day, the tenant had to make up his mind and plan his business based on the representation of the landlord. thought that he would be better off with month -to-month agreement now so that was like a counteroffer. And cashing out the check is like acceptance of month-to-month offer. Point: if you have a client like , 1. Treat him like a trespasser and get an eviction order from the court. 2. Hold him over for another term. --- The rule is that you dont need consent of the tenant but only need notice. There are three ways you can deal with because the third way is, although not law, a negotiation result going to month-to-month by negotiating and getting that on writing

b.

c.

The Lease (373) i. An arrangement explicitly declaring itself , in writing, to be a lease, might nevertheless be held by the courts to amount to something else, such as a license or a life estate 1. Why does it matter whether an arrangement amounts to a lease, as opposed to something else? a. Primarily because leases give rise to the landlord-tenant relationship, which carries with it certain rights and duties and liabilities and remedies that do not attach to other relationships 2. Conveyance versus Contract a. A lease is both it transfers a possessory interest in land, so it is a conveyance that creates property rights a. But it is also the case that leases usually contain a number of promises such as a promise by the tenant to pay rent or a promise by the landlord to provide utilities and in that sense, a lease is also a contract b. Courts today commonly rely, explicitly, on contract principles to reshape the law of leases b. Statute of Frauds a. Commonly, the US statutes provide that leases for more than one year must be in writing (to comply with the statute of frauds) b. All but a few jurisdictions permit oral leases for a term less than a year; those that do not usually hold that entry under an oral lease plus payment of rent creates a periodic tenancy that is not subject to the statute c. Form Leases a. Typically used by landlords standardized documents offered to all tenants on a take-it-or-leave-it basis, with no negotiation over terms 1. Landlord is trying to avoid the costs of negotiating and drafting a separate agreement with each purchaser 2. Sinister explanation the seller/landlord refuses to dicker separately with each purchaser because the buyer has no choice but to accept his terms 3. If one seller/landlord offers unattractive terms, a competing seller will offer more attractive terms and the process will continue until the terms are optional i. Whats important is that competition forces sellers/landlords to incorporate in their standard contracts terms that protect the purchasers ii. Under a monopoly, the buyer has no good alternatives to dealing with the seller, who is in a position, to compel the buyer to agree to terms that in a competitive market would be bettered by another seller iii. Remedy for a monopoly the courts can respond by policing lease terms on grounds of unequal bargaining power but this is a case-by-case approach that might be insufficient calling instead for statutory reform legislation requiring full disclosure of landlords and tenants duties, rights and remedies Selection of Tenants (376) i. Mrs. Murphy case on page 380. 1. Section 3603(b)(2)- fits her into exemption to everything but 3604 subsection c; so Mrs. Murphy is off the hook except for (c)on discrimination why? Certain amount of discrimination is ok because people cant force others to live with certain people. But the same reasoning doesnt apply to subsection a & b. This might be just a rough compromise that unless youre a big corporation or housing developer, we would allow some discrimination on your own quarters. 2. Should Mrs. Murphy be liable under Section 3604(c)? to publishthat indicate any preference based on race; Mrs. Murphy can argue that it was just informational; counter- why not other relevant information that are missing, how would a reasonable person look at the add and read it? Does the renter need to show intent or only effect? 3. Why only find discrimination for publishing her discriminatory feeling? We allow her to discriminate but just cant advertise it. If we care about individual rights and respect the personal discriminatory right for the premise, why prohibit the advertisement? In this

32

d.

time of the legislation, there was something exceptionally damaging on its own way by advertising discriminatory position. We dont want the market force to validate discrimination. ii. Landlords, once free to discriminate as they wished in selecting tenants are today constrained in a number of respects 1. The most significant constraints are imposed by the Federal Housing Act (portion on pp. 376-378) a. Provides for the award of reasonable attorneys fees to successful aggrieved parties from losing landlords a. Purpose behind awarding attorneys fees to encourage victims of discrimination to seek judicial relief Congress also believed that vindication of fair housing claims would be discour aged by awards to s b. A discriminatory motive need not be proved in order to make a prima facie case under the FHA proof of discriminatory intent is sufficient (only effect matters not intent) a. Claimants need merely to demonstrate that an action or practice carries a discriminatory or segregative impact in order to shift the burden to the b. Alternatively, when a single claims a housing denial without regard to a policy or pattern, the establishes a prima facie case by proof of disparate treatment, typically a denial to an eligible minority applicant followed by a subsequent transfer to another party or the continued availability of the dwelling in the market c. After the establishes either disparate impact or disparate treatment, the must then justify the action as one taken in pursuit of a bona fide, compelling governmental purpose, with no less discriminatory alternative available to achieve the goal, or in the case of private s, one taken pursuant to a rational and necessary business purpose c. Discrimination on grounds other than race as originally enacted, the FHA did not prohibit discrimination on the grounds of sex, handicap, or familial status a. All of the above have since been incorporated 1. Handicap defined in the FHA as a physical or mental impairment which substantially limits one or more of the handicapped persons major life activities, a record of having such an impairment, or being regarded as having such an impairment, but such term does not include current, illegal use of or addiction to a controlled substance i. Proposed amendments that would have excluded alcoholism and infectious, contagious, and communicable diseases from the definition failed to pass 2. Soules v. US Dept of Housing a. This case portrays how the FHA is adjudicated. b. Need to make a Prima Facie case in order to avoid the motion to dismiss a. Intent is difficult to prove, thus letting the come to the court and ask for adjudication without proving intent or make a prima facie case w/o proving intent + is in the best position to prove or give evidence for the intent b. Courts have created shifting of burden for FHA, and has to prove 4 elements 1. Plaintiff must show that they are a member of a statutorily protected class 2. applied for and was qualified to rent or purchase housing 3. was rejected 4. Housing remained available. c. As soon as can establishes a prima facie case, the burden of proof shifts to and then has the burden to show that he had a compatible/compelling reason other than what claims d. Here, the realtor () claimed that the attitude of was bad and that was the reason not to show her the apartment and not to discriminate her against her familiar situation. e. Now, the burden shifts back to . 1. shows the pattern of discrimination demonstrated by the testers 2. also says that the realtor later met with the but still was uncooperative and didnt show other apartments c. The court did not reverse and still won. Why? a. The job of the appellate court is to review the trial courts decision b. The AC gave a great deference to the trial court to find who is more credible d. Does the landlord have a legal right to want what they want? As a owner, yes. e. Does the downstairs tenant have a legal right to say I dont want kids upstairs.? No. f. Suppose really didnt mean to discriminate. How should you advise her not to be sued while making the best decisions? a. can say The housing application requires me to know how many children you have or I need that information to fill out your application b. If the realtor is worried about potential fighting with downstairs people, thats a good concern. But still show the apartment. c. How do you produce evidence for non-discrimination even before anything happens? Write down why you picked the person, i.e. the best credit history, a good letter of reference or ad letter of reference, etc. d. Is there a pattern? If there is a pattern, whether the broker realizes or not, of only giving the apartments to certain group of people or not giving them to some people, then they have a problem. You have to take care of that- dont do that all the time. e. Is it a sufficient to use bad attitude or I didnt think they would get along in the same building as a defense? g. 3504(c): court made it clear that the standard is what an ordinary listener infer of the statement. I n this case, what would an ordinary listener would infer when he listened how many children do you have? How old is your child? a. However, the ALJ considered Why she asked the questions. The standard doesnt care about why. b. So why didnt the AC find that ALJ errored? 1. The court reasoned that they only have 2 peoples oral statement and there is no way to definitely know and figure out what the tenor of the conversation was. If there was a legitimate reason to ask those questions, that might suggest that the question was asked in a very innocuous/non-offensive way 2. So, looking into why she asked questions, although such is beyond the scope of the standard, was justified. 3. But, the housing regulations only required different gender not in one room. The age question was not required but the realtor still asked Subleases and Assignments (388) i. Ernst v. Conditt Court of Appeals of Tennessee, 1964 (388)

33

1.

2.

3. 4.

5.

(owner) leased land to Rogers for a term of 1 year, 7 days. After Rogers operated a race track for a short time on the premises, he entered into negotiations with (sublease) for the sale of the business. The original lease was modified when Rogers and went to . The modified lease contained the following provisions: (1) The term of the lease was extended to 2 years; (2) Rogers was given the right to sublet the premises to on the condition that Rogers remain personally liable for the performance of the t erms or the lease. Subsequently, Rogers leased the premises to . In the contract between Rogers and , the term sublet was used. After took possession, he operated the race track for a short time. Soon, however, stopped paying rent, but he remained in possession. sued for the rent owed. contends that the agreement between Rogers and is an assignment of the lease, which would render liable to . contends that the agreement had with Rogers was a sublease and therefore Rogers would be liable to . filed suit seeking a recovery of $2,404.58 (balance due on the basic rent) and the sum of $4,200 (basic rent for the 2 nd year of the lease plus the sum necessary for the removal of the improvements constructed on the property. Chancellor found that the instrument was an assignment and a decree was entered for . appealed and the Court of Appeals of TN affirmed An assignment allows the land owner to recover from the assignee. A sublease does not allow the land owner to recover from sublessees. At common law, the following rule was used to determine whether a lessee assigned or sublet a premises: an assignment conveyed the entire interest in a lease; it left nothing to the original lessee. A sublease granted an interest that was less than that owned by the original lessee; the original lessee retained a reversionary interest. The modern method of determining whether there has been an assignment or sublease is to ascertain the intention of the parties. Under the common law rule, is liable to because he took Rogers entire interest in the land. The fact that the modified contract between Rogers and held Rogers to be personally liable to does not matter because the liability of Rogers to is not determined by an agree ment between Rogers and (the agreement was part of the agreement between and Rogers). Also, the fact that the modified agreement contained the word sublet does not matter. s consent to sublet has been held to be only a consent to one other than th e original lessee to go into possession; it authorizes both a subletting and an assignment. is liable to under the modern rule. By the terms of the agreement between Rogers and , Rogers retained no reversionary interest; he conveyed all that he had and Ro gers therefore assigned his lease. In addition, went into possession and personally paid . remained in possession for the entire term of the lease (Jaber v. Miller if the instrument purports to transfer the lessees estate ofr the entire remainder of his term, it is an assignment regardless of its form or the parties intention). By virtue of the sale of his business, became the owner of t he improvements with the right to their removal at the expiration of the lease In determining whether an assignment or a sub-leasing has occurred, the court looks to the intentions of the parties Notes on Ernst a. Privity of Estate and Privity of Contract a. Assignment privity of contract remains, but privity of estate changes 1. Privity of Contract between and Rogers and between Rogers and i. Still privity of contract between and Rogers because Rogers cant unilaterally get out of the contract to get out of his obligations in contract, both and Rogers would have to agree to end the contract if not, Rogers is still on the hook ii. Because there is privity of contract between and Rogers, could also sue Rogers 2. Privity of estate between and b. Sublease 1. Between and no privity of estate and no privity of contract (means theres no legal relationship between and ) i. Privity is between and Rogers contract and estate and would have to go after Rogers for the $ b. Property Law versus Contract Law - Older rule: if the primary tenant has transferred the entirety of their right to possession, it is an assignment c. Property argument is strongest if theres an assignment Rogers conveyed his entire interest to and maintained no reversionary interest d. Contract argument is strongest if theres a sublease a textual argument looking at the text of the lease, Rogers remained liable and therefore didnt transfer his entire term to , retaining the right to re -enter e. Court says its looking at the intent of the parties, which is a contract principle, and when it does so, Rogers transferred everything he had but the court looks to be sticking with the property rule Ernst v. Conditt (Class Discussion) a. Who are the Ernsts suing and why?- They are the LLs and theyre suing for back rent at the end of the lease agreement: They want 15% of revenues. b. Whats been happening every month? Tenant has not been paying up. c. Suppose court had found that agreement between Rogers and Conditt was a sublease, who would have won? Conditt would have won. a. Why? Because T is liable for the rent, not really the sub-tenant because you cannot unilaterally assign. Imagine what this would do to student loans, mortgages. b. There is only one way for Rogers to stop being liable. What is it? With Ernsts permission. You need a novation. You can freely assign the benefit of a contract but not the obligation. c. Why is this? Because of privity of contract - if there was a sublease, there would never have been privity of contract between the parties, no legal relationship to give reason for Ernst to sue Conditt directly. d. Why isnt there privity of estate between Ernst & Conditt? If this is a sublease the privity of estate between Rogers and Earns (and privity of estate) and between Rogers to Conditt. Problem for Earnst has no legal relationship with Conditt. Conditt argues it was a sublet. e. What does the court say? That its an assignment. f. How does this help Ernst? An assignment transfers, There is not privity of estate is between Rogers take his right and transfer to Conditt (privity of estate) and that gives Ernst privity of estate between he and Conditt. g. Why is Rogers still liable to Earnst? Privity of contract. The court got privity of estate and privity of contract wrong.

34

ii. iii.

Lets explore Conditts argument: As between E & R = Privity of estate & contract As between R & C = Privity of estate & contract As between E & C = a. Even if its a sublease does Earnst have a right to sue anyone? Yes, he can always sue Rogers; and he in turn can sue Conditt. b. If Conditt can be sued either way, why does Conditt litigate? c. Whats probably going on is that no one knows where Rogers is; Rogers may be judgment-proof. Theoretically, Rogers is liable either way but Why would anyone in Conditts position get into an assignment? a. Sometimes people dont know the difference between a sublease and an assignment. b. People in Conditts position want to continue to have privity with the landlord; to hold him to his obligations (e.g. repairs). c. What is the basis for courts holding (that it is an assignment)? One approach, how much was the sub-tenant assigned? If he transfers his whole interest, its an assignment; if less than subtenant. The rule (in property) here is if prime tenant transfers less than whole interst a sublease has been created and they are still in privity of estate. Does court give another reason? Whats Conditts best/obvious argument? : The writing says sublease. Why isnt that upheld here? Were looking at the intent of the parties (grounded in contract law). Here the court is saying, we have a lease under contract law. d. What is the presence of the word sublease still interpreted as assignment? Rogers transferred his entire interest to Conditt and therefore, that established intent to transfer an assignment. e. What is Conditts argument under property law that this is a sublease? Rogers had the implied right to enter, he did not give his entire possessory interest/right to possession because of the implied right to enter if Conditt defaulted. Why does this argument fail? The right to remain liable to Ernst was superfluous because unless Ernst releases him from the contract, he IS liable. A second point, even if Conditt had prevail under the implied right to re-enter argument, that still would not have been enough in a majority of jurisdiction. In a term of year lease, regardless of assignee/sub-tenant is on the hook for the entire lease term. 7. Test your knowledge: See problem on 394 problem C. a. L leases to T for a term of 3 years $1K/month; 6 months later T assigns to T1; 3 months later to T2; then 3 months after that T2 assigns to T3, who defaults. Here we have a series of assignments. What is their responsibility to each other and L? a. T3 is liable to L. In order to establish a legal right, there must be privity of estate because its an assignment (as opposed to contract). b. When L assigns to T, privity of contract and estate, when T assigns to T1 there is a contract between them but privity of estate is between L and T1 and when T1 assigns to T2 there is privity of estate between L and T2; the privity of contract relationships do not change. c. Is anyone else liable to L for value of the possession? T3 d. If L wants to sue T can this happen? Yes but if T wants to sue T3, T3 cannot sue directly, it must sue with whoever T3 has privity of contract with (e.g. T can sue T1 who would then sue T2 and T2 would then sue T3). e. What is the purpose of the language assumes all conditions in the lease; can you make and argument that T1 is still liable to L? If you were the L you would say that under the theory of third party beneficiary, L will argue that between T & T1 it was made to benefit L as a third party. That when T1 says I assume all conditions...that what he is saying is that L is a third party beneficiary to the contract between T and T1. b. Is there language that Ernst could have used to make a third party argument? a. For value received consideration of the promise...I hereby sublet to Conditt... he made a promise to faithfully perform the conditions of the prime lease. You need clear manifestation of 3rd party rights (e.g. mention of the prime lease). When there is a chain of subleases there is no privity of contact with the subsequent subtenants and L. Notes (393) 1. Ernst indicates the 2 ways in which courts have gone about distinguishing between a sublease and an assignment a. Formalistic Approach an assignment arises when the lessee transfers his entire interest under the lease when he transfers the right to possession for the duration of the term a. If the lessee transfers anything less than his entire interest, a sublease results 1. In the case of a sublease, the lessee retains a reversion 2. A second approach considers the intent of the parties and the actual words used are not conclusive, though they may be persuasive Kendall v. Ernest Pestana Supreme Court of California, 1985 (395) 1. The city of San Jose leased a hangar to Perlitch. Perlitch sublet the premises to Bixler for a term of 25 years. The lease provided that written consent of the lessor was required before the lessee could assign his interest. Bixler was to run an airplane maintenance business. Subsequently, Perlitch assigned his interest to . Later, Bixler attempted to sell his interest to . was more financially sound than Bixler. As per the terms of the original lease between Bixler and Perlitch, Bixler requested consent from for the sublease. demanded an increase in rent in exchange for consent . contends that he may arbitrarily refuse consent. contends that this provision is against public policy since it is an unreasonable restraint on alienation. brought suit for declaratory and injunctive relief and damages in the form of a declaration that the refusal of Ernest Pestana, Inc. to consent to the as signment of the lease is unreasonable and an unlawful restriction on the freedom of alienation The trial court s ustained a demurrer to the complaint without leave to amend (so won) and the SC of CA reversed 2. The law favors free alienability of property, including a leasehold. However, alienability may be restricted by contract so long as to protect the lessor. Such contracts are construed against the lessor, especially when the act of assigning terminates the lease. The

6.

iv.

35

3. 4.

majority rule allows the lessor to arbitrarily withhold consent. Even in the majority states, however, the lessor may waive his right or be estopped from asserting it. A growing trend asserts the following rule: A lessor may withhold consent only when he has a commercially reasonable objection to the assignment. The court adopts this rule for the following reasons: (1) Public policy favors alienability; (2) the relationship between lessor and lessee has become more and more impersonal, as a result, the lessor is just as likely to find a high quality tenant in the assignee as the lessee; (3) the lessors interests are protected by the fact that the lessee remains contractually liable to the lessor; (4) a lease is increasingly viewed as a contract. As a result, since the lessor has discretion in withholding consent, the lessor can withhold consent only in good faith. The lessor may withhold consent in the following circumstances, for example: (1) the property is ill-suited for the porposed use; (2) the proposed use is illegal; (3) the proposed use requires altering the premises. The lessor cannot withhold consent solely on the basis of personal taste. Nor may he demand higher rent in exchange for consent. This is because the original lease already exhibits an agreement between lessor and lessee, and lessor may not attempt to get more than he bargained for. There are 4 reasons advanced for allowing arbitrary withholding of consent: (1) lessor has picked his tenant and he should not have to look elsewhere for the rent. This is unpersuasive because lessor may still refuse consent if it is reasonable. If an assignee is a poor financial risk, lessor may reasonably withhold consent. Moreover, the original lessee is still liable to the lessor; (2) the lessee could have bargained for a contract clause that provided that consent could be withheld only if reasonable; the court should not place this clause in the contract. This is rejected because the fact that the parties agreed to a consent requirement must mean that they contemplated that the lessee give some reason for withholding consent; (3) the court must follow the rule because of stare decisis. Rejected, because the court has never ruled on this issue; (4) the lessor has a right to realize the increased value of property by demanding higher rent. Rejected because the lessee took the risk that property prices would fall. He is also entitled to any gain should prices rise. The lessor, as well as the lessee, must live with the risk incurred as a result of the contract made A lessor may not unreasonably and arbitrarily withhold his or her consent to an assignment Notes on Kendall a. Despite the majority rule, the court adopts the minority rule a. Majority Rule where a lease contains an approval clause, the lessor may arbitrarily refuse to approve a proposed assignee no matter how suitable the assignee appears to be and no matter how unreasonable the lessors objection b. Minority Rule (and trend) where a lease provides for an assignment only with the prior consent of the lessor (the provision bargained for Lessors right of approval), such consent may be withheld ONLY where the lessor has a commercially reasonable objection to the assignment, even in the absence of a provision in the lease stating that consent to assignment will not be unreasonably withheld 1. Court gave 2 justifications. This rule follows from both property law and contract law: i. Property follows that such provision would be an unfair restriction on alienability, thus interpret the provision narrowly to promote alienability ii. Contract theres an obligation of good faith and fair dealing, and lessor cannot enforce in bad faith 2. Dropped a footnote 23: parties bargaining came up with a clause and that provision would be valid if freely negotiated. This undermines property law approach. In this case, feudalistic approach of property law is undermined and court adopted contractual mode of interpretation of the provision. b. Court here is merely interpreting the contract or rewriting it? a. What is good faith mean? Providing a good, reasonable reason? b. Here, the good reason from s perspective is that he wants to make more money by directly dealing with the lease instead of dealing with the sublets. He has a financial incentive to say no because hes a businessman. c. Tenant agreed to pay for certain things in the contract. He assumed that the landlord would not act in any way to undermine tenants right. Basically, tenant has a year lease with a certain amount that is a guess and out of the control of the market value. Court is saying that anything that undermines that deal is in bad-faith. And here, the landlord wants more than what they had contracted for. d. Whos entitled to the mark-up based on the increase of market value? Tenant who captures the value by subleasing? Or landlord? If market goes badly, the tenant is not off the hook and still has to pay more than the market value. So how fair is it to say landlord benefits when the market is good and bad. That is not reasonable and in good-faith. e. Here, they use commercially reasonable standard. Its not about whether the landlord gets the most benefit. Its about whether it will be detrimental to the landlord by the move of the lease. c. Lease is a hybrid of property law and contract law. Under the property law, it protects the free alienation of the property. So, as was in this case, the landlords restriction of alienation by not giving the consent is bein g frowned upon by the court. The court is not absolutely limiting the ability to restrict the tenants alienation. But, its just saying that it must be reasonable. d. Its a matter of which law, contract law or property law, the court wants to view the case from. IN this case, if everything was disclosed and consented to s consent for subleasing, then there was no unbalance of power and that was the contract they created and should abide by it. e. Should the rule in Kendall apply in residential leases? If landlord reserves the right to reject, then the reason behind the rejection must be commercially reasonable? What is reasonable would be much broader in residential context and more litigation can be happening there. f. P. 402 problem 1b: its about our building. Was Yeshivas reason commercially reasonable one? Have two opinions. a. Wouldn't be forced to lease to Planned Parenthood as the main tenant, why can't they restrict them as a sublease? b. Tenant: In case Cite Kendall: landlord can only deny sub-tenant only when the denial has commercially valuable reason ("Denying consent solely on the basis of personal taste, convenience or sensibility is not commercially reasonable..."); Value argument is difficult to decide which should be respected by the court; i ts like Kendall because tenant bargained with the lessor as a market participant and landlords just using his value argument to cover-up the real intention to make more money just like Kendall; In Kendall, it was about shopping center usage. The landlord should think about how one sub-lease would affect the entire shopping centers economy. Think about Target coming to a high-end shopping center. That is a commercially reasonable argument. But here, there is no commercially reasonable argument similar. Planned parenthoods coming in would not affect other businesses in the building.

36

c.

d. e.

The Tenant Who Defaults i. Ways to default 1. Failing to pay rent or observe some other lease obligation 2. Lessee holding over after the termination of the lease 3. Tenant abandons the premises prior to the end of the tenancy, perhaps owing some back rent ii. The Tenant In Possession 1. Berg v. Wiley Supreme Court of Minnesota, 1978 (403) a. Wiley () leased land to Phillip. Berg for 5 years. The lease provided that no changes in the building structure should be made without s consent. reserved the right to retake possession should P. Berg fail to meet the terms and conditions of the lease. Subsequently, P. Berg assigned the lease to Kathleen Berg (). Without securing s permission, began to remodel the premises to make them suitable for a restaurant. A dispute arose between and as a result of s continued remodeling of the restaurant without permission and operation of the restaurant in violation of the health code. demanded that the health code violations be remedied and that complete the remodeling within 2 weeks. At the close of the 2 weeks, closed the restaurant, dismissed her employees and placed a sign on the premises that read, Closed for Remodeling. Also after 2 weeks, attempted to change the locks on the doors, but ceased his attempt when arrived. 3 days later, without s knowledge, changed the locks. contends that she was wrong fully evicted. raises the defense of abandonment. The jury found no liability on s claim for intentional infliction of emotional distress and no liability on s counterclaim for damages to the premises, but awarded $31,000 for the lost profits and $3,540 for loss of chattels resulting from the wrongful lockout. The jury also found that neither abandoned nor surrendered the premises. On appeal, sought an outright reversal of the damages award for wrongful eviction, but the SC of MN affirmed b. Though the testimony is in conflict, there is ample evidence to support the jurys finding that did not abandon the premises. At common law, a landlord could use self-help if: (1) the landlord is legally entitled to possession, such as when tenant is a hold-over; and (2) the landlords means of entry are peaceable. On the first requirement, contends that he was entitled to possession since breached the lease. However, it is for the courts, not to decide if a landlord is entit led to possession. Turning to whether entry was peaceable, contends that only actual or threatened use of violence should give rise to liability. The court is not convinced. There has been a public policy in this state, expressed by both courts and the legislature, aimed at discouraging self-help. To discourage self-help, the legislature has created summary proceedings to help the landlord regain possession quickly. To further discourage self-help, the legislature has provided treble damages for forced entry. In consideration of this policy, s entry was not peaceable. Given the historical relations between and , the only reason violence did not erupt was because was not present when the locks were changed . Indeed, any use of self-help has the potential for violence. Therefore, unless the tenant has abandoned or surrendered the premises, a landlord may not use self-help. The landlord must resort to the judicial process to regain possession c. A landlord may not use self-help to regain possession of his land d. Notes on Berg a. There were 2 requirements for self-help. The court reasoned that because the landlord breached the 2nd requirement of peaceful entry, no need to consider whether the landlord is legally entitled to possess. b. has to show that not only was the lease breached, but that his (the landlords) entry was peaceable 1. Because the court finds that this wasnt a peaceable entry, the court doesnt consider whether the lease was breached 2. Peacefulness is at issue because the court prefers more clear rule than self-help which can lead to potential violence. 3. Is there a way for the landlord to exercise self-help and court would have found it peaceful? If tenant surrenders the key, no problem. If landlord waits for three months and nobody shows up, then it would have been considered as abandonment, then no problem. c. Court changes the rule NO SELF-HELP. Period. Punishing the landlord. 1. It does appear that the court is applying the new rule retroactively. 2. Says should have taken advantage of summary proceedings i. Summary Proceedings statutes enacted to allow the landlord to file an action and on very quick notice, get the tenant out d. What should have the landlords attorney advised? What would have been the best advice? Go to the court and claim that there is a breach and asks for eviction. Then Landlord wouldnt have to pay $31,000 for lost profit. e. Policy tenant protections (no self-help and landlord always have to go to court to do summary proceedings) protect bad tenants and pass the costs to all tenants and as a result, all tenants are hurt rather than being helped 1. Self-help is better for the tenant tenants wrongfully possessed have a legal remedy wrongful eviction. It is possible to sue the landlord and be made whole because of wrongful eviction. This could be cheaper because it lets the landlord use self-help to get rid of those who should be evicted property 2. No self-help rule can encourage stability and commercialbility because people would know what they need to do before something happens. The bright line rule like this one also minimizes cost for everybody including the court. 2. Notes a. Under the common law, a landlord entitled to possession could resort to self-help without fear of civil liability so long as he used no more force than reasonably necessary b. The modern view on self-help appears to be that in Berg (no self-help) a. It should be noted, however, that while a flat prohibition on self-help is the trend, it is probably not yet the majority rule

Landlords (Yeshiva) argument: It's just not for personal taste, but it would disrupt the collegiate environment (protesters); Could also hurt the endowment of the school, etc. and hurt financially and thats a commercially reasonable defense; In Kendall, the court found the landlords reasoning problematic because he wanted to get more than what he bargained for and make more money by finding a new person for the lease; When tenant contracted with a religious organization and later wanted to sub-lease, it should have anticipated landlords objection to the sub-lease. This should not be a surprise; they should be able to reserve their image, especially knowing the fact that this building has religious headquarters, by using commercially reasonable justifications. Only using the value-based argument would not be sufficient but using some commercially reasonable justifications might give you a win.

37

iii.

Where self-help is allowed, of course, there still arises the difficult issue of just what constitutes reasonably or permissive force 1. Here, the courts have tended to be rather strict, so that self-help may be a theoretical but not a practical alternative c. Bergs reasoning involved a commercial lease, but the courts reasoning would appear to apply to all leases, residential and commercial alike 3. Summary Proceedings a. At one time, self-help was a very important remedy for the landlord who sought to recover possession of leased premises, because the only alternative was a cumbersome, time-consuming, common law procedure (ejectment) a. The time and expense associated with ejectment ill served the landlords needs; self-help, on the other hand, was problematic from the viewpoint of landlords, tenants and society generally b. Today every state provides some form of summary proceedings a. Intended to be a quick and efficient means by which to recover possession (and in some jurisdictions, rent) after termination of a tenancy b. To promote quickness, the typical statute requires only a few days notice to the tenant prior to bring an eviction action, and the range of issues subject to litigation is kept narrow c. A number of jurisdictions are beginning to provide that failure by a landlord to maintain leased premises in a habitable condition justifies certain self-help measures by a tenant, including withholding rent 1. Seems logical to allow the tenant of a premises to raise the condition of the premises as a defense to a summary eviction action brought for nonpayment of rent d. Courts have relied on summary proceedings as one reason to abrogate the common law remedy of self-help 1. Problem summary eviction procedures can be time-consuming and expensive, even if uncontested The Tenant Who Has Abandoned Possession 1. Sommer v. Kridel Supreme Court of New Jersey, 1977 (410) a. 2 cases combined Sommer and Riverview Realty Co. Sommer () leased an apartment to Kridel () for a term of 2 years. Before went into possession, however, he informed by letter that he could no longer afford the lease, but was forfeiting the rent money already paid to . did not respond to the letter. Subsequently, a 3rd party inquired about the apt. that had rented and though the 3rd party was ready, willing and able to rent that apt., she was refused because it had already been rented to . Over a year later, the apt. was finally rented for terms and conditions similar to those negotiated with . sued for damages and rent due between the date was to take possession and the date when the apt. was rented to the 3rd party. contends that there is no liability because both failed to mitigate and accepted s surrender of the premises. A trial judge found for and the appellate division reversed. The SC of NJ reversed the appellate decision and affirms the decision of the trial court a. Riverview Realty Co. entered a written lease with for a 2-year term (2/1973-1/31/1975). took possession of the apt. and stayed until 2/1974 when vacated the premises having paid rent through 1/31/1974 (one year less than the initial lease terms). The trial court granted the landlords motion for summary judgment against the , fixing the damages at $4,050+interest. The appellate division affirmed. The SC of NJ reverses and remands to the trial court for proceedings in accordance with this opinion b. The majority rule is that the landlord has no duty to mitigate. This is based on the property view of a lease. This view understands a lease to be a property interest which forecloses any control of the property by the landlord. (Muller v. Beck the tenant has an estate with which the landlord cannot interfere). It was understood that when the tenant vacated, he could retake the premises when he wished. Since the landlord had no control, it would be silly to require him to mitigate. Modern social forces, however, have exerted pressure on the law of estates. The law of estates has responded by allowing specific clauses in contracts to deal with these forces. Since we now view a lease in large measure as a contract, we apply a contract principle: the landlord must mitigate damages. This rule derives from considerations of fairness. It would be undair to allow a landlord to sit idly by and watch damages mount when he can prevent those damages. In Sommer, the landlord let over a year pass and allowed $4,658.50 in damages accrue before he made efforts to re-let. The landlord cannot argue that he will lose the opportunity to rent to another if hes required to first rent the apartment that has been abandoned. This is because each apartment has unique characteristics, and there is no reason to believe that a tenant who wished to rent the apartment that has been abandoned also would have been willing to rent another vacant apt. In fact, in Sommer, there was a specific request for the abandoned apt. So the rule is: if the landlord has many vacant apartments, he must treat the abandoned apt. as one of his vacant stock and make reasonable efforts to re-let it. Since the landlord is in a better position to demonstrate that his efforts were reasonable, he shall have the burden of showing reasonable efforts to relet c. A landlord is under a duty to mitigate damages by making reasonable efforts to re-let an apartment wrongfully vacated by the tenant d. Notes on Sommer a. Should the Sommer rule apply equally to commercial leases? Commercial lease mitigation is a much bigger deal. Commercial leasing is highly particular and more difficult and has to consider more complicated elements. So maybe dont want to force people to mitigate in commercial leases. Business tenants and residential tenants are different. Commercial tenants are negotiated leases and both sides have lawyers and already know the risks and consider them before they sign a lease. There is an equal bargaining power in commercial lease setting. And Sommer rule should apply only in residential setting. b. This is not like a lost-volume sale because 1. The rent is too high, 2. Each property is unique, 3. There is finite number of property. c. This case again considers the difference between the precedents of property law and that of contract law 1. The court opts for the contract law arguments, finding it necessary for a landlord to mitigate damages the minority rule the landlord will carry the burden of showing that he has used reasonable diligence in attempting to re-let the premises i. In judging each case on its own facts and in assessing whether the landlord has satisfied the burden, the court should consider, among other factors, such as: (1) whether the landlord, either personally or through an agency, offered or showed the apt. to any prospective tenants, or advertised it in a local newspaper actually demonstrate the effort, i.e., placed an ad, called a broker, showed it to others. b.

38

ii. iii. 2. Notes a.

The tenant may attempt to rebut such evidence by showing that he proffered suitable tenants who were rejected how would tenant show landlord failed? If the burden is on landlord and landlord has to show attempt, Landlord is more likely to mitigate and document the attempt. Landlord does not have to succeed but at least show an attempt to mitigate.

f.

Various justifications are given for the rule, forsaken in Sommer, that a landlord is under no obligation to mitigate damages in the event of abandonment by a tenant a. The tenant cannot by his own wrongdoing impose a duty on the landlord; the tenant has purchased an interest in real estate and is stuck with it; the landlord should not be forced into a personal relationship with a new tenant he does not wish to accept; the landlord should not be required to seek out new tenants continuously; abandonment of property is an invitation to vandalism and the law should not encourage such conduct by putting a duty of mitigation of damages on the landlord b. Theres a line of cases that suggests that efforts on the part of a landlord to mitigate damaged by reletting abandoned premises might be held to constitute an unwilling acceptance of the surrender offered by the defaulting tenant b. Surrender terminates a lease, provided, of course, that the landlord accepts the tenants offer. If he does if the surrender is effected this extinguishes the lessees liability for future rent, but not for accrued rent or for past breaches of other covenants c. The minority rule stated in Sommer has become the majority d. Justifications for the duty to mitigate a. Modern notions of fairness and equity; efficiency (no duty to mitigate encourages both economic and physical waste a mitigation requirement thus returns the property to productive use rather than allowing it to remain idle); helps prevent property damage b. NY sticks to the old common law rule no duty to mitigate 3. Landlords Remedies and Security Devices a. Rent and Damages a. The landlords right to sue for back rent and for damages occasioned by the tenants breach of lease obligations is straightforward 1. If the tenant is in possession, the landlord may also terminate the lease and recover possession b. Anticipatory Breach 1. In addition to collecting past rent and damages, and in addition to terminating the lease and evicting the tenant the landlord can recover damages equal to the difference between the rent reserved in the lease for the unexpired term and the reasonable rental value of the premises for that period 2. Absent a statute, it appears that the anticipatory breach remedy is generally unavailable, at least as to failure to pay rent 3. In the case of a tenants abandonment, however, repudiation is clearcut, and here anticipatory breach will apply if the jurisdiction in question extends that contract doctrine to leases c. Security Devices 1. A landlords best security is judicious selection of tenants 2. Security Deposits a. The purpose of security deposits is to protect the landlord in the event a tenant defaults in rent, damages and premises, or otherwise breaches the lease i. Problem things get bent in practice; the landlord has an incentive to imagine all sorts of reasons why he should be entitled to retain the deposit, and with money in hand if he has leverage that permits abuse 4. Lets put it together. Max is in lease contract with Laura. It was 5 year lease, $2,000 a month. After 2 years, Max leaves to attend his audition in CA. We are in No mitigation jurisdiction. Laura wants to know what her options are. a. Option 1: Sue at the end of the lease term. So do nothing for now. b. Option 2: Relets and let the original tenant off the hook a. The question for the landlord is do I accept SURRENDER? By leaving the property, Max surrenders. LL can accept or rejects. If accept surrender and relect, then Max is off the hook. c. Option 3: Relets and later sues the original tenant a. Rent on tenants account- shes rejecting the surrender but reletting the premise. So its like voluntary mitigation. d. Option 4: Anticipatory breach theory a. The court requires you to mitigate so calculation will be $2000/month current market value. So you get your money back but MITIGATION IS BUILT IN. 5. If its $2,500 a month market value right now, what do you advise? a. Rent on tenants account: nice to have back up because if new tenants dont work out, you can still hold the original tenant responsible. But windfall is that the original tenant may ask for the price difference you made from new tenants. b. A lot of times landlord would accept surrender. 6. Its $1,500 a month now. a. Probably not choose option 1. Possibility of not being able to find tenant later and possibility of vandalism. b. Anticipatory breach 7. In most cases, mitigation makes sense. The key is what Laura says to Max. Laura has to make sure whether she accepts the surrender. Tomorrow, do the property problem (Ho), then problems 422-429 for AM class (think about whether rent control makes sense). For PM Class, start recording act, 559-574, 580-583. Duties, Rights, and Remedies (Especially Regarding the Condition of the Leased Premises) i. Leases give rise to a rather obvious problem once a lease is entered into, the landlord has an incentive to neglect everyday repairs because the costs of neglect are borne primarily by the tenants 1. Tenants, in turn, have an incentive to neglect maintenance, especially toward the end of he terms, because the costs of neglect will soon shift to the landlord ii. Landlords Duties: Tenants Rights and Remedies

39

1.

2.

3.

4.

The early common law was hardhearted and hardheaded, about a tenants rights and remedies. The law implied covenants concerning title and possession, but not concerning the condition of the premises a. Absent some clause in the lease providing otherwise, the tenant took the premises as is, and landlords were under no obligation to warrant their fitness Given the culture in which the early common law rules developed, perhaps they made some sense but as the culture changed, largely in the direction of urbanization and specialization, so too did the common law a. Led to a body of conventional doctrine that was very stable until the 1960s when a period of sweeping reform began a. At first concerned with residential tenancies, much of it initiated by courts and not legislatures Disputed between landlord and tenant regarding the condition of the premises arise in essentially 2 ways: a. The tenant might wish to vacate, or to stay but pay less (or no) rent b. The tenant (or an invitee of the tenant) might be injured by allegedly defective premises and claim damages against the landlord in tort Quiet Enjoyment and Constructive Eviction a. Reste Realty Group v. Cooper Supreme Court of New Jersey, 1969 (422) a. In May 1958, leased the basement of a commercial office building from s predecessor in interest. used the basement for meetings and training of sales personnel. Whenever it would rain, the basement would flood with water because the driveway and surfacing was faulty. When the basement did flood, Donigan, an agent of the owners, promptly drained the water. After one year a new lease was negotiated for a term of 5 years. At this time, Donigan promised to remedy the flooding. The driveway was resurfaced, but the flooding continued. Donigan died in 1961. acquired the building and assignment of s lease in January, 1962. Whenever it rained thereafter, no one paid any attention to s complaints. It became very difficult for to hold her meetings there. Subsequently, vacated the premises. sued for the balance of the lease. contends that abandonment by was unjustified because the flooding was not permanent. also contends that abandonment was unjustified because inspected the premises and accepted the premises in their condition at that time. A trial court, sitting without a jury, sustained s defense of constructive eviction and entered judgment for . The Appellate Division reversed holding that (1) the proof did not support a finding of any wrongful act or omission on the part of the lessor sufficient to constitute a constructive eviction, and (2) if such act or omission could be found, waived it by failing to remove from the premises within a reasonable time thereafter. The SC of NJ affirmed the decision of the trial court, holding that had been constructively evicted from the premises in question and therefore was not liable for the rent was claiming b. may have signed a lease accepting the premises, but did not accept the driveway, the cause of the problems, because it was not part of the leasehold the condition should have been known to the lessor and the lessor should have disclosed the problem to the prospective tenant. s assertion that signed the 2nd lease is not of substance because it ignores the promise by c. S agent to remedy the problem the agent conceded the problem existed and promised to remedy the condition and therefore the is entitled to rely on the promise of its agent that it will provide a remedy. The lease in question contains an express covenant of quiet enjoyment for the term fixed . Where there is such a covenant then it is breached substantially by the landlord, the courts have applied the doctrine of constructive eviction as a remedy for the tenant. Under this rule, any act or omission of the landlord or of anyone who acts under authority or legal right from the landlord, etc. which renders the premises unsuitable for the purpose of which they are leased, or which seriously interferes with the beneficial enjoyment of the premises, is a breach of the covenant of quiet enjoyment and constitutes a constructive eviction of the premises. While constructive eviction usually requires that the flaw be permanent, the word permanent in previous cases does not signify that it must be an everlasting and unending condition regular flooding after rainstorms is sufficiently serious to amount to a substantial interference with use and enjoyment of the premises for the purpose of the lease, the test of constructive eviction has been met. The s decision to vacate was legal and whenever a tenants right to vacate leased premises comes into existence because he is deprived of beneficial enjoyment and use on account of acts chargeable to the landlord, it is immaterial whether the right is expressed in terms of breach of a covenant of quiet enjoyment, or material failure of consideration, or material breach of an implied warranty against latent defects. A tenants right to claim a constructive eviction will be lost if he does not vacate the premises within a reasonable time after the right comes into existence and what constitutes reasonable time depends on the circumstances of each case. In considering the problem, courts must be sympathetic toward the tenants plight. d. S building manager died 9 months before the vacated and patiently waited, hoping for relief from the landlord. When relief did not come, the tolerance ended and the vacation came after 10-days notice to the landlord e. A tenant may vacate premises and terminate the lease if his quiet enjoyment is interfered with by the landlord b. Notes on Reste Realty a. Landlord () relies on Stewart v. Childs, which said that constructive eviction was not possible unless the condition is permanent the court distinguishes 1. Independent Covenants the breach of one party does not excuse the performance of the other party 2. Dependent Covenants breach of one party excuses the performance of the other party 3. Historically, in landlord-tenant law there were 2 dependent covenants: tenant had an obligation to pay rent and the landlord had an obligation to give legal possession to the tenant i. This is the approach taken in Stewart but its not adopted by this court If the act/omission by the landlord creates a situation where there was substantial interference on the purpose for which the property was leased, whether or not its an express provision that was breached (can also be an implied covenant) the tenant may claim constructive eviction as a remedy (leaving the tenant who vacated off the hook for the remaining rent) c. Notes a. The Theory of Constructive Eviction 1. the obligation to pay rent was dependent upon the tenants having possession undisturbed by the landlord i. If one can characterize a shortcoming in the leased premises as an unlawful disturbance by the landlord as a breach of the covenant of quiet enjoyment implied in all leases and if

40

b.

c.

d.

5.

6.

The Illegal Lease a. Only applies where code violations are present at the time the lease is made and not if something becomes a violation after the creation of the lease b. If a lease is illegal because it is made in violation of statutory prohibitions, it is unenforceable c. Tenant in an illegal lease is a tenant at sufferance, and the landlord is entitled to the reasonable rental value of the premises, given their condition d. From the tenants point of view, the chief attraction of the illegal lease defense is the leverage it provides: the tenant can withhold rent and still stave off the landlords inevitable action to evict for nonpayment The Implied Warranty of Habitabilty a. Hilder v. St. Peter Supreme Court of Vermont, 1984 (431) a. In 1974, and her 3 children rented an apt. from for $140/month. has paid all rent due. Upon taking possession of the apt., discovered a broken kitchen window. promised to repair it but failed to do so. Eventually, repaired the window at her own expense because she was concerned that one of her children might get cut. There was also no key to the door. promised to provide a key but never did. The toilet was full and clogged; it would only flush if water was poured down it. The bathroom light and wall outlet were inoperable. In order to have light in the bathroom, attached an extension cord to an outlet in an adjoining room and ran it to the bathroom. Water leaked into the apt. from an apt. above. As a result, a large section of plaster fell onto s bed and her grandsons crib. Also, there was a strong and offensive odor in the apt. All of these things promised to remedy, but never did. The trial court ruled these faults breached the warranty of habitability and awarded the total rent paid. contends it was error to award the full rent because never vacated. contends that abandoning the premises is not required. The judgment is affirmed in part and remanded for a calculation of damages b. Historically the rule was that the lessee took possession in whatever state the premises were in. The landlord had no duty to make the premises habitable unless there was an express covenant in the lease. This is because a leasehold was viewed as an estate in land: the land was the essence of the conveyance, not the dwelling. The tenant was excused from paying rent only if the landlord ousted the tenant. In modern times, a tenant leases land for a safe, sanitary dwelling, not for arable land. Moreover, the characteristics of todays tenant have changed. The tenant of the middle ages was a farmer who was capable of making necessary repairs himself. In contrast, todays tenant is a city dweller who is unable to repair complex living units. On the other hand, landlords are in a much better position to repair residential units. Given these changes, the court now holds that there exists in every residential lease an implied covenant of habitability. Moreover, this warranty cannot be waived by a tenant. In determining whether this warranty has been breached, the courts may look to the local municipal housing code or to the minimum state housing code standards. These codes and standards need not be dispositive, however. These standards should be used to determine if the premises are safe. Once the warranty has been breached, the tenant may pursue rescission, reformation and damages. Damages awarded shall be the difference between the value of the residence as warranted and the value of the premises as defective. The tenant may recover for discomfort and annoyance, and he may repair the defect and deduct the expense from the rent. The tenant may also withhold rent until damages are calculated. The burden of bringing suit will then be on the landlord, who can better afford to bring the action. Finally, the may pursue punitive damages c. There is an implied warranty of habitability in every residential lease d. Notes on Hilder 1. If Reste were controlling, could have abandoned and claims constructive eviction however, the court here is giving the tenant more options i. Constructive eviction requires that the tenant move out; here, people wont be forced to move out in order to collect damages ii. Socioeconomically, poor people will be the ones dealing with this kind of situation and the court understands this there are people with very limited resources who wont be able to pay for lawyers, etc. These people should not be forced to litigate because they cant afford it

the disturbance was so substantial as to amount to eviction, and if the tenant thereafter abandoned the premises, then is was as though the tenant had been evicted (and the eviction was constructive) and once evicted, the tenant was relieved of the obligation to pay rent Scope of the Covenant of Quiet Enjoyment 1. Initially limited to cases in which the tenant was actually ousted physically, but with time (as in Reste) it was expanded to include beneficial enjoyment 2. Without quiet enjoyment and constructive eviction, breach of such obligations would ordinarily entitle the tenant only to sue for damages with the doctrines, the tenant may also abandon if the breach is substantial Partial Eviction actual and constructive 1. If there is an actual eviction, even though from part of the premises only, the tenant is relieved of all liability for rent notwithstanding continued occupation of the balance 2. In most jurisdictions, if there is a constructive eviction that makes only part, not all of the premises uninhabitable, most jurisdictions do not release the tenant of an obligation to pay rent Tenants Remedies 1. judicial decisions sometimes say that an eviction, actual or constructive, is necessary to constitute breach of the covenant of quiet enjoyment, but that is incorrect i. the view that eviction is necessary reflects conceptual confusion between a breach of the implied covenant on the one hand and the remedies available for breach on the other ii. Once the covenant of quiet enjoyment was broadened beyond that notion, it should have been understood that the actionable interference by the landlord could be remedied other than by the tenants abandoning the premises so the tenant should be, and usually is, able to stay in possession and sue for damages equal to the difference between the value of the property with and without the breach

41

2.

3.

4.

b.

Notes (437) a. The Implied Warranty of Habitability 1. The implied warranty of habitability does not render pointless the doctrines of quiet enjoyment, constructive eviction, and illegal leases i. First, a good handful of jurisdictions have yet to adopt the warranty ii. Second, the warranty, even where generally applicable, commonly does not apply to all residential leases iii. Third, a majority of jurisdictions has declined to extend the idea to an implied warranty of fitness or suitability for purpose in commercial leases 2. Standard and Breach of the Warranty

If the tenant has the power to withhold rent, deduct rent, etc., the landlord is much more prepared to sue which would allow the tenant the opportunity to respond an understanding that a response is easier than bringing a suit and that landlords and tenants are not operating at arms length Implied Warranty of Habitability i. When can a tenant claim theres a breach of the implied warranty of habitability? When the tenant has given notice of a serious problem and the landlord has failed to fix it; defect must be substantial and impact the tenants health and safety; and its not necessary to show that the property is not liveable just a necessity to show that there is an impact on health and safety Policy warranty of habitability is a good thing because there are things we care about such as basic human dignity i. This is good policy. Freedom of contract assumes people are autonomous and equal in bargaining power and any contract they make will make each party better off. However, there is a concern for unconscionability because in some situations theres a discrepancy in bargaining power that it doesnt make sense If the warranty of habitability can be waived, every landlord will put that provision in his contract ii. This is bad policy. Some people have different standards. There may be people willing to take something cheap at the cost of poor living conditions Posner for a certain class of tenants, implied warranty of habitability wont make a difference i. Where a landlord will make the most money if his apts. are habitable, he will make them so ii. Where a landlord wont make too much money but where there are statutes requiring a warranty of habitability, the apts. will be made habitable iii. Where it will cost the landlord more money to make the apts. habitable than he will profit, he will sell the building and turn it into condos or co-ops, or if in a neighborhood where that wont work, the landlord may abandon the building iv. Problem the last situation will happen in marginal neighborhoods and what it will end up doing is removing property from the rental market, which will drive apt. costs way up because there will be greater demand (since youve taken apts. out of the market because theyre uninhabitable) v. Market should take care of this problem. Over protecting is over paternalism. Tenants are market participants, so if one landlord waives the warranty of habitability, others will do so too. So market will correct and dont need to worry about waiving happening all at once. vi. One assumption Posner bases his arguments is that tenants are market participants and let tenants have freedom of contract and have autonomy. 1. Sometimes transforming what tenants want to dollar value is not always practical. Another assumption is that profit margin is so small that landlords wont stay in business if they have to spend money to fix up the housing. But where did that come from? 2. We dont know and we dont have real information about what landlords profit margin is. Cant quantify anyway, so no empirical data. 3. Not having the warranty has real hazardous consequences, i.e. fire code, etc. In other words, when people make agreements, they sometimes fail to consider all externalities or consequences their decision will create for others, i.e., pollution protection. Suppose two people agree to waive the warranty and their houses are sub-standard, even if they are fine for the parties, it devalues surrounding property, infrastructure around is not good, laboratories for crime, etc. 4. We are valuing freedom of contract because of its efficiency. But, when one party is not fully informed of an important fact, the contract is not efficient. Then, in those cases, there is market failure and its not working, so government intervention is needed. Posner also assumes that both parties have equal bargaining power. 1. Lack of choice, desperation limit poor peoples power. 2. Landlord thought it through and understands the rights and duties versus someone who didnt have education. 3. Unconsciounability- this is an interference of freedom of contract but its also a protection for a contract that was made in bad-faith or with lack of information. Warranty prevents race to the bottom by setting up the minimum habitable conditions. Posner assumes that every tenants goes into a contract with complete knowledge of what shes getting and what she will get in the future. 1. But thats not true. Sometimes you later find deterioration of quality of services. Then, youre paying more than what youre getting. vii. Warranty standard will happen anyway in high-end markets. Its the problem with lowend market. A lot of people wont know they are giving up their warranty. viii. Posners argument make a lot of assumptions. The difficulty is that we dont know the information he bases for his assumption is true or not. Some people will say Posner makes sense and some wont.

iii.

42

g.

Generally speaking, an adequate standard of habitability has to be met, and a breach occurs when the leased premises are uninhabitable in the eyes of a reasonable person ii. The objective is safe and healthy housing; substantial compliance is required means more than merely avoiding slum conditions 3. Remedies for Breach i. The implied warranty of habitability is based largely on contractual principles, and a number of cases agree with Hilder that in the case of breach a tenant may avail himself of all the basic contract remedies damages, rescission and reformation ii. Almost all jurisdictions permit the tenant to raise the defense in a summary eviction action. If the tenant is successful, rent is reduced partially or totally (depending on the degree of breach), and the tenant may retain possession if he pays whatever reduced amount is determined iii. Considerable disagreement over how to calculate the rent reduction or damages c. Retaliatory Eviction (439) a. Conventional common law doctrine gave landlords virtually unlimited freedom to terminate periodic tenancies and tenancies at will upon proper notice, and to refuse to renew expired terms of years the landlords reasons were irrelevant and could be malevolent b. Most jurisdictions today, whether by statute or judicial decision, forbid retaliatory action by landlords renting residential space 1. A fairly common approach is to create a rebutable presumption of retaliatory purpose if the landlord seeks to terminate a tenancy, increase rent, or decrease services within some given period after a good faith complaint or other action by a tenant based on the condition of the premises 2. NY has an alternative approach a tenant may be evicted anytime after repairs have been made but that courts should be generous in allowing the tenant sufficient time, without the pressure normally exerted in a holdover eviction proceeding, to find other suitable housing 3. A landlords freedom to terminate tenancies is constrained by more than retaliatory eviction prohibitions, such as antidiscrimination measures and rent control laws The Problem of Affordable Housing i. Chicago Board of Realtors, Inc. v. City of Chicago US Court of Appeals, 7th Circuit, 1987 (444) 1. In 1986, Chicago enacted a Residential Landlord and Tenant Ordinance which essentially codified the implied warranty of habitability. It also contained new landlord responsibilities and tenant rights, such as: (1) a tenant may withhold rent if the landlord violates a term of the lease; (2) the landlord must pay interest on security deposits; (3) the security deposit must be held in an IL bank; and (4) a landlord is forbidden from charging more than $10 for late rent. contends that the ordinance violates the contracts clause of the Constitution, procedural and substantive due process, equal protection clause, and it is void for vagueness. The trial court denied a motion for a preliminary injunction, concluding that the property owners did not have a reasonable likelihood of prevailing on the merits. The s appealed and the court of appeals affirmed, stating that the ordinance was sufficiently spe cific and reasonable in light of its stated purpose to promote public health, safety and welfare. The US Court of Appeals, 7 th Circuit affirms and goes further in analysis 2. The reasoning of the appellate court judge is permitted, but as far as it goes, the US Court of Appeals judges agree and take it even farther by acknowledging the case that can be made for the unreasonableness of the ordinance . It is unlikely that the ordinances real purpose is the public health, safety and welfare and the quality of housing in Chicago. Moreover, it is not likely to be the actual result because, if landlords cannot charge more than $10 for late rent and if tenants can withhold rent, landlords will not improve housing as much, since it will no longer pay. In addition, landlords will try to raise rents. This will hurt tenants and force those who cannot pay the higher rent to go homeless. If the landlords cant raise rent, they will devote more resources to condos, reducing the stock of housing. The provision that requires that security deposits be placed in IL banks serves only to transfer wealth from landlords and out-of-state banks to tenants and local banks. This is class legislation and protectionism. This will really benefit the middle class, not the poor. This is because the middle class will get preferential treatment by the landlords because they are a better risk against late rent. Further, the middle class will benefit from over-supplied and lower prices condos. This ordinance transfers wealth from out-of-state banks and tenants to the middle class. This becomes clear when one realizes that out-of-state individuals cant vote in Chicago elections and that the poor rarely vote 3. A rent control ordinance which makes minor re-allocations of rights between landlords and tenants is reasonably related to a legitimate public goal 4. Notes on City of Chicago a. Judge Posner articulates the argument against rent control and doctrines such as the implied warranty of habitability. Though such attempts are noble, he says, they have the opposite effect of that intended. Courts are reasonably free to develop doctrines to help the plight of the poor, and the implied warranty of habitability is an example of such an attempt. However, the courts are almost powerless in striking down attempts by the legislature to help the poor, even when these attempts have the opposite effect of that intended ii. The Debate Over Landlord-Tenant Reforms 1. Rent Controls a. Impetus for rent control a response to WWII a. Pretty much everyone was behind the War and when it ended, there was a huge increase in demand for housing. The costs went way up because supply was limited (there was no residential construction during the War because all supplies were diverted to the War effort) 1. It seemed almost immoral to tell the returning soldiers that the market would determine the cost of housing 2. NY legislature froze all rents on existing apts. in NYC except for new construction (incentive to build) i. The permanent freeze cant be sustained and so eventually, theres a change to rent stabilization b. Virtually all American economists view rent controls as counterproductive (447) a. Now that theres a certain pool of housing stock that is unregulated and older construction that is rent stabilized is this a good solution to the problem of affordable housing? 1. Yes (reasons why we dont get rid of this)

i.

43

2.

VIII. Title Assurance a. The Recording System (559) i. The system our country has developed to assure purchasers of land that they have good title to the land purchased 1. At the heart of the system is the public records office, where all instruments affecting land titles are recorded 2. From the evidence of title in the records office, a professional will conclude who has the fee simple title to the land, which may be encumbered with a mortgage or a servitude 3. Under title registration, the state registers title and issues a title certificate to the owner, which is reissued to each new purchaser of the property ii. Introduction (559) 1. Public recording of deeds, mortgages, leases, and other instruments affecting land title began in this country in Plymouth, MA colonies around 1640 it was not an English custom 2. Statutes provide for land title records to be maintained by the county recorder in each county a. Records include copies of documents filed with the recorder and indexes to these copied documents 3. A deed is valid and good against the grantor upon delivery without recordation 4. Recording system serves 2 other functions: a. Establishes a system of public recordation of land titles. Anyone can ascertain who owns land in the county by searching the records b. The recording system preserves in a secure place important documents that, in private hands, may be easily lost or misplaced 5. Recoding statutes often specify what instruments can be recorded, but generally any kind of deed, mortgage, lease, option, or other instrument creating or affecting an interest in land can be recorded a. Prior to rejudgment in a lawsuit affecting title to real property, any party may record a lis pendens (notice of a pending action), which will effectively put subsequent claimants on notice of the claims being litigated 6. Recording acts have the function of protecting purchasers for value and lien creditors against prior unrecorded interests 7. The recording acts in general have adopted and broadened the equitable doctrine of bona fide purchaser a. Under the recording acts, a subsequent bona fide purchaser is protected against prior unrecorded interests b. Thus a purchaser of property will want to search the records to make sure that there are no adverse prior recorded claims, and a purchaser records his deed in order to prevent a subsequent purchaser from a previous owner from prevailing over him iii. The Indexes 1. 2 types of indexes currently used in the US a. Tract Index indexing documents by a parcel identification number assigned to the particular tract, do not exist in most states b. Grantor-Grantee Index the most common method of indexing a. Separate indexes are kept for grantors and grantees 1. Grantor Index all instruments are indexed alphabetically and chronologically under the grantors surname 2. Grantee Index all instruments are indexed under the grantees surname b. There may also be separate indexes for each type of instrument (deeds, mortgages, wills, liens, etc.) c. The reference to a document sets forth its essentials: the grantor, the grantee, description of the land, kind of instrument, date of recording, and volume and page numbers where the instrument can be found set forth in full 2. How far back? a. How far back in time a title searcher must search to find a satisfactory root of title varies both with local custom and with the identity of the client b. Purchasers of commercial properties make far more extensive searches to ascertain previous owners and possible polluters of the particular property they are interested in buying c. Agencies in the federal government customarily require a search back to the original source 3. Luthi (subsequent assignee Burris) v. Evans (Tours) Supreme Court of Kansas, 1978 (565) a. Owens owned interests in several oil and gas leases in Coffey County. On 2/1/1971, she assigned all of these oil and gas interest to International Tours, Inc (Tours) through a written instrument. This document stated that Owens and the other

this is about homes and not businesses. We want people to be able to count on rent stability because if left to the whims of the market, rent will suddenly jump up and the people in those apts. are out of a home because of an increased demand ii. allow people to plan their lives and neighborhood iii. Getting rid of it will NEVER happen because its political suicide. No there is no qualification for a rent stabilized apt. (you just have to be there before it became a rent-control building) and the people in the building will get to stay for as long as they want and can even pass their interests on to family. The people seriously hurt by this will be the newcomers and not the people who are already there. i. Rent stabilization keeps apts. off the market, decreasing the availability of apts. on the market and driving up the prices of those that are available (newcomers have to pay because theyre not residents and they cant vote) ii. This isnt about helping people with low incomes because it protects entrenched voter interests the people who vote are the ones with rent-stabilized apts. iii. Landlords will not invest money, so more goes to be co-op. So middle to low income housing decreases because buildings are decreasing in number. iv. Because rent-controlled buildings tenants stay although they dont need that much space and it prevents newcomers to get that apartment. If the apartment is on market-value rent, then if they dont need space, its more likely they will downsize and that space will be available to others. v. Also once a rent-stabilized apt. opens up, a landlord will be more likely to discriminate/show personal preference in order to select the best tenant; the landlord might also find other ways to get money; or select only the best, most financially solid tenants

i.

44

assignors intended to convey all interest whatsoever of working nature in all working interests and overriding royalty interest in all Oil and Gas Leases in Coffey County. This sort of clause, conveying all of a grantors property in a certain county, is commonly referred to as a Mother Hubbard clause. This assignment was recorded in the county office of the register of deeds on 2/16/1971. In addition to the sever leases specifically outlined in the agreement, Owens owned a working interest in an oil and gas lease known as the Kufahl lease. This lease was also located on land in Coffey County. Although it was not specifically described in the assignment, the Kufahl lease nonetheless fit the description in the Tours assignment. On 1/30/1975, Owens executed and delivered a 2nd assignment of her interest in the Kufahl lease to Burris (). Before this assignment took place, personally checked the records in the office of the register of deeds. After the assignment, obtained an abstract of title to the same property. Neither his inspection nor the abstract revealed the prior assignment to Tours. Tours contends that the 1971 assignment effectively conveyed Owens interest in the Kufahl lease to Tours and that its recordation with the county gave constructive notice to all subsequent purchasers, including . , however, asserts that the language of the assignment provided an inadequate description of the interests conveyed. prevailed in district court. On appeal, the general description in the 1971 assignment was held to be sufficient, when recorded, to provide constructive notice. The judgment of the district court prevails (the assignment to prevails) Feb, 1971, Owens Kufahl lease to Tours (recorded) Jan, 1975, Owens Kufahl lease to Burris A Mother Hubbard clause is upheld as between the parties to the instrument that contains it, but it s insufficient to give constructive notice to subsequent purchasers without actual notice of it. This court agrees with both parties that the 1971 assignment constituted a valid transfer of Owens interest in the Kufahl lease to Tours, but only as between the 2 parties to that assignment. Likewise, a single instrument can convey separate tracts by specific description, and by general description which can be made specific. This can be done when the language of the assignment expresses that clear intent. The state statutes, however, require the register of deeds to keep a general index of title that records a description of each tract conveyed when appropriate. Further, county commissioners are required to record a numerical index of all deeds, complete with brief descriptions of the properties involved. It is clear that these statutes are intended to notify a subsequent purchaser regarding instruments which affect the title to a specific tract of land. Thus, the legislature likely intended that recorded instruments describe the conveyed land well enough to that it could be identified later on. An instrument using a Mother Hubbard clause is ineffective as to subsequent purchasers unless they have actual knowledge of the transfer. A Mother Hubbard clause can be used in the event of emergency, so long as the grantee takes steps to protect his title against subsequent purchasers. These steps could include taking possession of the property or filing an affidavit with the county containing a more appropriate description of the property. Here, because had no actual notice of the 1971 assignment to Tours, the assignment to prevails c. A Mother Hubbard clause is upheld as between the parties to the instrument that contains it, but is insufficient to give constructive notice to subsequent purchasers without actual notice of it d. Notes on Luthi a. Mother Hubbard clause is not in the recording, its in the deed. That copy of index should have a copy of the lease of Tours. can argue that Burris should have looked at the lease on file to see if there was a mother hubbard clause, but he failed to do so, so he looses. b. Ramifications if the court held that the assignment to Tours prevailed 1. If the unclear and non-specific Mother Hubbard clause is valid, it creates a huge cost for someone in s position couldnt rely on the index and would have to do research another way (expensive and time consuming) i. Would have had to read every single deed the seller might have engaged in while he owned the property ii. Toures, or someone in s position would not have too much burden to clarify what he owns in the recording iii. Is describing all the property youre buying must be described in the index so burdensome? No. So Tours is the cheapest cost avoider. c. Mother Hubbard clauses are ok, but need to be specific and if theyre done hastily, someone needs to go back and make it clear what land is sold d. Tours could have avoided all of this if he had recorded the deed properly e. If Burris knew about Tours transaction, would it change the outcome? Yes. Right now, Burris wins because he was a subsequent bona fide purchaser. But if he knew he had notice, so why protect him? Hes no longer a subsequent bona fide purchaser. He should have figured it out and not go through with the transaction with Owens. Types of Recording Acts 1. Race Statutes the person who wins the race to record prevails. Whether a subsequent purchaser has actual knowledge of the prior purchasers claim is irrelevant a. Virtue of a race statute for the tile searcher is that it limits inquiry into matters off the record the question of who knew what, which is often difficult to ascertain and harder to prove, is not relevant. Transfer of title is more efficient where offrecord inquiries are eliminated 2. Notice Statute if a subsequent purchaser had notice of a prior unrecorded instrument, the purchaser could not prevail over the prior grantee, for such would work a fraud on the prior grantee a. Virtue of a notice statute is its fairness as between 2 conflicting claimants, but inasmuch as the question of whether the subsequent purchaser has notice depends on facts not on record, notice statutes are less efficient than race statutes b. Shelter Rule A person who takes from a bona fide purchaser protected by the recording act has the same rights as his grantor. This rule is necessary if the recording act is to give a bona fide purchaser the benefit of his bargain a. Ex. O conveys to B and before B records, A records his deed. Thereafter, C desires to purchase from B. C, searching title would find As deed on record, and C would have to ascertain from facts off the record whether B had notice of As deed. If B did not have notice, B would prevail over A and C can buy from B and, standing in Bs shoes, can prevail over A 3. Race-Notice Statute a subsequent purchaser is protected against prior unrecorded instruments only if the subsequent purchaser is (1) without notice of the prior instrument and (2) records before the prior instrument is recorded b.

iv.

45

4.

5.

6.

7.

8.

It has been suggested that a race-notice statute tends to eliminate lawsuits turning on extrinsic evidence about which deed was delivered first b. It has also been suggested that a race-notice statute is preferable because, by punishing nonrecording, it provides motivation to record, making the public records complete Example 1: O conveys to A. Later O conveys to B. B has actual notice. A records first. A v. B a. Statute on page 569: 58-2223- notice statute (subsequent purchaser can prevail if they had no notice, but the presumption is that that person must record first too) a. Notice statute jurisdiction: For B to prevail A, b has to show 1. Subsequent bona fide purchaser of value without notice b. There are 2 innocent parties, A&B. The court tries to protect them. If court says A prevails, its not fair to B because B had no way of knowing about A. A could simply avoid all this by recording promptly. So simple but didnt do. Example 2: same facts as 1 a. Statute on page 585: Race notice statute. Presumption is A wins (whoever recorded first). But the statute says every conveyance not recorded.. may be void in good faith = without notice. b. In order for B to prevail, B has to have recorded first and had no notice. c. But B had actual notice, so he cant win. Example 3: O to A; O to B with notice. A records. B transfers to C. A v. C a. In Notice jurisdiction: B had actual notice, B cant prevail A. A prevails from A v. B. a. When C does title search, As recording should pop up, so C should have known that B cant transfer to C. This is record notice which is a part of constructive notice. b. We are charging C with actual notice although C may not have had it. b. In Race Notice jurisdiction: A prevails because B had actual notice and A recorded first. In other words, B had nothing to convey to C. a. Then, is C a subsequent bona fide purchaser? No because C is presumed to have actual notice. c. In race jurisdiction, A wins because A recorded first. d. Important: Presumption is the original purchaser, A, wins. But depending on what B did and what A did, B may prevail. So, dont ask question whether A had notice at the time of purchase, because A purchased first and there was nothing for A to be have notice about. e. Notice structure: 1. Actual; 2. Constructive- a. record; b. inquiry f. Change: B transfers to C. Then A records a. In notice jurisdiction: A prevails because B has actual notice. But A didnt record promptly and B conveyed to C and C is now a subsequent bona fide purchaser. So, now C prevails over A because A could have prevented all problems by recording promptly. b. In race jurisdiction: A prevails because A recorded. c. In Notice-Race jurisdiction: A prevails because B had actual notice. B had nothing to convey. But A recorded first, before C. So C cant prevail although C might not have had an actual notice. Example 4: O to A; O to B with no actual notice; A records; B conveys to C; A v. C a. In Race statute: A wins because A recorded first b. In notice statute: B prevails because B had no notice and A didnt record promptly. a. Then B could legitimately convey to C. So between A and C, C prevails [shelter rule] C is protected by an effort to protect B. So C is secured/protected under the shelter of B. b. If A recorded before O conveyed to B, A would prevail because B should have been on notice by title search. If B didnt search or didnt know after the search, thats Bs fault. c. In Race-notice statute: B couldnt prevail against A because B didnt record first. So B had nothing to convey to C. So between A and C, A prevails. Messersmith v. Smith Supreme Court of North Dakota, 1953 (583) a. Caroline Messersmith and her nephew, Frederick Messersmith () each owned an undivided interest in 3 sections of land in ND. On 5/7/1946, Caroline executed and delivered a quitclaim deed to the property to which was not recorded until 7/9.1951. From January to April 1951, oil fever hit the County, as brokers, oil men, and speculators flocked to ND. On 4/23/1951, Caroline met with Herbert Smith () to discuss a gas and oil lease to the properties. Caroline claimed that only royalties were negotiated, which claimed the matter of the mineral deed was discussed. On 5/7.1951, Caroline executed a mineral deed for an undivided interest in the oil, gas and minerals under the 3 sections of land. Caroline claims that she thought she was signing a transfer of royalties only. paid a consideration of $1,400 for the deed. After leaving, discovered that the deed incorrectly stated the term his heirs instead of her heirs. returned to Carolines home the same day and explained the mistake to her. then tore up the deed and prepared another one in the same form and with the error corrected. then, according to s testimony, took the deed to the same notary public to whom Caroline acknowledged the first deed. The notary called Caroline for her acknowledgement over the phone, and then placed his seal and signature on the deed. 2 days later, executed a mineral deed conveying the interest conveyed by Caroline to E.B. Seale. Both deeds were recorded 5/26/1951. In July 1951, records. Seale apparently relied on the second deed between Caroline and when making his purchase of the interest. filed an action to quiet title to the land. Seale () answered by claiming he was a purchaser without actual or constructive notice of s claim. further answered by claiming s mineral deed was never acknowledged, not entitled to be recorded and was obtained by fraud, deceit and misrepresentation. The trial court found that the deeds were not procured through fraud or false representation. The SC of ND does not disturb the finding that there was no fraud or deceit, but reverses the judgment a. 5/7/46 Caroline to Frederick () 5/7/51 Caroline to mineral deed of her interest to Smith () 5/9/51 Smith to Seale 5/26/51 Smith records; Seale records 7/9/51 Frederick records b. ND is under Race-Notice Statute. An instrument that is not properly acknowledged is not entitled to be recorded. The trial court found that the deeds were not executed through fraud or false representation, and there is no reason to disturb that finding. Carolines mineral deed to however, was invalid, for she had already conveyed her interest to . s conveyance

46

v.

to was also invalid. The only way can assert title to any interest in the property is by citing the fact that s deed was not recorded until July 1951. This reasoning is insufficient, however, to support s claim because s deed was nonetheless improperly executed. Under the governing state statues, a deed to real property cannot be recorded without proper acknowledgement by the parties. Caroline did not appear before the notary and acknowledge she executed the deed that was recorded; she allegedly did so over the phone. Because the deed was not entitled to be recorded, the record then did not constitute notice of its execution. Because the record did not constitute notice, the purchaser (), did not become a subsequent purchaser in good faith within the meaning of the statutes. The right tries to claim is dependent on compliance with the recording statutes. It is likewise dependent on the deed that was actually recorded, not the one that was destroyed c. An instrument that is not properly acknowledged is not entitled to be recorded d. Notes on Messersmith a. The lower court found that in this race-notice jurisdiction, would win he records first and has no notice b. The SC reverses says that could not win because he was not a bona fide purchaser. The chain of title out from Smith is not valid because Smith didnt first record because the deed wasnt properly acknowledged 1. Race-notice requires both notice and recording first and here, didnt record first because he cant record a deed that hasnt properly been acknowledged c. Is this case decided correctly? 1. Yes because it looks like the court felt that Caroline was being taken advantage of (she had Alzheimer and there was oil fever and outsider was coming in like crazy) and they want to protect her to do justice in this case 2. No, this was not decided correctly this determination undermines the efficiency of the recording system if this rule were upheld, it would cause confusion and reluctance to purchase because it would mean you couldnt trust any recorded deeds, making a search nearly impossible d. Petition for rehearing the court emphasizes the fact that at the time Caroline conveyed to Smith, Caroline had nothing to convey, so Smith received nothing e. In race-notice jurisdiction: Between F and S, Smith wins because F didnt record first and S didnt have a notice. So why S looses? Isnt S, a subsequent bona-fide purchaser? a. Problem stems out of improperly executed notary. Over the telephone notary is not a properly acknowledged instrument. So it cant be recorded, so Ss recording which occurred before Fs recording is not acknowledged. (page 586) b. Were not worried about validity of deed. If deed was invalid, then no reason to worry about recording statute. f. If youre a lawyer for someone who wants to purchase after doing title search and it looks clear, then what should you advise? a. Deed looks like its properly executed. So, isnt it too much to ask everyone to go all the way to ask how the notary was done etc.? b. This makes no sense as a holding. It doesnt support the importance of creating an efficient recording system. SO how do you make a sense of it? c. Caroline not only had lucid and nonlucid period and she refused to take the stand and state she was sick. Nobody wanted to humiliate her and she didnt want to be humiliated. Thats why there is no record saying that getting someone to sign a contract when that person has Alzheimer and incompetent is invalid. So, hard case makes bad law. d. Later, ND legislature overturns this case. It said whats recorded is recorded, you cant say the recording is not entitled because deed wasnt properly executed. g. Example 5 on page 588- essentially Massersmith: O to A; O to B with no notice; B records; B to C; C records; A records h. Example 6- O to A; A records; O to B; etc??? Chain of Title Problems 1. Chain of Title refers generally to the recorded sequence of transactions by which title has passed from a sovereign to the present claimant a. Also has a more technical meaning: the period of time for which records must be searched and the documents that must be examined within that time period b. The standard title search required against each owner: from the date of execution of the deed granting title to the owner to the date of recordation of the first deed by such owner conveying title to someone else c. The meaning of chain of title varies from jurisdiction to jurisdiction; it includes the series of recorded documents that, in the particular jurisdiction, give constructive notice to a subsequent purchaser 2. Board of Education of Minneapolis v. Hughes Supreme Court of MN, 1912 (590) a. On 5/16/1906, Hughes () offered Hoerger $25 for a vacant lot. Hoerger accepted the offer, and sent a check for this amount to Hoergers husband, along with a deed to be executed and returned. The space for the name of the grantee on this deed was left blank. The Hoergers executed and acknowledged this deed on 5/17/1906, and mailed it back to . The grantee space on the deed, however, was still blank. The Hoergers cashed s check. On 4/27/1909, Duryea and Wilson, real estate dealers, paid Mrs. Hoerger $25 for a quitclaim deed to the same lot. This deed was executed and delivered. On 11/19/1909, Duryea and Wilson executed and delivered a warranty deed to the lot to the . recorded this deed on 1/27/1910. only found the time to fill in his own name on his deed shortly before recording it on 12/16/1910. Duryea and Wilson finally recorded their deed from the Hoergers on 12/21/1910. No explanation was given as to why none of these parties, with the arguable exception of , failed to file their deeds any sooner than they did. In an action to determine adverse claims to a lot, the trial resulted in a decision in favor of and appealed. SC of MN reversed and granted a new trial 5/17/06 Hoerger to Hughes () [deed: grantee is left blank] 4/27/09 Hoerger to D&W 11/19/09 D&W to Board of Ed () 1/27/10 Board records [wild deed because the chain of title is not connected] 12/16/10 Hughes inserts name & records --- court says this is when Hughes became grantee 12/21/10 D&W records

47

3.

A deed from the grantor, even if recorded, is treated as though it were unrecorded and gives no constructive notice. First, a preliminary issue must be discussed. The initial deed to from the Hoergers did become operative when put his name on it before recording it. Before that name was filled in, the deed itself was a nullity. had implied authority to fill in the blank. It is to be presumed that the grantee has the authority to do this when the grantor retains the consideration for the deed, and delivers the deed to the grantee, just as the Hoergers did here. Turning to the issue of the deed to , the facts support s claim to title. When Duryea and Wilson deeded the property to in November 1909, there was no record showing they had anything to convey. The Duryea and Wilson deed was not recorded until 12/21/1910. By contrast, was a subsequent purchaser, and is protected by the fact that his deed was recorded 5 days before the deed from the Hoergers to Duryea and Wilson. At the time recorded, Duryea and Wilson were still not a record owner anywhere in the chain of title. was thus the rightful owner of the lot c. A deed from a grantor outside the chain of title, even if recorded, is treated as though it were unrecorded and gives no constructive notice d. Notes on Board of Ed. of Minneapolis a. This was a race-notice jurisdiction and was a subsequent purchaser with no notice (Duryea and Wilson hadnt recorded title when the recorded) 1. When the Board recorded, the problem was that the Boards predecessor in interest had not recorded, making the Boards deed wild 2. If Hughes were to do a title search, the Boards title wouldnt come up and a wild deed is not sufficient to give notice to a subsequent purchaser 3. In order to win, the Board would have to show that Duryea and Wilson would prevail over under the Shelter Rule (if a notice jurisdiction) b. Wild Deed generally, it is a deed to property that is recorded, but not within the chain of title because neither the grantor nor the grantee is known to the searcher of the title in official records. Here, the term wild deed refers to the deed from Duryea and Wilson to the Board c. Board of Ed should have made recording of D&W to be condition to the conveyance. d. What if Hughes recorded after 12/21/10? Then Hughes loose because he then has a notice by looking at D&Ws recording and Boards recording e. If you have a client in Board of Ed situation and youre in Notice jurisdiction, what do you do? 1. The court said that board was the first purchaser and not subsequent purchaser. And this is a precedent. SO deal with this first. 2. Courts timeline is: Hoerger to D&W (O to A)- original purchaser D&W to Board (A to C) Board records (C records) Hoerger to Hughes (O to B) Hughes records (B records)- subsequent purchaser D&W records (A records) 3. We can try to limit the holding as wild deed holding. Sort of overrule/disregard first part of the opinion and focus on the wild deed argument. 4. When Hughes put his name in is irrelevant to the final holding, thus the first part of the opinion is dicta. In Race-Notice jurisdiction, if you look at Hughes as the first in time, Hughes wins. If you look at Hughes as the second in time, Hughes still wins. 5. But now in Notice jurisdiction, whether he is first in time or second in time by when he inserted his name becomes outcome-determinative. If Hughes was the first purchaser in 06, then D&W becomes the subsequent purchaser and is because didnt have a notice (because Hughes didnt record first) hes protected. So, transaction between D&W and Board is valid because D&W is a bonafide subsequent purchaser. (do not need shelter rule because Board didnt have notice; you need shelter rule if the 3 rd transactor had notice but need to be protected under the 2nd purchasers protection) Daniels v. Anderson Supreme Court of Illinois, 1994 (598) a. In 1977, bought 2 lots from Jacula (). The contract of sale gave the right of first refusal (if want to sell, has the first right to buy. If refuse, then can be sold to others) if ever decided to sell an adjacent parcel for the same price as any prospective buyer offered. received and recorded the deed, which did not mention the right to first refusal. The contract of sale (which contained right of first refusal clause) was not recorded. In 1985, Zografos () contracted with Jacula to buy the contiguous parcel for $60,000. was not notified of the offer. Zografos () paid $10,000 and gave Jacula () a note for the balance. Zografos paid $30,000 more in early 1986. s wife tells Zografos wife about the prior right of refusal interest. In August 1986, Zografos () paid the remaining $20,000 to Jacula (). Zografos received and recorded the deed to the contiguous parcel. sued Jacula () and Zografos () for specific performance of the preemptive option. Zografos () contended that he was a bona fide purchaser without notice of the option. The trial court held that Zografos () was not a subsequent bona fide purchaser because he had action notice of the option at the time he took title. The court ordered Zografos to convey the contiguous parcel to , and ordered to pay Zografos the full purchase price of $60,000 plus $11,000 in property taxes that Zografos had paid on the contiguous parcel. Zografos appealed on grounds that he became a bona fide purchaser because he took equitable title prior to receiving actual notice of s interest, even though he did not take legal title until after he had notice. The appellate court affirmed, holding that Zografos waived this theory because he did not assert it any time prior to the appeal. Zografos appealed to the SC of IL, which affirmed b. Where a buyer receives notice of an outstanding interest subsequent to paying some, but not all, of the full purchase price, the buyer is not considered a bona fide purchaser. A bona fide purchaser, by definition, takes title to real property without notice of the interest of others. Some appellate courts have held that partial payment of the consideration is insufficient to render the buyer a bona fide purchaser. A majority of jurisdictions have relaxed this harsh rule and have applied a pro tanto rule, which protects the buyer to the extent of the payments made prior to notice, but no further. There are 3 methods employed to apply this pro tanto protection: (1) the most common method is to award the land to the holder of the outstanding interest and award the buyer the payments that he or she made this case; (2) award the buyer a fractional interest in the land proportional to the amount paid prior to notice; (3) allow the buyer to complete the purchase, but to pay the remaining installments to the holder of the outstanding interest. Courts can exercise considerable latitude in reaching an equitable resolution. The court holds that the trial courts disposition of the issue between and Zografos was fair and not an abuse of discretion b.

48

4.

5.

6.

Bona fide purchaser status attaches only when the full purchase price has been paid Courts have a lot of latitude, and in this case he the judge did not abuse his discretion. Its not fair for Problem: Were in IL after Daniels. 2005, J to prior purchaser for $40,000; 2006, J to subsequent purchaser for $30,000 now and $30,000 in two years; In 2007, prior purchaser records; Prior brings action to quite title; the house is now worth $100,000. Daniels v. Anderson is controlling and Lewis v. Superior Court is decided in CA (persuasive authority). a. Daniels Rule is that unless a subsequent purchaser paid in full and bought the land without notice, he is not a bona fide purchaser. Pursuant to Daniels, the subsequent purchaser cannot be a complete bona fide purchaser because he did not pay in full for the purchase although he didnt have notice. However, using the Pro Tanto Rule, court can 1. Let the prior purchaser have the land and let the sub. Buyer get his $30,000 back, 2. This is difficult because of the increase in value of the land. When 2nd buyer bought, he paid for 50% of the land price, so he feels that hes entitled to half interest in the land. But for 1st buyer, now the value is 100K so 2nd buyer should take 30% or 50%? 3. Let the 2nd buyer pay 300K to 1st buyer and 2nd buyer holds the land. b. Class discussion: What prior would want after Daniels, he probably wants to get the land and just give up $30,000 to the subsequent based on priors failure to record. The point is the subsequent is on notice before paying up the rest of the money; Priors argument: he should take the title even though he didnt record promptly because based on Daniels the subsequent cannot be a bona fide purchaser until he pays up. Strict application of Daniels. I will make the subsequent whole by giving him 30K. c. Subsequent probably want the property because its much better argument. But he only paid $30K. So the subsequent will pay the prior $30K and let me take the land; Argument: cant use Daniels but can use Lewis. Daniels was about actual notice and this case is about construction notice and this should make a difference because requiring the subsequent to do title search every time he pays is too burdensome; equitable title the minute the subsequent paid the first payment, the deposit, he got the title and he recorded first, so hes entitled + in Daniels the equitable title argument wasnt raised and the court was sympathetic. Now, we can make this one and only ask whether he had notice then at the time of first payment, so the problem of subsequent notice doesnt matter; must encourage recording immediately by punishing the prior who failed to record; property is unique so paying $30K would not make him whole made future plans based on purchase and expectation to get it, so equitable title makes sense. Lewis v. Superior Court California Court of Appeal, 2nd District, 1994 (600) [important reasoning] a. In February 1991, Lewis () contracted to buy a residence for $2.3 million. On February 24, Fontana Films () recorded a lis pendens [somebody has contingent interest and freezes the asset from being sold] on the property. On February 25, the s paid $350,000. Escrow closed on February 28, 1991, when gave the seller a note for $1.95 million. s had neither actual nor constructive notice of the lis pendens at this time. On February 29, 1991, the lis pendens was indexed. Within the next year, the s paid the note in full and spent over $1 million in renovating the property. In September 1993, the s were served with s lawsuit and learned about the lis pendens. brought suit to remove the lis pendens and clear title. contends that even if took title before the indexing of the lis pendens, they nevertheless were not bona fide purchasers because they did not fully pay for the property until after indexing. The trial court denied s motion for summary judgment and s appeal. The CA Court of Appeal issues a peremptory writ of mandate directing the superior court to vacate its order denying the s motion for summary judgment, and thereafter issue a new order granting the motion and expunging the lis pendens b. A seller need not be paid in full before the buyer can be considered a bona fide purchaser. relies on the courts antiquated holding in David v. Ward (a buyer becomes a bona fide purchaser only upon payment in full). The Davis payment of value rule cannot be reconciled with modern real property law and practice. The Davis rule was premised in part of the assertion that a purchaser who loses his property is not hurt if he has not fully paid for the land. This clai m is inconsistent with modern market considerations. Any purchaser without notice who makes a down payment and unequivocally obligates himself to pay the balance has every reason to believe that, if he makes the payments when due, his right to the property will be secure. Such a purchaser may drastically alter his position by, for example, selling his prior residence and making significant improvements to the property. The landowner cannot be adequately redressed by simply returning the money he paid so far. Real property is unique and its loss cannot be compensated in money. Furthermore, the Davis holding cannot rationally be applied to cases involving only constructive notice. A completely innocent purchaser who has only partially paid should not be punished for simply living up to his payment obligations. Moreover, he should not be required to undertake a title search before each and every payment. Finally, applying Davis would unfairly penalize the s for paying cash for the property, rather than financing t he purchase price. In Davis, the court recognized that if the buyer had taken out a mortgage and given a note to the bank, he could be a bona fide purchaser because the seller would have been paid in full. On the other hand, a buyer who gives a note to the seller would only be considered a bona fide purchaser when he made his final payment to the seller. This distinction makes no sense. It unfairly penalizes a buyer who arranges a cash transaction with the seller rather than taking out a mortgage c. A seller need not be paid in full before the buyer can be considered a bona fide purchaser d. Notes on Lewis a. Overturns the payment of value rule seen in Daniels when considering whether a buyer is a bona fide purchaser, it should make no difference when the seller is fully paid Alexander v. Andrews (603) a. Thomas and Mary, husband and wife, owned their home, worth $7700, as tenants in common. Mary died on April 25, 1946, devising her interest to their son Charles, who lived with his parents. On May 8, Thomas conveyed his interest in the house to his daughter, Sarah, for love and affection. On May 14, Thomas by a second deed conveyed his interest in the house to Charles, who did not know of the prior deed to Sarah. Charles paid Thomas $1000 and promised to take care of him for the rest of his life and bury him. Charles promptly recorded his deed. Sarah recorded on July 8. Charles lived in the house with Thomas and took care of him until he died b. The court held that Charles is a protected subsequent purchaser only to the extent of the $1000 paid before Sarah recorded her deed. The remainder of his consideration, lifetime care and burial costs, was paid after Sarah recorded her deed, which gave constructive notice to Charles of her prior claim Notes a. Creditors a. A number of recording statutes protect creditors against unrecorded deeds and mortgages c. d. e. f.

49

1. 2. 3.

Courts have interpreted these statutes to protect only creditors who have established a lien, such as by attachment or judgment, and not all creditors Merely lending money to the record owner does not give priority over unrecorded instruments In many states, a creditor is not protected until the creditor prosecutes a lawsuit to judgment and forecloses a lien or holds an execution sale i. The buyer at the sale, who may be the creditor, is protected under the recording act as a subsequent bona fide purchaser for value if the buyer has no notice of the unrecorded claim at the time of sale ii. In many states, special statutes give persons who provide labor or materials on a building project a lien on the property from the time such labor or material is provided

b.

Quitclaim Deeds b. In a few jurisdictions, a purchaser by quitclaim deed cannot claim the position of a bona fide purchaser without notice c. In a large majority of jurisdictions, however, a quitclaim deed is treated the same as a warranty deed for purpose of giving notice

vi.

Inquiry Notice 1. 3 kinds of notice a person may have with respect to a prior claim: a. Actual Notice arises where one is personally aware of a conflicting interest in real property, often due to anothers possession of the property b. Constructive Notice notice that the law deems you have regardless of your actual knowledge a. Record Notice notice one has based on properly recorded instruments b. Inquiry Notice based on facts that would cause a reasonable person to make inquiry into the possible existence of an interest in real property 2. Harper v. Paradise Supreme Court of Georgia, 1974 (604) a. In 1922, Susan Harper conveyed a deed for a farm to Maude, for life (Maud had life estate) with remained in fee simple to Maudes named children, including Clyde (). The deed was misplaced and not recorded until 1957 when found the deed in a trunk and recorded it. Susan died in or about 1925 and was survived by her heirs. In 1928, all of the heirs except one (John Harper) executed a deed to Maude. The language of this deed, which was recorded in 1928, expressly noted that the prior deed from Susan to Maude had been lost (Maud now has fee simple absolute). In 1933, Maude executed a deed which purported to convey the property to Ella Thornton as security for a loan. When Maude defaulted on the loan, Ella foreclosed. Ella received a sheriffs deed in 1936, which she recorded in 193 6. An unbroken chain of title existed between Ella and , who were grantees to a 1955 deed which was recorded in 1955. also assert title by adverse possession which began in 1940. claims their direct title from the 1928 deed from Susans heirs to Maude. alleges that the do not hold valid title because the 1928 deed provided notice of the prior 1922 deed to Maude. Maude died in 1972. From a directed verdict in favor of , appeals and the SC of GA reversed and remanded, with judgment to be entered in favor of 1922 Susan Harper to Maude for life Maude lost the deed 1928 Susans heirs to Maude as fee simple absolute 3/1928 Maude records 1933 Loan 1936 Maude to Ella 1955 1957 Deed 1 recorded 1972 Maude Died Subsequent grantees are held to have inquiry notice of the contents of prior recorded deeds. The 1928 deed, on which the s rely to establish their chain of title, expressly referenced the 1922 deed to Maude. Thus, Maude is bound to have taken the 1928 deed with knowledge of the 1922 deed. The recitals in the 1928 deed put any subsequent purchaser on notice of the earlier misplaced or lost deed. Thus, the 1928 deed is not entitled to priority, even though it was recorded years before the 1922 deed. Furthermore, the cannot rely on th e 1922 deed, because any interest they may have obtained under this deed would only be Maudes life estate, which terminated upon her death in 1972. The court concludes that it was incumbent on the to ascertain through diligent inquiry the contents of t he 1922 deed. The 1928 deed provides constructive notice of the 1922 deed. However, did not make any effort to inquire as to the interests conveyed in the 1922 deed when they purchased the property in 1955. Furthermore, the cannot claim title by adv erse possession from 1940 to 1955, because the adverse possession period would not have begun to run until Maudes death in 1972 , so they didnt have the right to take action until then. Subsequent grantees are held to inquiry notice of the contents of prior recorded deeds in the chain of title for purposes of a race-notice recording act Notes on Harper a. This case turns on the recording statutes and the importance of the reference to the 1922 deed in the 1928 deed case about inquiry notice s (paradise) should have been on inquiry notice to look up what was in the 1922 deed (record about prior deed) 1. Resembles Guillette in a sense that when a document refers to another document, the buyer needs to make an effort to find out what the referenced document said the burden of proof is on the subsequent purchaser to show that they made an effort i. Problem seems to put an enormous burden on the to do such a detailed search; want to put the burden on the cheapest cost-avoider (here, it should be the remaindermen) ii. By requiring inquiry notice, whats paradise supposed to find out? Just because there was a reference about 1922 deed, theyre supposed to find out all the details about who the people were, the conditions of the conveyance, track them down and meet with them to talk, etc? iii. Maybe this is a weaker than adverse possession holding b. Seems as though the adverse possession point in this case is decided wrongly

b.

c. d.

50

IX.

Adverse possession was designed to protect the occupant who has been there forever and that anyone with a claim should have come and kicked them off 2. The court says that the statute didnt start to run until the death of life tenant in 1972. But what if the court started the running period at the time of possession in 1936, remaindermen have cause of action to eject present living people. Court could give permission to occupy the land but you dont own the land. 3. Waldorff Insurance and Bonding, Inc. v. Eglin National Bank District Court of Appeal of FL, 1st District, 1984 (608) a. Choctaw Partnership developed certain properties in Okaloose County, FL by constructing condos. In June 1972, Choctaw executed a promissory note and mortgage on the properties. The $1.1 million note and mortgage was eventually assigned to Eglin National Bank () in January 1975, when less than $42,000 remained due on the note and mortgage. In April 1973, Waldorff () entered into a purchase agreement with Choctaw to buy condo unit 111. began occupancy of the unit immediately, continually occupying and maintained the unit until the date of the hearing. In October 1973, Choctaw executed another note and mortgage for $600,000 in favor of . Unit 111 was included in this mortgage. In June 1974, Choctaw executed yet another note and mortgage in favor of for $95,000, securing a number of units including 111. In March 1975, Choctaw, an insurance client of , agreed to consider the purchase price of 111 paid in full in exchange for a past due debt for insurance premiums owed by Choctaw to (cut it even). recorded. In 1976, brought a foreclosure action against Choctaw. A final judgment of foreclosure was entered in September 1976, but this judgment explicitly retained jurisdiction to determine the ownership of 111. Finally, in February 1983, a hearing on this issue was held. The trial court determined that s ownership interest was superior to s. The court held that did not have superior title, finding that s occupancy of 111 was equivocal because Choctaw allowed several other condo units to be used for free, and finding that did not pay adequate consideration for 111 so its quitclaim deed was void. The District Court of Appeal of FL reversed and remanded b. Actual possession gives constructive notice to all who claim an adverse ownership interest in a piece of property. In this situation, openly and exclusively occupied and possessed 111 at the time Choctaw executed the October 1973 and June 1974 mortgages to . Thus, is held to have had constructive notice of s adverse title, so cannot claim the benefit of the recording act. It is irrelevant that several other condo units were occupied by persons who possessed no legal or equitable title to the units. The units were intended to be conveyed as separate parcels, and thus the status of the other units is inconsequential. contends that it would have been difficult to ascertain whether actually had a claim of ownership of 111, but the court follows the holding in Phelan v. Brady (actual possession of real estate is sufficient to a person proposing to take a mortgage on the property, and to all the world, of the existence of a right which the person in possession is able to establish). In addition, the court holds that the trial court erred in finding that the conveyance from Choctaw to was void due to lack of consideration. Although may have erred in taking a bad debt tax deduction for the cancellation of his debt, the fact that Choctaw was relieved from payment of the debt constituted valuable consideration. All in all, only s 1972 mortgage lien is superior to s interest, and the satisfaction of this lien can co me from the 1976 foreclosure sale of the other condo units c. Actual possession gives constructive notice to the world of any right which the person in possession is able to establish d. Class discussion a. Waldorf, , took 111 after the bank gave out the first mortgage and recorded. If 111 was included in the first mortgage, Waldorf took with notice. So thats not what is arguing b. Is the bank subsequent bona fide purchaser? There is no record notice of Waldorf being the superior claim because Waldorf doesnt record until 1975. c. But, the problem is that Waldorf occupied 111 since 1973, so the issue of inquiry notice is at issue because Waldorf argues that Bank should have gone to the unit and searched the property before giving out the mortgage. Bank argues that it didnt inquire because there were other units that were occupied by people who didnt hold the deed. d. Is requiring the bank to inquire every time before giving out mortgage too burdensome to the bank or fair? Is this another bad case? Or is this justifiable? Waldorf could have provided the problem by being the cheapest cost avoider. SO why not put the burden on him? e. Waldorf probably couldnt file the contract of sale because to file a document with property interest, each signature has to be notorized. A lot of sellers would not acknowledge their signature on document in real life because when buyer defaults on payment, seller cannot take the recorded or filed document away. The seller now doesnt have any leverage to have the buyer take care of the problem because the buyer doesnt car e anymore, he defaulted. To keep the title clear, seller wouldnt sign the document that is required to file the contract of sale. Thus, Waldorf in this case is not the cheapest cost avoider. f. Look at page 611, Problem 1: You are the attorney for a prospective purchaser of a 50-unit apartment house. The present owner has shown the prospective purchaser copies of the leases on file in her office. Can the purchaser rely on these leases? Probably Not. Advise the purchaser to put a restriction that the seller must get estoppels certificate from every tenant that certifies that their lease is accurate and there is no other contract not recorded in the lease. Avoid notice question by doing this. e. Notes on Waldorff a. Bank argues it had no notice why not? had not yet recorded his interest Private Land Use Controls: The Law of Servitudes a. Intro B A Road b.

1.

Roadmap of the topic i. Easement 1. Express Easements 2. Implied easements

Question: B wants a path through As land to the road - Various things can be done but many of them is not enough to guarantee a long-term resolution - Easement travels with the estate in land to the subsequent purchaser so that the easement or equitable servitude between B and A travels to C when A transfers the estate to C 51

c.

d.

3. Easement by Estoppel 4. Easement by Prescription ii. Restrictions on Easement 1. Servitude 2. Covenant Servitudes Land use agreements arising out of private agreements i. Usually, but not always, the agreements involve 2 or more parcels of land, and the purpose of the agreements is to increase the total value of all the parcels involved ii. And usually, but not always, the effect of the agreement is to burden one parcel of land for the benefit of another parcel Easements i. Historical Background, and Some Terminology 1. Affirmative Easements gives a neighbor the right to enter or perform an act on the servient land 2. Negative Easements easements forbidding one land owner from doing something on his land that might harm a neighbor 3. Easements are also classified as appurtenant or in gross both types give easement owners the right to make some specific use (or in the case of a negative easement, restrict some particular use) of land that they do not own 4. Easement Appurtenant gives the right to whomever owns a parcel of land that the easement benefits a. Benefits the easement owner in the use of land belonging to that owner b. Requires both a dominant tenement (land B from above diagram)and a servient tenement (land A) c. Usually transferable a. The easement transfers along with the dominant tenement to successive owners b. Can be made personal to the easement owner only and not transferable to others d. When the classification of an easement is unclear, courts will usually go with easement appurtenant a. Why would this be the case? When theres an easement, the only way to terminate it is to get the benefited party to release the easement 1. When the easement is appurtenant, its easy to locate the party who is benefiting (because the easement would run with the land) but if its an easement in gross, it might be hard to locate the owner of the easement rights or the easement could have been assigned, etc., making an easement appurtenant much more efficient 5. Easement In Gross gives the right to some person without regard to ownership of the land a. Benefits the easement owner personally rather than in connection with use of land which that person owns b. Because it does not benefit any land, it involves no dominant estate, only a servient estate c. Can be alienable or inalienable ii. Creation of Easements 1. Being an interest in land, an easement is within the Statute of Frauds and creation of an easement generally requires a written instrument signed by the party to be bound thereby a. An easement may, under certain circumstances, be created by implication or by prescription 2. Willard v. First Church of Christ, Scientist Supreme Court of CA, 1972 (672) a. McGuigan owned 2 lots, numbered 19 and 20. These lots were located across the street from . McGuigan was a member of this church , and she allowed lot 20, which was vacant, to be used as a parking lot during church () services. She sold lot 19 to Petersen, who used the building on that lot for office space. Wishing to resell the lot, Petersen listed it with , a realtor. At the time, Petersen did not own lot 20. He did approach McGuigan with an offer to buy it, but she would only sell if the could continue to use it for parking space. She then had s attorney draw up a provision for the deed to the lot (20) stating that the change of ownership was subject to an easement for automobile parking during church hours such easement to run with the land only so long as the property for whose benefit the easement is given is used for church purposes. After this clause was inserted in the deed, McGuigan sold the property to Petersen, and he recorded the deed. paid Petersen the agreed purchase price and received Petersens deed 10 days later. recorded this deed, which did not mention an easement for parking by the church (). Apparently, Petersen did mention that would want to use lot 20 for parking, but did not tell of the easement clause in the deed from McGuigan. became aware of the clause several months later he bought the lot and then began an action to quiet title against the church. McGuigan testified that she had bought lot 20 to provide parking for . She also stated that she would not have sold it unless she was sure could use it for parking. While the trial court found that both McGuigan and Petersen intended to convey an easement for , the easement clause in the deed was invalid because a person cannot reserve an interest in property to a stranger to the title. The SC of CA reversed b. An owner who is granting property to a second person can reserve an easement in that property for a 3 rd person. The old common law rule stated that such a reservation of an interest was not possible. This rule was based on the feudal concept of reservation from a grant. IN that time, a grantor could pass his whole interest in property to a grantee, but a new interest was created in the grantor. Early common law courts opposed the possibility of vesting an interest in a 3rd party. While CA used to follow this rule very closelt, todays courts should not feel so restrained by feudal methods. The main objective should be carrying out the intent of the grantor. Thus, property grants are to be treated in the same way as contracts. Dealing with grants under the more rigid feudal approach would lead to unfair results. This is because the original grantee has likely paid a reduced price on the property in exchange for allowing a certain use of the property to continue. has not presented any evidence that or any others have relied on the common law rule when purchasing the property. Neither can claim he was prejudiced because the lot had not been used by the for an extended period of time. Indeed, the used lot 20 for parking throughout the period when was trying to buy it. Here, the interests of the grantors outweigh the interests the grantees may have if the old rule is followed. Looking at the clause as a whole, it is clear that McGuigan and Petersen intended to convey the easement for parking to c. A grantor can reserve an easement in property for a person other than the grantee 3. Notes on Willard a. When McGuigan transferred lot 20, it was fee simple absolute b. Willard had record notice: the condition was not on his deed but it was recorded on other deeds. He at least had inquiry notice. Willard had no argument that they were subsequent bona fide purchaser who should be protected because they didnt have notice.

Necessity Prior use

52

4.

5.

6.

There was old feudal rule and now court asks what was the reason of that rule? Court wants to look at the intention of the parties rather than just applying the old feudal rule. d. Did the churchs attorney make a mistake? The rule is that you have to use 2 pieces of paper for the transaction. By that, he didnt claim servitude. e. Difference between two kinds of easement. Assume that the Church moves to the next block and it sells the building to a Baptist church. The church still wanted to use the parking lot and Baptist church wanted to use the parking lot. Willard claims neither has right. a. Baptist church: Easement in appurtenant. The easement was to benefit this parcel because the provision in the deed specified the church on THAT property. Furthermore, easement runs with the parcel because the deed said that as long as it is used for church purpose the easement is transferred to subsequent purchaser. Thus, Baptist church wins. b. Original churchs argument: Easement was easement in gross. It was intended to benefit the particular church that requested the use of the lot for parking purposes. c. Willard: intended to benefit the church on that spot only. f. When there is easement and third party tries to get out from the burden of easement, ask these questions. Was there a notice? What kind of notice? If there was, the modern rule is to hold them liable. If theres some rule for ambiguity, easement in pertinent or easement in gross. Court does nt like easement in gross. Easement in appurtenant is easy to know who owns that land and when theres ambiguity, its much clear for courts to figure out. g. Is a subsequent bona fide purchaser of lot 20 without notice? a. No. is on inquiry notice because it looks like Petersen told the church used the lot for parking and because the lot was used for parking while was buying b. If had really read the deed, the easement is in the chain of title Licenses (677) a. A license is oral or written permission given by the occupant of land allowing the licensee to do some act that would otherwise be a trespass a. License is revocable (easement is not) 1. Exceptions to the rule that a license is revocable i. A license coupled with an interest cannot be revoked ii. A license can become irrevocable under the rules of estoppel Holbrook v. Taylor Supreme Court of Kentucky, 1976 (677) a. Holbrook () purchased the property in question in 1942. 2 years later, allows a mining road to be sit on the property. The road was used for that purpose until 1949, and received a royalty for use of the road during that period. In 1965, the Taylors () bought a 3-acre building site next to the s property. built a house on that site the following year. During the preparation for and the actual building of the house, the were permitted to use the roadway for access for workmen, for transporting equipment and materials, and for the construction and improvement of the house. s home was completed at a cost of $25,000. After the house was complete, s continued to use the road on a regular basis. At trial, testified that he allowed to use and repair the road so could reach their home. This was the only location where a road could be created to provide an access route. widened the road and graveled part of it at a cost of roughly $100. and had no disagreement over the use of the road until 1970. Then, according to , he wanted to give a writing which would relieve of any responsibility in case someone were injured or oth erwise damaged on the old mining road. testified that the writing was an attempt to make buy the land the road was situated on for $500. The disagreement continued, and later raised a steel cable across the road to block passage, and set up no trespassing signs. filed suit to remove the obstructions and to declare s right to use the road without interference. The lower court held that s use of the road was established by estoppel. The SC of TX affirmed b. An easement can be established by estoppel, even when the person making use of the property does not do so adversely, but with the permission of the property owner. It has long been recognized that a right to the use of a roadway over another persons land may be established by estoppel. T he established rule in this state was set forth by this court in Lashley. As stated in that case, a licensor may not revoke a license which includes the right to erect structures and acquire an interest in the land similar to an easement after the licensee has exercised the privilege of the license and erected improvements on the land at considerable expense. Here, it is clear that used the road with the consent, or at least the tacit approval, of . Moreover, used the road to build and improve their $25,000 home, and widened and maintained the road at their own additional expense. s license to use the road may not be revoked and s right to use the road has been established by estoppel c. A license cannot be revoked later the licensee has erected improvements on the land at considerable expense while relying on the license Notes on Holbrook a. Bilateral monopoly problem an investor counts on having the land and makes improvements on the land based on reliance ( built a house because he relied on the fact that hed have access) maybe its unfair to allow the person granting the easement to ask for money after the other party has relied heavy reliance on a license makes the license irrevocable a. Once theres an easement by estoppel, it should be recorded 1. If not, problems will be created for the next purchaser, who will likely have to go through the process of getting permission/relying/etc. all over again to ensure that the next purchaser has the right to the easement b. Holbrook claims permissive easement and Taylor claims prescriptive easement. Court said it cant be prescriptive because there was disruption and later Holbrooks predecessors gave permission and collected royalty and thats permissive easement. So, if it doesnt make sense to find prescriptive easement..?? c. Would the court probably have found for estoppels? IS it fair to Holbrook who was nice enough to give permission? Courts saying that by implication Holbrook knows better. Once Holbrook saw all these trucks and house, he should have said something if you didnt want them to use it any more. You cant do it now. It was an oral agreement that gives property interest because someone was nice enough and was being neighborly. The third party who purchases a land with a path, if the easement is not in writing, has no way of knowing. There is no notice. d. Court tries to make a rule that says that lets minimize litigation by only recognizing easement recorded on the deed. Oral argument only would not be recognized. Isnt easement by estoppels creates more litigation? If someones gonna get hurt, lets have a rule that facilitates everybodys knowledge of the interest in property. First, put the burden on Taylor because c.

53

7.

he knows he needs the road, so he should get the easement and record. But in neighborhood or friendship, its an insult to ask to put it in writing. That undermines the reality, and law should support neighborly arrangement. So whats good? Have it in writing or oral is enough? e. Holbrook had to prove what to justify the easement by necessity? At the time when the land lock occurred and there was a common owner of both land who separated the lot, thats when the easement by necessity can be rasied. Van Sandt v. Royster Supreme Court of Kansas, 1938 (682) a. In 1904, Bailey owned a play of land directly south of 10th Street and east of Highland Ave. The plot was divided into 3 lots numbered, from east to west, as lots 19, 20 and 4. Baileys home was on lot 4, the eastern part of her land. Early that yea r, the city of Chanute built a public sewer on Highland Ave. At roughly the same time, a private lateral drain was built, running from the Bailey home on lot 4, across lots 20 and 19, and to the public sewer. Also that year, Bailey conveyed lot 19 to Jones by a general warranty deed with not exceptions or reservations, and conveyed lot 20 to Murphy by similar deed. Each one built a home for himself on his respective lot. The title to lot 20 eventually passed to . Gray (also a ) had succeeded to the title to lot 4 by the time Bailey sold lots 19 and 20. By 1924, lot 19 had been conveyed to , who continued to own and occupy the premises there. In March 1936, discovered his basement had been flooded with several inches of sewage. Upon investigation, discovered that the sewer drain which extended across the property of and of Gray. This drain pipe has several feet under the surface of the ground. There was nothing visible on the ground behind the houses to indicate the existence of the drain or its link to the houses. brought action to stop from using an d maintaining the drain. Judgment was rended in favor of and appealed. claimed that no easement was ever created in s land and that even if one was created, s property could not be burdened with it because had no notice. argues that an easement was created by implied reservation when lot 19 was severed from Baileys lot as a result of the sale to Jones. The SC of Kansas affirmed b. The implication of an easement will depend on the circumstances under which the conveyance of land was made, including the extent to which the manner of prior use was or might have been known by the parties: each party will be assumed to know about reasonably necessary uses which are apparent upon reasonably prudent investigation. An easement is an interest which a person has in another persons land. While this means an owner cannot have an easement in his own land, an owner can nonetheless use one part of his land to the benefit of another part of the land. This arrangement is generally known as a quasi-easement. Accordingly, the part of the land that benefits from this use is a quasi-dominant tenement, while the part of the land burdened with the particular use is a quasi-servient tenement. Early cases held that when the owner of the overally property transferred the quasi-servient tenement to a new owner, an implied reservation of an easement was made in favor of the conveyor of the property. Here, this factor, that the grantor of property is the one who claims the easement, is but one of many to consider in determining whether an easement can be implied. An implied easement arises as an inference of the intentions of the parties to a conveyance of land, and such an inference is to be drawn from the circumstances, not the language, of the conveyance. Under the restatement, those circumstances should include the extent to which the manner of prior use was or might have been known to the parties. Thus, the parties will be assumed to know and to contemplate the continuance of reasonably necessary uses which would be apparent upon reasonably prudent investigation. When Jones bought lot 19, he was aware of the lateral sewer drain, and knew that it was built for the benefit of Bailey. The easement for the drain was necessary for the comfortable enjoyment of her property, and an easement can be implied on the basis of necessity alone. Moreover, cannot claim that he had no notice when bought the property. made a careful inspection of the property, and knew that the house had modern plumbing and that the plumbing had to drain into a sewer. Appearance and visibility are not synonymous, and that the pipe may be hidden underground does not negative its character as an apparent condition; at least, where the appliances connected with and leading to it are obvious c. The implication of an easement will depend on the circumstances under which the conveyance of land was made, including the extent to which the manner of prior use was or might have been known by the parties; each party will be assumed to know about reasonably necessary uses which are apparent upon reasonably prudent investigation; an easement may be implied for a grantor or grantee on the basis of necessity alone d. Notes on Van Sandt a. Why does this easement run with the land? 1. Necessity 2. Prior use i. When Bailey owns all the land, the sewer is built and its present w hen lots 19 and 20 are purchased; 19 and 20 would know of the existence and expect the use to continue b. There are two arguments. First, no easement. Second, subsequent BFP. c. First, no easement? 1. Could have asked to put it in the writing 2. But, notwithstanding the failure to get it in writing, the court found that there was an implication that parties agreed to continue this easement although parties did not write them down 3. Why? Jones knew the sewer was there. More than that, there was quasi easement meaning that the use was there before severance. If parites knew (knowledge) that there was use and parties took the property as it is, then it meant they accepted implied easement + Necessity: its not the same with easement by necessity based on the public policy. This is based on imputing intent that it was necessary for the comfortable enjoyment of the property. Its true that parties failed to get it in writing, but court is trying to facilitate agreement between parties. Some jurisdictions like TX or NY do not recognize easement by necessity unless there is a strict necessity. So, if parties were in such jurisdiction, then what would the party in lot 4 have to do? Now court might views sewage as absolute necessity. Or they can bargain with lot 19, but the owner of lot 19 is in a much better bargaining power position because he knows there is no easement so he might as for something unreasonable. Or they can petition the government to solve this by increasing the sewer line. Court is looking at what will happen if the court finds no easement terrible cost for lots 4 and 20 and for the city would need to reconnect plumbing to the main sewer line, etc. If there was another sewerline close by, then the plaintiff probably has weaker case because there is another access of ingress and exgress. d. Second, subsequent BFP ?

54

1. 2.

8.

9.

3. Implied Easements a. Easements are implied in 12 basic situations a. Easement is implied on the basis of an apparent and continuous (or permanent) use of a portion of the tract existing when the tract is divided 1. Implied to protect the probably expectations of the grantor and grantee that the existing use will continue b. Easement is implied when the court finds the claimed easement is necessary to the enjoyment of the claimants clan and that the necessity arose when the claimed dominant parcel was severed from the claimed servient parcel 1. NY follows the old rule that strict necessity is required for implied easements in favor of the grantor 2. However, the weight of authority today holds that only reasonable necessity is required for an implied easement, regardless of whether the easement is implied in favor of the grantor or the grantee b. If the dominant tenement and the servient tenement come into the same ownership, the easement is extinguished altogether a. The easement will not be revived by a severance of the united title into the former dominant and servient tenements b. When the united title is subsequently redivided, a new easement by implication can arise if the circumstances at that time indicate a new easement was intended Othen v. Rosier Supreme Court of Texas, 1950 (689) a. Hill once owned the entire plot. On August 26, 1896, Hill conveyed a 100-acre tract just west of the public highway that was eventually conveyed to in 1924. In 1897, Hill conveyed a 60 -acre tract just southeast of, and contiguous to s 100acre tract. acquired this tract in 1904. By this point, Hill owned a 53 -acre tract just east of the 100-acre tract and a 16.31 acre tract just west of the 60-acre tract. On January 26, 1899, Hill conveyed the 53 and 16.31-acre tracts to separate buyers. These buyers later conveyed the larger tract to in 1913 and the smaller tract to in 1924 (now owns 60+53, owns 100+16.31). s acres are not contiguous with the highway. Before this case, would r each the highway by going through a gate on the west line of his 60-acre tract and the east line of s 16.31-acre tract, then into a fenced lane which runs along the south side of s 100-acres. This lane went to a gate which opened onto the highway. The south fence was built in 1895, while the north fence and outside gate were built in 1906. Near this exit gate was the s home, orchard, and barns. used it to haul wood and to permit their livestock to go to the pasture on the 16.31 -acre tract. s tenants on the smaller tract have also used the roadway in the way has. made all necessary repairs to the land, and no one else recognized any obligation to maintain it. Surface waters threatened to make the road impassable and erode the s farmland. TO prevent this, built a 300-ft. long levee along the southern fence of the road. This levee made the lane so muddy that the land was impassable, except by horseback, for several weeks at a time. filed suit claiming this act of deprived of access between the highway and his home. wanted a temporary writ of injunction to keep from maintaining the levee and a mandatory writ of injunction to keep from interfering with s use of the roadway. The trial court found that had an easement by necessity along the roadway, and ordered to ensure that it would be in usable condition. The Court of Civil Appeals initially affirmed the judgment but reversed the injunction because it was too vague. On rehearing, the appellate court concluded that has no easement either by necessity or prescription. The SC of TX affirmed b. An easement can be created by implied reservation only when it is shown that there was unity of ownership between the alleged dominant and servient estates, that the easement is a necessity and not a convenience, and that the necessity existed at the time the 2 estates were severed. As previously stated, the entire plot had been owned by Hill. Unity of ownership is satisfied. There is no evidence, however, that the roadway was a necessity when Hill conveyed the 100-acre tract on August 26, 1896 and still retained the 60-acre tract which now owns. At the time, it appears the roadway was merely a convenience. Hill may easily have been able to cross the 53-acre tract and go around the 100-acre tract, or be able to go around the small 16.31-acre tract, to get to the highway. Obviously no such easement could be created over his own land. The mere fact that s land is completely surrounded by other peoples land does not automatically mean that has a way of necessity over other property. No easement by necessity can be implied here. Moreover, an easement by prescription can only be acquired if the use of the easement was adverse. If one owners property is put to use by another by virtue of permission or license, then the non-owner cannot claim he has a separate right to use the property. Here, the roadway has been fenced on both north and south sides since 1906. The and their tenants have used it for general farm purposes as well as to haul wood and more livestock. In light of those facts, it appears that s use of the roadway has been merely with the permission of . Thus, s use could not develop into a prescriptive right. s evidence of his use of the roadway prior to 1906, when the fences and gate were completed is also too vague to establish a prescriptive right to the road c. An easement can be created by implied reservation only when it is shown that there was unity of ownership between the alleged dominant and servient estates, that the easement is a necessity and not a convenience, and that the necessity existed at the time the 2 estates were severed by establishing that there was no other way; an easement by prescription can only be acquired if the use of the easement was adverse d. Notes on Othen a. Othens argument is that it was easement by implication. If landlock parcels is bad, why not just grant easement by necessity? For O to be able to establish necessity existed at the time of the severance of 2 estaets, he has to say that at the time of severance although it was Hill do severed the parcels, there was no other way to get to the road other than the way through Rs land. But the problem is O cant prove that due to bad lawyering or lost of record or whatever. b. According to the court, being landlocked isnt enough to create an easement by necessity 1. Its necessary to show that at the time the necessity arose, there was an easement i. At the time Hill broke up the land, Hill still owns enough of the land not to require an easement (cant have an easement across his own land)

Was there a notice? How? If you have a toilet, then you have inquiry notice? Thats why the court said. Is it right to charge him with notice then? Isnt it too much to ask them how all the toilets in the neighborhood were constructed to have sewage line? Pretty tough to determine where the sewer lines are and if every purchaser had to search for sewer lines, its a daunting task s best argument is that he is a subsequent bona fide purchaser with no notice

55

iii.

Easement by Prescription (this is like AP) Court says not possible because s use wasnt adverse, it coincided with s use of the same land 2 possible times could try and show adverse possession 1. O has been using the pathway through Rs land for years to cultivate his 100 -acre farm land and nobody ever told him he cant. The court says that it was permissive use because there was a gate and O is a trespasser going through the gate and the land. 2. Pre-1906 not possible to prove he was consistently using the same land and couldnt establish how he accessed his land no clearly drawn lines and no definite path i. Not sure why it has to be a clearly defined path, although if theres no defined path, theres no notice to the landowner and the adverse possessor would have to consistently assert the same right to put the owner on notice 3. Post-1906 definitely used for sufficient amount of time, but not used by exclusively i. Exclusivity doesnt seem to make sense when all a party is asking for is a right -of-way. wouldnt care if others could use the property too as long as he could use it ii. Courts argument that the use was permissive shouldnt be a revocable license it leaves with absolutely worthless farmland. 4. The reasoning of the court basically shows that there is no such thing as a prescriptive easement. Thus, courts reasoning here is problematic. 5. would prevail in a Holbrook jurisdiction where the court expressly states its understanding that acquiescence is required but it doesnt defeat the prescriptive easement claim no exclusivity necessary i. What would defeat s claim in this jurisdiction would be an express grant of permission 6. Argument of estoppels can be used here too. Was permission, then prescriptive argument. No permission then use estoppels argument But Texas rejected estoppels so couldnt make that argument here. 10. Easements by Necessity a. Argument s you would make if you purchased a land that has no access other than trespassing another land. 1. Dont want to create land lock parcels that are useless because its inefficient & 2. Reasonably have contemplated that buyer would have an access to his land by crossing the sellers land b. Policy perspective courts are hesitant to render landlocked property valueless because landlocked parcels will be unmarketable c. Where a party sells part of his land and the sold portion is landlocked a. Implied understanding between parties from someone who buys a landlocked parcel that the person selling would be accessible through the sellers property d. Most courts, like the one in Othen, require strict necessity a. Some courts, however, have granted an easement by necessity where access to the land exists but it is claimed to be inadequate, difficult, or costly 1. In a few jurisdictions a surface way of necessity will not be implied if the tract has access by navigable water b. An easement by necessity endires only so long as it is necessary 1. If the dominant owner secures another way out from the landlocked parcel, the easement by necessity ceases 11. Easements by Prescription a. Adverse Possession versus Easements by Prescription a. Adverse possession involves a statute of limitations running on the right to bring an action to recover possession of land; the statute operates to extinguish the remedy of the previous owner, leavint the adverse possession in indefeasible possession 1. The result is that the adverse possessor has a new title based on his possession b. A statute of limitations upon the recovery of possession does not cover actions concerning easements, which involve use and not possession of land c. Reasons underlying the protection of long-continued adverse possession apply to long-continued use 1. Legislatures enacted no legislation to protect ancient easements, courts developed the doctrine of prescription, which rests upon the idea that rights can be acquired simply by passage of time b. Evolution of the Prescriptive Easement and the Fiction of the Lost Grant a. Earliest type of prescriptive easement was the easement based upon a use from time immemorial 1. In 1275, Parliament enacted a statute prohibiting challenges to rights of possession enjoyed since the accession to the throne of Richard I (1189) 2. As time progressed, it became difficult to prove that the claiomant and his predecessors had enjoyed the use since 1189 i. The first rule set down by judges was if a use had existed so long as any living person could remember, it was presumed to have existed from 1189 ii. Later, by analogy to a statute, the judges held that, if a use had continued for 20 years, it presumptively had existed since 1189 b. Judges invented the fiction of the lost grant 1. If a use was shown to have existed for 20 years, it was presumed that a grant of an easement had been made and that the grant had been lost. The presumption of a grant could not be rebutted by evidence that no grant had in fact been made 2. Once prescriptive easements were put on the same basis as adverse possession, the lost grant theory did not become irrelevant i. Under a lost grant theory, the owner of land is presumed to consent of acquiesce in the use on the other hand, if the use is made with the permission of the owner, the use is not adverse. To secure a prescriptive easement under lost grant theory, the claimant must show that the use was not permissive and that the owner acquiesced 3. In a jurisdiction not following the fiction of the lost grant, to prevent a prescriptive easement from being acquired, the owner must effectively interrupt or stop the adverse use Summarizing the law of unwritten easements c.

56

iv.

v. vi.

Easement by Express Grant - Willard a. Binding on all successors to the benefited and burdened parcels and runs with the land (or the person, if in gross) big issue is whether subsequent purchasers were on notice a. Notice will be the same kinds as in recording acts record notice, inquiry notice, actual notice b. If a person cant prove the benefit of an express easement, move to the next theories 2. Easement by Estoppel - Holbrook a. Must have permission to use or a license which is presumed revocable unless there is substantial reliance on the permission 3. Prescriptive Easement Othen (no prescriptive easement) a. Adverse use must not be permissive (although theres ambiguity as to what constitutes permission standing by and watching or is express permission necessary) b. Open and notorious (owner must be on notice of the possession c. Possession must be exclusive (open question depends on the jurisdiction Othen jurisdiction says exclusivity is imperative) d. Possession is continuous for the statutory period 4. Implied Easements Van Sandt a. Implied by Necessity a. Property must have initially been under a unity of ownership b. Upon severance, the necessity for the easement must have arisen b. Implied by Prior Existing Use a. Property must have initially been under a unity of ownership b. Use of the easement must have existed before the severance (quasi easement) c. Evidence that the parties reasonably anticipated the use would continue 1. Factors listed on p. 686 when looking at possible intent 2. Necessity is one of these elements if the easement is necessary, the more likely it is that prior parties knew about the easement and thought it would continue d. Notice (of any kind record, inquiry, actual) to subsequent purchasers Martha v. Trump class hypo 1. Is there an express easement? No. 2. Implied easement by necessity- its weak argument for Trump because beach access is not really a necessary. He might say that hes buying a beach house with no access to the beach? We need to know the purchasing price of the house because if he real ly paid for beach house amount or no access to the beach was reflected on the price and he paid less? 3. Implied easement by prior existing use- Trump has to establish first, there was quasi-easement because the use was there before severance. The use was there when Oprah owned both properties. Next, parties intended to use after the time of the severance? First figure out who the parties were. At this time were not concerned with Martha. Trump argues that the parties intended to continue the use, although its not strictly necessary, it was necessary to enjoy the property. Third, Subsequent purchaser, Martha, h ad notice. Martha argues that 1. No easement by prior use, 2. Even if there was easement, no notice so shes subsequent BFP. First, it was not in writing and Oprah did not allow easement but she was being just friendly and allowed Donald to pass by her property to get to the beach. Although it is not an element for easement by prior existing use, the fact that the use was not continuous might provide evidence against parties intended to use. Another thing is that Oprah knew how to include easement because she did one for herself, but she didnt give easement to Trump so the fact that she didnt give easement meant that she didnt mean to give. Second, even if there was easement, Martha had no notice because the walkway was deteriorating and didnt think someone would use it. In VanSant, the sewage was buried down and didnt look like it was abandoned or not being used anymore. Here you can see it and because of the condition of the walkway, she didnt have inquiry notice because she assumed nobody would use something like t hat. Trump counters by saying that In Vansant court found easement for sewage that is not even visible by charging inquiry notice, here, its an easier case because its visible, so the court should charge inquiry notice. 4. Prescriptive easement We have Othen and Holbrook which are opposite. So we have to deal with both cases because its not specified which jurisdiction we are in. Start with Holbrook (continuity, etc.). Trump will argue that it was continuous (AP case where court found summer usage of summerhouse to be continuity, Howard v. Kunto) because the ordinary usage of the property is to be used as a beach house. Martha counters that its not continuous . Also, in Kunto, the evidence of usage was always there because house was always there. But here, Martha cant catch Trump all the time he crosses because shes not there all the time. In Othen, exclusivity and knowledge. For exclusivity, nobody else but Donald use the walkway, so he argues that he has exclusivity. Even in Texas, no problem. Martha also says she acknowledged the usage so there is permissive use so cant make AP. 5. Easement by Estoppel Trump say even if it was permissive use and if it was not an easement by prior use, I should get easement by Estoppel. First, must establish permission. Then, if there was permission, based on that permission, he had detrimental reliance. Donald had permission from Oprah because she didnt object to his walkway use. Did he rely on the permission? Harder to estab lish. Problem here is that the permission must be at the time of the purchase. Donald gets his permission after the purchase of the property. So both elements are difficult to establish and weak, so dont spend much time writing about this if this was on th e exam. Read about termination of Easement on your own Scope of Easements 1. Brown v. Voss Supreme Court of Washington, 1986 (716) a. In 1952, the then-owners of parcel A granted a private road easement across their property to the owners of Parcel B for ingress and egress from parcel. acquired parcel A in 1973. bo ught parcel B from one owner on April 1, 1977, and then parcel C from another owner on July 31, 1977. The previous owners of parcel C were not parties to the easement grant. planned to build a single home that would straddle the boundary line between parcels B and C. began clearing both parcels in November 1977. began trying to sop from using the easement in April 1979, by which time the had spent over $11,000 in developing the properties. At this point, placed logs, a concrete pit, and a c hain link fence within the easement. sued for removal of the obstructions, an injunction against further interference with the use of the easement and damages. counterclaimed for damages and an injunction against using the easement to get to parcel C. The trial court awarded each pary $1 in damages, with the award against being for a minor inadvertent trespass. The trial court also found that made no unreasonable use of the easement while developing their property, not had acted unreasonably in making these developments. Also, if s injunction were granted, parcel C would be landlocked and would not be able to make use of their property. The trial court found that would suffer no appreciable hardship or damage and their requested injunction was denied. Moreover, the trial court held that framing and enforcing such an injunction would be impractical. Based on these and other findings of fact, was denied his injunction and was allowed

1.

57

2.

to use the easement as long as their properties were used solely for single-family residence. The Court of Appeals reversed and the SC of WA affirmed the decision of the trial court b. The easement in this case resulted from an express grant made in 1952. Thus, the scope of this right acquired through the easement is to be determined from the terms of the grant and the way they give effect to the intention of the parties. By the express terms of the 1952 grant, the previous owners of parcel B acquired a private easement across parcel A and the right to use it to enter and exit parcel B. While acquired these same rights to enter and exit parcel B, has no such easement rights with regard to parcel V. Parcel C was not part of the original dominant estate under the terms of the 1952 grant. If an easement benefits its owner in the use of a particular parcel of land, any extension of the easement to other parcels is a misuse of the easement. Even though, as contends, s use of the easement to gain access to a home located partially on parcel B and partially on parcel C is at most a mere technical misuse of the easement, it is nonetheless a misuse. This doesnt necessarily mean is entitled to injunctive relief. As the proceeding for determining the validity of an injunction is an equitable one, deference should be paid to the findings of the trial court unless an abuse of discretion is shown. No such abuse is demonstrated in this case, as the trial court is vested with a broad discretionary power to shape and fashion injunctive relief to fit the particular facts, circumstances, and equities of the case before it c. If an easement benefits its owner in the use of a particular parcel of land, any extension of the easement to other parcels is a misuse of the easement d. While the extension of this easement to nondominant property did constitute a misues of the easement, should be entitled to injunctive relief. Misuse of an easement is a trespass. Consequently, s continued misuse of the easement in building and residing in the home they proposed would result in a continuing trespass. Damages on such a prolonged trespass would be too difficult to measure; thus, injunctive relief would be the appropriate remedy under such circumstances. should have known from public records that the easement was not conn ected to parcel C If an injunction were granted for the , could still acquire access to parcel C through other means, either by renegotiating the easement or some other statutory means injunctions are appropriate remedy when remedy is difficult to give e. Issue: There is express easement for the benefit of Parcel B. But he wants to use the road to benefit C. Is it a misusage of the easement? Yes the court agrees, so won on law but the court didnt grant injunction so he couldnt do anything! f. Notes on Brown a. Although the court agrees with s legal argument, it still affirms the decision of the trial court, denying s request for injunction. Why didnt trial court grant injunction? 1. The court balances the equities (mushy) and the factors that cause the court to deny the injunction 2. s acted reasonably would still be a single-family home and the easement would not have an increased burden (no burden on ) 3. s allowed s to accrue costs in development (allowing reliance) - Voss should have stopped this (Estoppel) 4. No irreparable harm done to Voss 5. s land is landlocked (according to the facts given to the court) 6. s counterclaim looks like its made in bad faith (bad motive) looks like should be punished for making the counterclaim b. Parcel A isnt hurt at all. And the balancing of equities is so mushy. Court characterized Voss as someone whos just trying to be a jerk. Its a complete mischaracterization of Voss. Voss was just upset because his neighbor was not being neighborly and was getting angry about that and acted out right away to stop Brown from using part of the land that he doesnt have easement benefit. c. (Voss)s attorney really screwed up and it had affected directly to the denial of injunction that Voss asked for 1. Parcel C wasnt actually landlocked and the wrong maps were submitted to the court, and court adopted Browns presentation w/o questioning- this led to reason of s parcel being landlocked (#5 above) 2. Attorney should have painted a better picture of the relationship between the parties- this led to the loss based on estoppels (#6 above) 3. Should have made prove where the house would be located (looked like it would be 100% on parcel C) 4. didnt actually let reliance build up because he did speak up from the start, its just that the keeps ignoring him this is different from #3 above 5. Counseling Voss to file a counterclaim wasnt a bad lawyering. But because Brown tells the story and Voss didnt let his clients story be told, thats bad lawyering. Three basis for denial of inju nction could have been avoided if Voss lawyer didnt screw up. d. Argue that granting injunction would have been appropriate 1. Browns was in the best position to have anticipated and dealt with this problem before. Brown was in notice of the easement that extends only to Parcel B. This was an expressed easement recorded. So when they bought the parcel, they should have known through the recording that there is limit to easement- constructive notice. They should have brought a consultant, or negotiating with Voss, or make a different but more expensive road, etc. 2. Injunction is given when granting damages is not practical. 3. Even with grant of the injunction, Brown is not out of other recourse. Brown can buy out another easement from Voss or buy out the injunction. e. Suppose Brown wanted to replace his place with a 8-room inn. Would Voss argument weaker or stronger? Stronger probably because then Parcel A that Voss owns might be hurt by ongoing customers who would have to use or would be permitted to use his land to travel to Parcel B or C. The easement was not to benefit this way either. Brown could look for another way to make a road to make his inn more accessible. After all, parcel C is not landlocked. Counter would be that as long as the intention of the easement, its not clear but it only says engress and egress and using the easement is not out of the scope of the easement. Counter of Voss can be that the original easement was given to a single house family so it goes under the intent of the easement to be used only for such purpose. Counter of Brown to that would be the development was reasonably foreseeable to original parties and in such case, the added burden doesnt stand as strong. Then better recourse is to bargain out the injunction or bargain a new easement and try to work it out rather than litigating. Notes (721)

58

e.

The manner, frequency, and intensity of the use may change over time to take advantage of developments in technology and to accommodate normal development of the dominant estate or enterprise benefited by the servitude a. Unless authorized by the terms of the servitude, the holder is not entitled to cause unreasonable damage to the servient estate or interfere unreasonably with its enjoyment b. A private easement of way does not usually permit the easement owner to install on the easement aboveground or underground utilities, such as electrical lines and sewer pipes. Most courts hold such uses are not reasonably foreseeable by the parties c. The established rule is that the location of an easement, once fixed by the parties, cannot be changed by the servient owner without permission from the dominant owner a. Restatement of Property 4.8, comment f changes this rule it grants the servient owner the right to change the location of an easement, at his expense, if the change does not significantly lessen the utility of the easement, increase the burdens on the owner of the easement in its use and enjoyment, or frustrate the purpose for which the easement was created d. A prescriptive easement is not as broad in scope as an easement created by grant, by implication, or by necessity a. Although the uses of a prescriptive easement are not confined to the actual uses made during the prescriptie period, the uses made of a prescriptive easement must be consistent with the general kind of use by whith the easement was created and with what the servient owner might reasonably expect to lost by failing to interrupt the adverse use vii. Termination of Easements (735)- read on your own and get a loose grip of what these are 1. Easements can be terminated a number of ways a. Release the owners of the servient and dominant tenements can draft a document and as owner of the servient tenement, its wise to record immediately b. Expiration where the duration of an easement has been limited and comes to an end because easements are interests in property and therefore subject to the Statute of Frauds, this is normally in writing a. Similarly, an easement created to end upon the occurrence of some event (defeasible easement) expires automatically if and when the stated event occurs 1. For example, an easement by necessity ends when the necessity ends 2. Another example - Willard c. Merger if the easement owner later becomes the owner of the servient estate d. Estoppel if the servient owner reasonably relies upon a statementor representation by the easement owner e. Abandonment normally, mere non-use by the easement owner does not constitute abandonment, but in several states a prescriptive easement ends by abandonment upon non-use for the statutory period of time f. Condemnation if the government exercises its eminent domain power to take title to a fee interest in the servient estate for a purpose that is inconsistent with continued existence of the easement g. Prescription if the servient owner wrongfully and physically prevents the easement from being used for the prescriptive period Covenants Running with the Land i. Class notes: why dont we just categorize all private agreement as easement? Its all historical. Because in old England there was no re cording system and thus easement was not usually visible and had a way to notice could not have been always known to a subsequent purchaser. So, courts limited negative easement strictly. However, against the will of the court not to recognize or encourage negative easement of individuals, people want to get their way, i.e., I want residential neighborhood so enter easement with neighborhood not to build a gas station ii. Covenants Enforceable at Law: Real Covenants Historical Background 1. If a restriction can be enforced at law and the remedy is damages and then, its a real covenant a. Most people just want injunctions and theres very litt le case law about enforcing a restriction at law b. If you get injunctions and asks for equitable relief, then thats equitable servitude. 2. Judicial recognition of a contract right respecting land use enforceable not only against the promisor landowner, but against his successors in title as well a. Property right enforceable by and against subsequent purchaser 3. In the early 19th century, contract rights and duties were not generally assignable a. Promises were not enforceable against a person who was not a party to the contract b. The law had developed one exception to the rule of nonassignability where there is privity of estate, the judges held, the contract is enforceable by and against assignees a. Unlike the English courts, US courts did not define privity of estate to include only landlord-tenant relationships 1. Permitted, under varying circumstances, covenants to run in favor of and against successor owners i. Developed the real covenant a promise respecting the use of land that runs with the land at law 4. The question whether a covenant runs arises only when a person who is not a party to the covenant is suing or being sued a. If the original promisor or promisee is a party, he has the burden of showing whether the burden or benefit runs to the successor of the other original party b. If both parties to the action are successors, the burden and the benefit must run a. Important to keep win mind whether the running of the benefit or the running of the burden is involved in the case because the test for the running of the burden is involved in the case because the test for running of the burden is traditionally more onerous than the test for running of the benefit 5. Privity a. The First Restatement of Property declared that horizontal privity (privity of estate between the original covenanting parties) is required for the burden of a covenant to run at law, but not required for the benefit to run a. All commentators have since rejected the 1st Restatements requirement of horizontal privity some on the ground that the requirement has no case support, others on the ground that, case support or not, the requirement is unsound as a matter of policy b. The Third Restatement of Property repudiates the 1st Restatement and takes the position that horizontal privity of estate is not required for a covenant to run at law to successors a. Nonetheless, the requirement of privity appears to persist in a number of states, if only in dicta of a court b. Section 5.2 discards the vertical privity requirement for both the burden and the benefit a.

59

iii.

Rather, as to both real covenants and equitable servitudes it distinguishes between negagive promises and affirmative ones 1. Negative Promises (a promise not to do an act) treated the same as easements, meaning that all owners and possessors of burdened land are bound by negative covenants regardless of the extent of their interest or the manner in which they ovtained their interest. Likewise, all owners and possessors of benefited land are entitled to enforce the covenant 2. Affirmative Covenants (a promise to do an act) sets out separate rules for lessees, life tenants, and adverse possessors, and it distinguishes between benefits and burdens i. As to lessees, the benefit of covenants to repair, maintain, or render services to the property run to lessees. So also do benefits that the lessee may enjoy without diminishing the benefits value to the lessor and without materially increasing the burden of performance on the person obligated to perform the covenant. On the burden side, the only affirmative covenants that bind lessees are those that can more reasonably be performed by a person in possession than by the holder of the reversion ii. Both the benefits and burdens of affirmative covenants run to life tenants, however, the life tenants liability for performance of an affirma tive covenant is limited to the value of the life estate iii. Adverse possessors who have not yet gained title are liabile on the affirmative covenants burdening the property, but the benefits of affirmative covenants run to adverse possessors who have not yet gained title to the property only under limited circumstances. Those circumstances are that the covenant was to repair, maintain, or render services to the property, or that the benefit is one that can be enjoyed by the person in possession without diminishing the benefits value to the owner of the property and without materially increasing the burden of performance on the party obligated to perform the covenant. All of the appurtenant benefits and burdens of servitudes burdening the land when adverse possession began run to adverse possessors who have acquired title 6. A covenant is not enforceable against an assignee who has no notice of it Covenants Enforceable in Equity: Equitable Servitudes 1. Hypo: Blackacre is owned by A originally. Then A splits the land and gives half of it to B. A and B enters into an easement not to build anything other than residential building. A sells his parcel to D. Then B starts to building a gas station. Does D have a right to damages? Equitable relief? a. First, D has right to damages? Figure out whether D can meet the requirement for real covenant. He has to establish vertical privity (between A and D) for benefit. If B sells to C, then D needs to show 1. Vertical privity that the benefit ran from A to D. 2. Horizontal privity that the restriction ran from A to B and B to C. If A didnt own the whole land in the beginning and A and B just entered into an agreement, the burden/restriction would not have run with the land because horizontal privity has not met because it was just a contract and no property right has been transferred. So as soon as B sells to someone, that horizontal agreement/privity has been broken. But until B sells to someone, the contract can be enforced. [confusing] b. Leslie does not believe that modern court would hold a subsequent purchase without notice to restriction. We have a recording system. So, if A and B enter into an easement and recorded, then the subsequent purchaser would be held to the restrictions because of notice. And most people ask for injunction not for damages. They dont want money but just want the other party to stop what they are doing or plan to do. 2. If a restriction can be enforced in equity, the remedy is injunction and its an equitable servitude 3. Tulk v. Moxhay Court of Chancery, England, 1848 [law or real covenant is all they had then] a. In 1808, sold a vacant piece of land in Leicester Square to Elms. also owned several of the houses that formed the Square. The deed of conveyance contained a covenant by which Elms, his heirs and assigns would keep and maintain the property as a pleasure ground and square garden, enclosed by an iron railing. The covenant also stated that the property was to be uncovered with any buildings. This property passed by various means conveyances from Elms to . s purchase deed contained no similar covenant against building on the Square. did admit, however, the he purchased the land with notice of the original covenant in the 1808 deed. tried to assert the right to build structures on the garden as saw fit, and filed for an injunction to prevent from using the pleasure ground and garden for any purpose other than as an open area, uncovered with buildings- so go to equitable court b. There was a clear contract between and Elms, by which Elms promised not to use the land adjoining s houses in the Square for anything other than a square garden. If were allowed to purchase this land from Elms and violate their contract, then any owner of land who tried to sell part of his land, like , would risk having his remaining land be rendered worthless. This is because the sale price of the land in question would be affected by the covenant. likely paid less for the land than he would have had to pay for land unburdened by such a covenant. If were allowed to build on the land, would have effectively received unburdened land for the lower price of burdened land. Nothing could be more inequitable than allowing this, for could then resell the unburdened land at a higher price, and thus would be unjustly enriched. If a covenant is attached to property by its original owner, no one with notice of that covenant can purchase that property and not be bound by the covenant. An injunction was granted by the Master of the Rolls and the injunction is affirmed (the motion refused) c. A covenant will be enforceable in equity against a person who purchases land with notice of the covenant d. Notes on Tulk a. Could have gotten damages under real covenant at that time? needs vertical and horizontal privity? He has vertical. There was no horizontal privity in England unless a landlord leased a property to a tenant. So he could not estabslish horizontal privity. b. This didnt help too many people although it was an important development and takes off in US because it had recording system, because it still required notice and unless there was one, this case still doesnt help. So when there is a case, ask 2 questions. 1. Was there a original restriction between original parties? 2. Was there a notice for the subsequent purchaser? Actual, inquiry, etc. c. This case represents the first time it was acknowledged that there were equitable servitudes 1. The problem in England is that there is no recording system c.

60

2.

iv.

Notes (750) a. In most states today, law and equity have been merged, and a court in an equitable action for an injunction can give damages instead a. According to the Restatement (Third) of Property, it doesnt matter when a covenant is characterized as a real covenant or an equitable servitude 1. There isnt and shouldnt be a difference in the rules applicable to real covenants and equitable servitudes 2. The restatement drops real covenant and equitable servitude and refers to them as covenants running with the land b. The court may enforce the covenant by any appropriate remedy or combination of remedies, which may include declaratory judgment, compensatory damages, punitive damages, nominal damages, injunctions, restitution, and imposition of liens Creation of Covenants 1. A real covenant must be created by a written instrument signed by the covenantor a. It is an interest in land within the meaning of the Statute of Frauds b. If the deed creating a real covenant is signed by the grantor only, and it contains a promise by the grantee, the promise is enforceable against the grantee the grantee is bound by the act of accepting such a deed a. A real covenant cannot arise by estoppel, implication, or prescription (as can an easement) 2. Similarly, an equitable servitude is an interest in land a. It may be implied in equity under certain limited circumstances, it arises out of a promise and cannot be obtained by prescription 3. Sanborn v. McLean Supreme Court of Michigan, 1925 (751) a. In 1891, 91 lots were subdivided along Collingwood Ave. Each lot was designed for a sold solely for residential purposes. In December 1892, McLaughlin, then owners of the lots on Collingwood, deeded lots 37 to 41 and 58 to 62, inclusive, with restrictions that provided no residence shall be erected which shall cost less than $2,500, and nothing but residences shall be erected upon said premises. IN July 1893, they conveyed lots 17 to 21 and 78 to 82, both inclusive, and lot 98 with the same restrictions. On September 7, 1893, the McLaughlins sold lot 86 to by a deed which did not contain these restrictions. The s occupied a house on the lot. later started to erect a gasoline filling station at the rear of s lot. , who owned a neighboring lot, filed for an injunction. was enjoined by decree, and appealed. The SC of MI affirmed b. A negative servitude, such as a covenant restricting a lot to residential use, can be implied on a lot if a developer has set up a scheme for a residential subdivision and if the purchaser of the lot has notice of the covenants used to set up the scheme. Here, the McLaughlins imposed the restrictions on the Collingwood lots for the benefit of the lands they retained, namely to carry out the scheme of a residential district. Because they sold these lots with restrictions in order to benefit themselves, the servitude became a mutual one, and so the McLaughlins were bound by the same reciprocal negative servitude attached to lot 86 before acquired the land. Such an easement was still attached to lot 86 after the sale to and can still be enforced by if had actual or constructive knowledge of it. Because the abstract of title to lot 86 showed that lot 86 was part of a much larger subdivision, and because the deeds resulting in reciprocal negative servitudes were on record, was bound by constructive notice to follow the servitude. Furthermo re, the general plan for the residential district had been observed by all lot purchasers, whether explicitly restricted or not, for over 30 years. could not have avoided noticing the strictly uniform residential use of the neighborhood lots, and therefore were on inquiry notice to learn why all the lots conformed with each other. The least inquiry by would have revealed the servitude on lot 86 c. An equitable servitude can be implied on a lot, even when the servitude is not created by a written instrument, if there is a scheme for development of a residential subdivision and the purchaser of the lot has notice of it d. Notes on Sanborn a. Was the McLeans deed restricted at all? Nowhere in McLeans deed had the restriction that the lot was restricted to residential building. In Guillette, the factual distinction is clear that there was notice of the restriction. No restriction on deed, no restriction on recording anywhere, nothing. So how does the court get to this result? b. The legal reasoning is when a developer owns multiple lots and sells one lot with a restriction, all the lots that are still owned by the same owner is reciprocally bound by implication.--> implied reciprocal servitude c. The second argument is on the lack of notice. The courts protecting the purchasers who bought the land from the developer and didnt understand that the developer might default. Should they have been reasonably expected to make inquiry notice? In Guillette it was possible for them to have inquiry notice! But here? d. If the owner of 2 or more lots sells one with restrictions of benefit to the land retained, the servitude becomes mutual and during the period of restraint, the owner of the lot cannot break the servitude e. Reciprocal Negative Easement 1. Must start with a common owner 2. Subsequent purchaser must have actual or constructive notice f. If the developer wants to protect the interests of the homeowners in the subdivision, why not put the restriction in everyones deed? 1. Developer is trying to keep his options open if the development tanks, there wont be restrictions on all the lots 2. The court is trying to protect the people in the subdivision who think theyve p urchased in a singlefamily subdivision 3. Doesnt seem fair to the s

4.

admitted he had actual notice, but if he had denied actual notice, there couldnt and wouldnt be enforcement of the equitable servitude unless the opposition could prove notice ii. Court of equity which looks at fairness and not the law thinks that having a notice is the same with having a recording system. What this case does is create an idea necessary elements for an equitable servitude: i. Intent to create something that runs with the land ii. The promise must touch and concern the land iii. Subsequent purchaser must have notice of the servitude

i.

61

v.

A majority of states do have precedent allowing implied reciprocal servitudes 1. However, in CA, an equitable servitude must be created by a written instrument identifying the burdened lot; it will not be implied from the existence of restrictions on other lots in a subdivision 2. And in MA, covenants will not be implied from a general plan, but if the covenants on the burdened lot are in writing a general plan may be used to show that the neighbors in the subdivision were intended as beneficiaries and may enforce the covenants 4. Sanborn Hypo in Class [separate pages] important stuff~! 5. tries to enjoin from building something [outline]- only address issues that are present, dont write about everything and waste time & dont forget the counterargument (on the other hand) a. s 3 possible issues to prove 1. Restriction i. Express? ii. In the chain of title? iii. If not, is it express in someone elses chain of title in writing [Gillette]? iv. If not, is it implied restriction? 2. Standing for i. Original beneficiary? ii. If not, intended beneficiary at the time of transfer of s deed? iii. If not, 3rd party beneficiary 3. had notice i. If in the deed=notice ii. If in the chain of title, recording issue iii. If not, other chain [Guillette]? iv. If not, implied notice [Sanborn]? b. s 3 possible issues: my deed is not restricted; no standing; no notice Validity and Enforcement of Covenants 1. Equity imposes 3 requirements: a. Intent that the benefit and/or the burden of the covenant run to successors of the original parties; b. Notice on the part of purchasers of the original promisor; c. That the covenant touch and concern the land d. Sometimes required vertical privity for the benefit (but not the burden) of a covenant to run in equity 2. Neponsit Property Owners Association, Inc. v. Emigrant Industrial Savings Bank Court of Appeals of New York, 1938 (755) a. In January 1911, Neponsit Realty Company, owner of a tract of land, caused a map of the land to be filed in the office of the clerk of the county. This tract was developed for a strictly residential community. Neponsit Realty sold lots in the tract to purchasers. IN 1917, Neponsit deeded a lot to Deyer. The deed contained a covenant which provided that the property would be subject to an annual charge not exceeding in any rear the sum of $4, and the charge was to be used for the maintenance of the roads, paths, parks, beach, sewers, and such other public purposes. The covenant also stated that the charges would be payable to a Neponsit Property Owners Association (), as an assignee of Neponsit Realty who would be responsible for the use of the charges. IN addition, the covenant stated that owners would be required to pay the charge on the 1st of May every year, and that failure to do so would result in a lien on the land until fully paid. Emigrant () later acquired title to the Deyers land at a judicial sale. The deed from the referee of the sale to and every deed in the chain of title since the conveyance by Neponsit Realty claims to convey the property subject to the original covenant. brought this action to foreclose the lien and enforce the covenant for the annual maintenance charge. filed a moti on for judgment on the pleadings and the motion was denied. appealed. The Court of Appeals of NY held that the , as the agent of the property owners, can rightfully enforce the covenant b. The key question to consider is to what degree the covenant substantially affects the legal rights of the parties to the covenant. This distinguishes a covenant that runs with the land from a mere agreement between a promisor and promisee. Here, by paying the annual charge, an owner in the tract acquired a right of common enjoyment with other property owners in roads, beaches, public spaces and improvements to those areas. To fully enjoy these areas, the owners must help to pay for their maintenance. Thus, the burden was inseparably attached to the land, held by the various owners, which enjoys the benefit, and so the covenant clearly touches and concerns the land. There was also concern over a possible lack of privity of estate between and . never owned the roads or other public places mentioned in the covenant, and was created solely as the assignee of the property owners. However, looking beyond the corporate nature of the , it is clear that was formed as a convenient means of advancing the common interests of the property owners. It would be almost impossible to separate the interests of and . An affirmative covenant to pay money for improvements or maintenance done in connection with, but not upon, the land does touch and concern the land, and a homeowners association, as the agent of the property owners, can rightfully enforce the covenant c. An affirmative covenant to pay money for improvements or maintenance done in connection with, but not upon the land which is to be subject to the burden of the covenant does touch and concern the land, and a ho meowners association, as the agent of the actual owners of the property, can rightfully enforce the covenant d. Notes on Neponsit a. Courts reasoning requirement of Touch and Concern [murky] 1. The court doesnt really know where to go with this the rule is vague says it will use guideposts of precedent and holds to the English standard i. A covenant which runs with the land must affect the legal relations the benefits and the burdens of the parties to the covenant, as owners of particular parcels of land and not merely as members of the community in general (such as taxpayers): must be tied to the land ii. The difference between covenants which run with the land and covenants which are personal must depend on the effect of the covenant on the legal rights which otherwise would flow from ownership of land and which are connected with the land iii. Even though this calls for payment of a sum of money to be expended for public purposes upon land other than the land conveyed by Neponsit Realty to s predecessors but by that conveyance, the grantee has a right of common enjoyment with other g.

62

b.

3.

Notes a. b.

property owners in roads, beaches, public places, etc. For full enjoyment of the purchased piece of land, the public places must be maintained iv. For negative covenant, identity of the owner doesnt matter. As long as you get it from the owner of the land, its fine. v. But for covenant, we are looking for substitute for the benefit of the covenant (Diana writing comedy example) Courts reasoning Privity of Estate [who holds the right to enforce? Which owner of a parcel? But here, the Association didnt own a parcel] Its just saying that one can only hold someone to the restriction if hes a successor of benefit conferred to the original owner; Does he have a standing to enforce? 1. was organized to receive the sums payable by the property owners and expend them for the benefit of such owners 2. The covenant doesnt fall within any classification of restrictive covenants which have been enforced in this state and no covenant has been sustained in this state where the did not own property which would benefit by such enforcement so that some of the elements of an equitable servitude are present 3. The corporate has been formed as a convenient instrument by which the property owners may advance their common interest 4. Only blind adherence to an ancient formula devised to meet entirely different conditions could constrain the court to hold that a corporation formed as a medium for the enjoyment of common rights of property owners owns no property which would benefit by enforcement of common rights and has no cause of action in equity to enforce the covenant upon which such common rights depend i. In substance not in form, theres privity of estate between and

b.

c.

This decision was the first major decision on the validity of a very important type of covenant, the assessment covenant in common interest communities 2 problems with enforcement of assessment covenants at the time of the case a. Historically, courts in NY and other places were wary about enforcing affirmative covenants b. The party seeking to enforce the covenant had not succeeded to any land owned by either of the original parties and indeed did not own any land at all 1. Vertical privity on the benefit side i. The issue is best understood in terms of standing to enforce. Traditionally, a 3rd party had standing to enforce an equitable servitude only if that party had succeeded to land from the original covenantee, that is, if vertical privity existed between then even a 3rd party for whose express benefit the covenant was made could not enforce it if this vertical privity relationship was lacking ii. This limitation on standing does not cause any difficulty when subdivision restrictions are by neighbors in the subdivision, because the neighbors received their titles from the developer or other original covenantee iii. Neponsit established that in substance, not in form, there is privity between the parties the because it purchased in the chain of title as the original purchaser and the Property Owners Association as the agent to represent the property owners whose property was reciprocally benefited and burdened by the covenant c. Today its well settled that homeowner associations have standing to enforce development covenants if they have been granted enforcement power (has standing to enforce the development covenants unless the declaration of covenants expressly denies it) 1. Most jurisdictions today dont require vertical privity for the enforcement of covenants in law or in equity i. The general view is that s have standing to enforce covenants on the basis of the 3 rd party beneficiary doctrine of contract law Touch and Concern covenants restricting the use of land have almost always been held to touch and concern land. These negative covenants directly affect the uses to which the land can be put and substantially affect its value. On the other hand, courts have been wary of enforcing affirmative covenants against successors d. Courts are reluctant to issue orders to perform a series of acts requiring continuing judicial supervision e. Enforcing an affirmative covenant, which requires the covenantor to maintain property or pay money, may impose a large personal liability on a successor. Enforcement of a negative covenant restricting use of land limits the successors loss to the investment in the land itself f. An affirmative obligation, unlimited in time, resembles a feudal service or perpetual rent g. Affirmative covenants are often viewed as clogs on titles New Restatement Discards Touch and Concern a. Restatement (Third) of Property 3.2 supercedes the touch and concern requirement with a default rule that a covenant is valid takes the position that almost all negative and affirmative covenants are initially unobjectionable and that the intentions of the parties should be given effect ; maybe this is better approach because it enforces the court to say why they hold the way they do, but it is not an absolutely clear test either 1. The bases for holding that a covenant is invalid at its inception are that it is illegal or unconstitutional or [against] public policy i. Purpose of replacing the touch and concern rule with the new rule is to encourage courts to articulate more specifically why they find a particular covenant objectionable at its inception b. We still have the touch and concern rule because in reality, people.? c. Problems on page 767 (a) no flag of any kind may be displayed under the touch and concern test, fine; under the Restatement, may violate (2)s constitutional right (b) no sign other than address may be displayed, no Christmas light outside under the touch and concern test, its a violation b/c it touches the premise; under the Restatement, may violate (2) but can be countered by nuisance argument

63

(c) vi.

Scope of Covenants 1. Hill v. Community of Damien of Molokai Supreme Court of New Mexico, 1996 (773) a. In a planned residential subdivision, opened a group home for 4 individuals with AIDS. The individuals were unrelated and required varying degrees of home nursing care. lives on the same dead-end street where the home was located. and other neighbors noticed an increase in traffic once the group home was opened. claimed that the use of the property as an AIDS group home violated a restrictive covenant, applicable to all homes in the area, which mandated that no lot be used for any purpose other than single family residence purposes . sued to enjoin the further use of the property as a group home. argued that single family residence did not include group homes in which unrelated people lived together. challenged this argument and contended that the restrictive covenant violated the FHA . The trial court granted the injunction. appealed on the ground that the group home is permitted use under the covenant and, alternatively, the enforcing of the covenant would violate the FHA. The SC of NM reversed and vacated the injunction b. Ambiguous restrictive covenants should be construed in favor of the free use and enjoyment of property and against restrictions. The restrictive covenant at issue requires homes in the area to be used for single family residence purposes. 1) Operating the AIDS group home constitutes the residential use of property. The individuals share communal meals, do their own shopping, and provide emotional, financial, and spiritual support to each other. They contract with private health-care workers, none of whom reside at the home. The Community provides administrative assistance, collects rent from the residents, and enforces policies. The Communitys ac tivities do not render the home a nonresidential operation such as a group home or a boarding house. 2) Moreover, the residents of the home meet the single-family requirement. The word family is not defined in the restrictive covenant, and thus the use of the term is ambiguous. The court must interpret any ambiguity in favor of the free enjoyment of the property. There are a number of other factors that lead the court to conclude that family includes unrelated individuals: (1) The Albuquerque zoning ordinance defines family as any group of not more than 5 [unrelated] persons living together in a dwelling; (2) a strong public policy exists in favor of removing barriers preventing disabled persons from living in residential settings. (Policy is squarely set out by the FHA); (3) other courts have held that the controlling factor in considering whether a group of unrelated individuals constitutes a family is whether the residents bear the generic character of a relatively permanent functioning family unit. The court concludes that the residents of the AIDS home do have this character and should be considered a family. s use of the home does not violate the restrictive covenant. Restrictive covenants violate the FHA when they have a discriminatory intent, a discriminatory effect, or constitute a failure to make responsible accommodations. Even if the court were to adopt s proposed definition of family to include only individuals related by blood or by law, the court would still find for the C ommunity because the restriction would violate the FHA. Although s enforcement of the restrictive covenant was not intended to discriminate against AIDS victims (a protected class according to the FHA), the enforcement certainly had the effect of denying housing to the handicapped. The court must balance the s interest in avoiding increased traffic against s interest in providing housing to disabled individuals. The court concludes that the FHA factors weigh in favor of the Community, especially in light of the strong public policy favoring placement of disabled individuals in community living environments. failed to make reasonable accommodations as required by the FHA. A reasonable accommodation in this instance would have been not to seek enforcement of the covenant. c. Ambiguous restrictive covenants should be construed in favor of the free use and enjoyment of property and against restrictions; restrictive covenants with a discriminatory effect violate the FHA d. Hypo: 5 college men living in one house enjoin? First have to figure out whether the operating a group home this way constitutes residential use. 1. Land owner is profiting (Molokai didnt make profit directly), 2. Violating the covenant because the covenant says no more than 5 people. e. Why are they looking at the zoning law? They re trying to give a context of what family means. f. Court considered quality and nature of the relationship people in the home had. Is it ok to use that to evaluate the relationship for 5 college men living together? Is it a bad precedent to set up this standard of looking into nature of relationship? Maybe not b/c after couple of this kind of cases, cases may go away because court set aside what family means here. g. They dont want rooming house, leasing out rooms for profit. And 5 guys situations is much closer to the rooming house situation than the Molokai house. h. Went on to the FHA point although holding could have stopped before that because they dont want any bad precedent to be set. FHA(f), an addition to FHA later, had protection for the disabled. Even all the things the s did was not enough to show discriminatory intent, and in essence thats why FHA enacted the (f). i. Court understood that had traffic reasoning, but the c ourt said covenant had nothing to do with traffic and it could not be weighed more favorably against community interest. j. If the court finds for the , it undermines the protection FHA tried to give for the disabled 2. Shelley v. Kraemer (783) a. On February 16, 1911, 30 out of 39 owners of property along both sides of Labadie Ave entered into a covenant relating to the use and occupancy of the property. This covenant provided that, for the next 50 years after the date of signing, no party of any of the property was to be used or occupied by any person not of the Caucasian race, it being intended to hereby restrict the use of said property for said period of time against the occupancy by people of the Negro or Mongolian race. These 30 owners held title to 47 out of the total 57 parcels of land in that district. On August 11, 1945, (African American) received a warranty deed to one of these 47 houses from Fitzgerald. had no actual knowledge of the restrictive covenant at the time of purchase. On October 9 , 1945, and others brought suit to restrain from taking possession of the parcel bought. s also requested that judgment be entered divesting title out of and revesting title i n some other person the court saw fit. The lower courts held for and granted the injunction, judicially enforcing the covenant. However, the Missouri SC reversed b. The equal protection clause of the 14th Amendment prohibits judicial enforcement by state courts of restrictive covenants based on race. There is no question that the civil rights meant to be protected from discriminatory state action by the 14 th Amendment included rights to acquire, own, enjoy, and dispose of property. These rights are essential to the enjoyment of other civil rights and liberties. If the restrictions imposed by the covenant on the Labadie Ave property had been imposed by statute or ordinance, they would be clearly unconstitutional. Because it is the product of private, not state, action, however, the covenant by itself does not violate the 14 th Amendment. So long as the covenant is put into effect through

no solar energy device shall be installed on the roof of any house under the T&C test, violation; under Rest., it may violate public policy b/c energy saving and environmental issue is something state has interest

64

c. 3. Notes a. b.

voluntary action, it remains constitutional. Here, by contrast, the purposes of the covenant were secured through judicial enforcement by the state courts. It has long been established that the official action by state courts and judicial officers constitute action of the state, and this proposition applies to action regarding the 14th Amendment as well. The undisputed facts clearly indicate that s were willing buyers of a parcel of property on Labadie Ave, and that Fitzgerald was a willing seller. would have been free to occupy the property if not for the interventi on of the Missouri state courts. Though racial restriction was initially defined by a private agreement, the state nonetheless has the responsibility of complying with the 14th Amendment, and its action in judicially enforcing the covenant violated that Amendment Judicial enforcement of a restrictive covenant based on race constitutes discriminatory state action, and is thus forbidden by the equal protection clause of the 14th Amendment of the Constitution

vii.

The fact that the state must use the power to enforce individual covenant held them to 14th Amendment standard. If people wanted to deviate from the covenant, they could. Court just wouldnt do anything. So this was not a death note for covenants. Now they might be able to use FHA. c. If you have a client who were rejected from buying a house, 1) First, Look into Constitution [probably cant use it unless the court did something to uphold the covenant] 2) Then, FHA exemption applies? 3) Then, Discrimination applies? Termination of Covenants 1. Covenants, like easements, can be terminated in a number of ways including: expiration, release, abandonment, merger, estoppel, prescription, and condemnation a. They may be modified or terminated with the consent of all interested parties b. Modification and/or termination do not require unanimous consent c. Courts may modify and/or terminate covenants on the basis of changed conditions 2. Western Land Co. v. Truskolaski Supreme Court of Nevada, 1972 (786) a. In 1941, subdivided a 40-acre development outside the Reno city limits. subjected the lots to restrictive covenants which restricted them to single-family dwellings and prohibited any stores, butcher shops, grocery or mercantile business of any kind. The land around the subdivision was put to mainly residential and agricultural use, with very little commercial development. At the time, Plumb Lane, which bordered the subdivision to the south, went only as far west as Arlington Ave, and Reno had a population of about 20,000 people. By 1961, Plumb lane had extended further east, and had been extended by the city of Reno into a 4-lane blvd. Plumb Lane had become the major east-west artery through the southern party of Reno. By 1969, Renos population jumped to roughly 95,100 people. In addition, a major shopping center had been built across from the east end ot the subdivision, along with 2 more even further east. Despite these changes, the amount of traffic within the subdivision remained low. The homeowners () stated that this low level of traffic resulted in a safe living and playing environment for s children. The homes in the subdivision were also very well maintained. brought an action in district court to enjoin from building a shopping center on a 3.5 -acre parcel in the subdivision. The court held that the restrictive covenants were still enforceable and appealed. The SC of NV affirmed b. There is enough evidence that the subdivision is still suitable for residential purposes despite the changed conditions of the area, and that the restrictions remain of substantial value to the landowners. The lower level of traffic has left to a safe environment for children that is valued by . This safety would likely be compromised if commercial traffic were allowed. Though points to intent by the Reno city council to rezone the area in question for commercia l use, this intent does not prove that the property is more suited for commercial, rather than, residential use. A zoning ordinance does not override a private restriction and a court cannot be forced to terminate a restrictive covenant merely because of a change in zoning. Even if the parcel in question would be more valuable if converted to commercial use that does not permit to escape its earlier restrictions. Moreover, though points to violations by landowners, including a nursery in one house an d a contractors office in another, the violations are too isolated to frustrate the purpose of the restrictions and do not equat e to a waiver of the restrictions c. A restrictive covenant establishing a residential subdivision cannot be terminated as long as the residential character of the subdivision has not been adversely affected by the surrounding area, and it is of real and substantial value to the landowners within the subdivision d. Notes on Western Land a. Although the court grants the injunctive order, eventually theres commercial development on the property does that mean the case was decided wrongly? 1. No. Shows that injunctions are for sale. The s had an opportunity to bargain and what probably resulted was the most efficient outcome b. Why did the developer, , agree to create the covenant for residential use? At that time, it seemed like a good idea, but over time, land around developed and more people live there. Poor foresight! 20 or 30 years down the road, things can change and now wants to use his property more efficiently. c. But, the homeowners bought the property because things change and wanted to make sure they get what they want. They entered into the covenant because that was exactly what they wanted and they paid for it. They entered into the covenant to keep their condition against the changes of the circumstances or surroundings. d. Here, inside the subdivision, the covenant still was benefiting the people. But what if all the value of everyones residential use property goes down, but value for commercial goes up so much, and 4-5 home owners want to get out of covenant and sell it with destroying the covenant, would the court allow it? What if one person holds up and she has right to enforce? Everyones suffering so its a balancing test. Can argue that covenants hurting people and not helping people. Or you can argue that deal is the deal and that one person paid for it and it has sentimental value so cant force someone to give up their right to claim. Or purpose of contract is frustrated. e. Most of the time, we dont know the genuine reason of why people hold up. Sometimes people tried to get money and self-interested. If the hold up doesnt value the property and covenant anymore, then the court would probably let the covenant to terminate. 3. Rick v. West New York Supreme Court, Westchester County, 1962 (790) a. subdivided 62 acres of vacant land in 1946. Covenants that restricted the land to single -family dwellings were filed. In 1956, sold a -acre lot to , who built a house. IN 1957, the land was zoned for residential use. later contracted for the sale of 45 acres to an industrialist with the sale being conditioned upon the tract being rezoned for industrial use. The 45 acres was rezoned, but the sale fell through after refus ed to release her covenant. After selling only a few more lots, conveyed the remaining land to the other lot owners (). In 1961, contracted to sell 15 acres from the tract to a

65

viii.

hospital, but again refused to consent to the release of the covenant. The other owners () sued, claiming the covenant was no longer enforceable because of a change in conditions. The trial court held for and appealed. The NYSC affirmed b. In 1946, owned the land free and clear of all restrictions, and had the right to do what he felt was best for the property. chose to turn it into a residential development, and to encourage purchase of the resulting lots, imposed the residential restrictions. relied upon those restrictions when she purchased the property and has the right to continue relying on them. The balancing of equities and the potential advantages of having a hospital on the property are irrelevant. The fact that is the only person opposed to releasing the covenant does not make her right to have the covenant enforced any less deserving of this courts protection. has done nothing but insist upon adherence to a covenant which is just as binding as it was when and the other owners () entered it. s refusal to release the covenant, resulting from her satisfaction with the covenant as it stands, must be protected c. A landowner in a subdivision under a restrictive covenant has the right to insist upon adherence to the covenant even when the other owners consent to its release 4. Notes (791) a. Restatement (Third) of Property 7.10 Modification and Termination of Servitudes Because of Changed Conditions a. When a change has taken place since the creation of a servitude that makes it impossible as a practical matter to accomplish the purpose for which the servitude was created, a court may modify the servitude to permit the purpose to be accomplished. If modification is not practicable, or would not be effective, a court may terminate the servitude. Compensation for resulting harm to the beneficiaries may be awarded as a condition of modifying or terminating a servitude b. If the purpose of the servitude can be accomplished, but because of changed confditions the servient estate is no longer suitable for uses permitted by the servitude, a court may modify the servitude to permit other uses under conditions designed to preserve the benefits of the original servitude b. In MA a statute makes damages rather than an injunction the only remedy in a number of cases (793) a. No restriction shall be enforced or declared to be enforceable unless it is determined that the restriction is, at the time of the proceeding, or actual and substantial benefit to a person claiming rights of enforcement. Further, even if a restriction is found to be of such benefit, it shall not be enforced except by award of money damages if any of several enumerated conditions are found to exist. Those conditions are as follows: 1. Changes in the character of the properties affected the neighborhood which reduce materially the need for the restriction or the likelihood of the restriction accomplishing its original purposes or render it obsolete or inequitable to enforce except by award of money damages 2. Conduct of persons 3. In case of a common scheme the land of the person claiming rights of enforcement appropriate for accomplishment of its purpose 4. Continuation of the restriction on the parcel against which enforcement is claimed 5. Enforcement, except by award of money damages, is for any other reason inequitable or not in the public interest Common Interest Communities 1. Almost every state has adopted a statutory scheme for organizing a common interest community some are modeled after the Uniform Common Interest Ownership Act a. These statutes require a declaration of rules governing the community, which must be disclosed to purchasers b. In most common interest communities, a homeowners association, in which all homeowners are automatically members, enforces the servitudes set forth in the declaration establishing the common interest community a. The association, governed by a board elected by its members, may adopt new regulations reasonably necessary to manage the common property, administer the servitude regime, protect community members from unreasonable interference in the enjoyment of their individual property, and carry out other functions set forth in the declaration establishing the common interest community 1. The distinctive deature of a common interest community is the obligation that binds the owners of individual lots or units to contribute to the support of common property, or other facilities, or to support the activities of an association, whether or not the owner uses the common property or facilities, or agrees to join the organization 2. Condominiums a. The basic idea is simple: each unit (or interior space) in a condo is owned separately in fee simple by an individual owner a. The exterior walls, the land beneath, the hallways and other common areas are owned by the unit owners as tenants in common 1. Because each unit is owned separately, each owner obtains mortgage financing by a separate mortgage on the owners individual unit i. The failure of one unit owner to pay mortgage interest or taxes does not jeopardize the other unit owners b. The condo form of ownership can be adapted to residential or commercial use, and can apply to units in highrise buildings or to lateral developments such as townhouses and detatched dwellings a. The declaration of condos is filed before the first sale is made, and will provide for an association of unit owners to make and enforce rules, to manage the common areas, and to set maintenance charges levied against unit owners 1. Each purchaser, by accepting a deed, becomes an association member and must abide by its laws c. In common interest communities, any requirement of horizontal or vertical privity is met because the original purchasers are all in privity with the developer and subsequent purchasers are in privity with the original purchasers. Any requirement that a covenant touch and concern the land is usually satisfied 3. Nahrstedt v. Lakeside Village Condominium Association, Inc. SC of CA, 1994 (800) a. lives in a condominium at the Lakeside Village Complex. Article VII, 11 of he recorded covenants, conditions, and restrictions (CC&Rs) on the condominium provides that no animals (which shall mean dogs and cats) shall be kept in any unit and that the Association () shall have the right to prohibit maintenance of any pet which constitutes, in the opinion of the Board of Directors of the Homeowners Association, a nuisance to any other owner . owns 3 cats, which claims make no noise and are not a nuisance. has allegedly harassed by imposing increasingly large fines as a penalty for the cats. filed this action to obtain, among other things, a declaration that is e ntitled to keep the cats, in spite of the

66

b.

CC&Rs and that has no legal obligation to pay the fines. argues that the restriction was unreasonable under Civil Code 1354 (restrictions in the declaration of CC&Rs shall be enforceable equitable servitudes unless unreasonable). The trial court sustained demurrers on all 5 causes of action in s original complaint. The court of appeal, divided, reversed the trial courts judgment of dismissal agreed with and concluded that the could enforce the restriction only upon proof that the s cats would be likely to interfere with the right of other homeowners to the peaceful and quiet enjoyment of their property. The SC of CA reversed the decision of the court of appeals and remands The chief attribute of owning property in a common interest development is subordination of individual property rights to the collective judgment of the owners association together with restrictions on the use of real property. Inherent in the condo concept is the principle to promote the health, happiness, and peace of mind of the majority of the unit owners since they are living in such close proximity and using facilities in common, each unit owner must give up a certain degree of freedom of choice which he might otherwise enjoy in separate, privately owned property (Hidden Harbor Estates). In Ca, the legislature has made common interest development use restrictions contained in a projects recorded declaration enforceable unless unreasonable.

In Hidden Harbor Estates, the FL court distinguished 2 categories of use restrictions: (1) those set forth in the master deed of the condo project itself these should not be evaluated under a reasonableness standard, rather, such use restrictions are cloaked with a strong presumption of validity and should be upheld even if they exhibit some degree of unreasonableness nonenforcement would be proper where the restrictions were arbitrary or in violation of a public policy or some fundamental constitutional right. And (2) rules promulgated by the governing board of the condo owners association or the boards interpretation of a rule. MA followed FL in Noble where it held that properly enacted and evenly-enforced use restrictions contained in a master deed should be insulated against attack, except on constitutional or public policy grounds. Giving deference to use restrictions contained in a condo projects originating documents protects the general expectations of condo owners that restrictions in place at the time they purchase their units will be enforceable. Civil Code 1354 use restrictions for a common interest development that are set forth in the recorded declaration are enforceable equitable servitudes, unless unreasonable. Such restrictions should be enforced unless they are wholly arbitrary (p.804), violate a fundamental public policy, or impose a burden on the use of affected land that far outweighs the benefit. CC&Rs are essential to achieving stability and predictability so essential to the success of a shared ownership housing development. When courts accord a presumption of validity to all such recorded use restrictions and measure them against deferential standards of equitable servitude law, it discourages lawsuits by owners of individual units seeking personal exemptions, provides assurance to prospective condo buyers, and protects owners in the planned development from increases in association fees to fund the defense of legal challenges to recorded restrictions. The burden is on the challenger to show the restriction is unreasonable. The judiciary is a beneficiary of the presumption of validity because there will be fewer suits and those filed can be disposed of quickly where there is a basic rule for courts to apply. The dangers of not enforcing CC&Rs include: requiring courts to look at each suit on a case-by-case basis, imposition of a strain on the social fabric of the common interest development; frustration of owners who purchased in reliance on CC&Rs; substantial litigation costs on owners through their associations which would need to defend against such suits. Both cases relied on by the Court of Appeals are inappropriate because they fail to apply the deferential standards of equitable servitude law c. The enforceability of restrictions on the ownership and possession of pets should be decided in a trial court after evidence is heard as to whether the restriction was reasonable as applied to the particular facts of a case d. Given the substantial benefits derived from pet ownership, the undue burden on the use of property imposed on condo owners who can maintain pets within the confines of their units without creating a nuisance or disturbing the quiet enjoyment of others substantially outweighs whatever meager utility the restriction may serve in the abstract. The majority considers only one factor when justifying the preservation of health and happiness whether or not the restriction was recorded in the original declaration. If so, the majority says i ts presumptively valid, unless in violation of public policy. The activity at issue here is strictly confined to the owners interior space; it does not in any manner inva de other units of the common areas. The majoritys view is devoid of humanity nece ssary for a civilized society Hypo: Leslies sister in NV and her niece. Put off the law in this case and just use what weve learned so far. a. For home owners association: restriction was clear and it was backed by a good policy argument (protect the neighborhood and happiness); people cant interpret it in their own convenient way because there is too many litigation and after all, the cost of litigation will be distributed to the owners and that will be expensive (clear rule reduces cost); b. For Leslies sister: the purpose of the policy was not to prevent childrens activity but it was to prevent cluster of signs in the neighborhood; everyone in sub-division has a reasonable ground to argue their own case, cant be just one strict interpretation; campaign signs are exceptions because its value is on Constitutional reason and thats a rule. So, we can argue that this is not asking for case by case application of the rule but its asking to make this another rule that all the kids should be exempt from the rule and the rule should say all family children activity is fine. Notes (809) a. Contest between Blanket application v. Case by case application b. Hypo of Leslies sister after Nahrdstat, Based on the black letter law the court depends on (restriction must be applied unless its 1. Arbitrary, 2. Against public policy, 3. Outweigh benefit). If Leslies sister says the mine was arbitrary (cou rt defined arbitrary as irrationally related to health, . Or on page 804 bearing no rational relationship with preservation by saying that her activity bears no rational relationship with preservation of the sign. Home owners association argues that it does bear rational relationship with preservation because if all individuals do it, it affects the whole neighborhood. c. Hypo: the rule was not on the original declaration but in one of the home owners association meeting, the rule was passed and now wants to enforce it. First, there is no notice. You bought the house without the knowledge of the restriction. Would this still give reasonableness for home owners association? Now the burden of proof (page 811) shifts to home

4.

5.

67

X.

owners association to apply reasonableness standard and they must prove there is harm that is greater than benefit of freedom of choice of color (balancing the utility of having this restriction against the harm caused by not having the rule). d. Sometimes organization makes decisions on how to deal with present situation (i.e. termites) and an individual sues for that choice. Business judgment Rule says even if the outcome was bad, as long as they are reasonable choice, fine. e. For Co-ops, the association is treated as corporations and court uses corporation law so Co-ops have so much more power than individual associations. f. The new Restatement says of the standard of reasonableness determining reasonableness requires balancing the utility of the purpose served by the restraint against the harm that is likely to flow from its enforcement g. Business Judgment Rule a. In Levandusky, the court held that cooperative and condominium boards should be afforded the greatest possible degree of deference and were subject only to the business judgment rule 1. Prohibits judicial inquiry into actions of corporate directors taken in good faith and in the exercise of an honest judgment in the lawful furtherance of corporate purposes b. Difference between Business Judgment Rule and the Reasonableness Test 1. Unlike the business judgment rule, which places on the owner seeking review the burden to demonstrate a breach of the boards fiduciary duty reasonableness review requires the board to demonstrate that its decision was reasonable 2. Although in practive certain amounts of deference appeats to be accorded to board decisions, reasonableness review permits indeed, in theory requires the court itself to evaluate the merits or wisdom of the boards decision c. The more limited judicial review embodied in the business judgment rule is preferable why? 1. Board decisions concerning what residents may or may not do with their living space may be highly charged and emotional allowing an owner who is simply dissatisfied with particular board action a 2nd opportunity to reopen the matter completely before a court, which generally without knowing the property may or may not agree with the reasonableness of the boards determination, threatens the stability of the common living arrangement 2. The prospect that each board decision may be subjected to full judicial review hampers the effectiveness of the boards managing authority. The business judgment rule protects the boards business decisions and managerial authority from indiscriminate attack. At the same time, it permits review of improper decisions, as when the challenger demonstrates that the boards action has no legitimate relationship to the welfare of the cooperative, deliberately singles out individuals for harmful treatment, is taken without notice or consideration of the relevant facts, or is beyond the scope of the boards authority d. Criticisms of the Reasonableness Test 1. Courts shouldnt intrude into the internal rules of these private associations, and then only when subsequent regulations adopted by the association board are unreasonable 2. Homeowners regimes are not entirely voluntary and home buyers are coerced into accepting obligations they do not want in order to get the property they do want 6. New Yorks Cooperative Apartments (815) 7. Mulligan v. Panther Valley Property Owners Assn. (815) a. Panther Valley is a residential community in NJ that consists of more than 2,000 houses, including single-family, town and condo units. The members of the association voted to amend the communitys declaration of covenants and the association by-laws by declaring that no individual registered as a tier 3 sex-offender could reside in Panther Valley. One member of the association () challenged the amendment on the ground that it violated public policy. The court decided to apply a reasonableness test rather than NYs business judgment test because the changes were made by a vote of the members rather than by the board of directors. The court found that the record was insufficient to decide the question, but since it was the s burden to establish such a record, the association was entitled to judgment b. The record did not disclose that other similar common interest communities within the state have passed similar restrictions upon residency by tier 3 offenders. The court is unable to determine whether the result of the provision is to make a large segment of the housing market unavailable to one category of individual and indeed perhaps to approach the potential dangers recognized by the SC. It is entirely unclear if the association performs quasi-municipal functions, such that its actions perhaps should be viewed as analogous to governmental actions in some regards. Tier 3 registrants are not a protected group within the terms of NJs Law Against Discrimination and the court doesnt have the authority to deem the offenders handicapped Legislative Land Use Controls: The Law of Zoning a. Introduction i. Historical Background 1. Courts were reluctant to hinder the development of newly forming cities that were making the US rich. Didnt want to declare anything a nuisance except a use highly objectionable in a particular context a. Nuisance law did not prevent nuisances from arising, but merely gave damages or an injunction after the fact in an expensive lawsuit by one neighbor against another from a developers point of view, nuisance laws added the risk that capital sunk in land development might later be declared a nuisance b. Restrictive covenants were also incapable of dealing effectively with relations among neighbors in urban areas. They were useful only in new subdivisions and other developments of large acreage occurring under a single owner, who imposed the covenants c. In the early 20th century, environmentalists or social reformers or city planners turned to zoning, a theory designed to prevent harmful effects being visited upon neighbors 2. Howard came up with the idea of garden cities, which US city planners were interested in separation of used, protection of the single-family home, low-rise development, and medium-density of population a. Other visionaries also contributed to the public acceptance of city planningin particular what was on display at the Chicago World Fair a. In the 1920s, after zoning ordinances had incorporated the Garden City model, LeCobusier came up with the idea of building up and not out the low-rise, medium density took up too much land 3. Zoning Programs a. 1909 LA enacted an ordinance restricting industry to specified districts, away from residential areas

68

ii.

1916 NYC enacted the first comprehensive zoning program Important interests, then, had a stake in controlling the course of development and preserving property values a. Zoning was essential to healthy housing, with light and air, and to environmental planning for a great metropolis b. Spread rapidly after 1916, especially after the appearance of a Standard State Enabling Act issued in 1922 d. With the spread of zoning orders came constitutional attacks assertions that the new controls effected takings of property without compensation or worked deprivations of property without due process of law a. The following case was a test case came at a time when the USSC was striking down Congressional actions on grounds of substantive due process the state cant unreasonably interfere with life, liberty and property 1. The realty boards thought it a favorable case for a broad holding of unconstitutionality for several reasons. In 1924, a federal district court found the Euclid ordinance unconstitutional 4. Village of Euclid v. Ambler Realty Co. USSC, 1926 (828) a. is a suburb of Cleveland most of the land is used for forms or is undeveloped. owns a 68-acre tract of land in the western end of the village. There are residential lots with buildings to the east and west of s land. In 1922, (Village ) adopted a comprehensive zoning plan to regulate and restrict the use od land, as well as the size of the lots and the heights of the buildings. The ordinance divided the city into 6 districts and each higher numbered district included the uses of the district below it. The ordinance is enforced by the inspector of buildings under the board of zoning appeals and the board is authorized to make rules in order to implement the ordinance, as well as impost penalties for violations. The board can also interpret the ordinance in harmony with its general purpose and intent, so that the public health, safety, and general welfare may be protected. claims this land is vacant and that it has been held in order to sell and develop for industrial purposes (s land falls mostly in the residential area). If used for industria l purposes, the land is worth $10,000/acre, but if its kept for residential use, its worth only $2,500/acre. The record incidates that the use of the land (normal and reasonanbly expected use) along the highway is for industrial and trade purposes. claims the ordinance violates s constitutional rights, at both the state and federal levels, against deprivation of property and liberty without due process of law and denies equal protection of the law. brought suit for an injunction restraining the enforcement of the ordinance and all attempts to impose or maintain as to s property any of the restrictions, limitations and conditions. The court below held the ordinance to be unconstitutional. The USSC reversed b. It is not necessary to decide separately if this ordinance violates the OH constitution, as the question is substantially the same. Building zoning laws are relatively new, but are necessary and valid. The increasing urbanization of modern life has required the use of regulations which, 50 years ago, would have been considered arbitrary or oppressive. The ordinance under review must be justified through some aspect of police power. The line is not a clear one, but varies with circumstances and conditions. As with the law of nuisances, th e maxim sic can serve as a guide. The building of a structure should be considered after the building itself is looked at in connection with the circumstances and its surroundings. , though essentially a suburb, is a separate political body and as such , it has the power to govern itself as it sees fit within the limits of the OH and Federal Constitutions. Here, the serious question is over the exclusion of apt. houses, stores and shops and other similar establishments from residential areas. The majority of courts sustain these restrictions. Commissions and experts produce reports about the separation of residential, business and industrial buildings. They have point to the ease of providing appropriate firefighting measures, increased safety for children, etc. The restriction on apt. houses helps preserve the quiet, open character of single-family residential neighborhoods by preventing heavy traffic, overcrowding, etc. Unless the arbitrariness or unreasonableness is proven, or it is shown that there is no legitimate state interest, the ordinance cannot be declared unconstitutional. It is possible that the provisions set forth in a zoning ordinance may be found arbitrary or unreasonable in specific situations, but not here. When an injunction is sought, however, because of the mere existence or threat of enforcement of the ordinance which may cause injury, the court will not go over the ordinance, sentence-by-sentence, to find which parts are constitutional and which are not. Without a specific complaint of actual injury, a land owner cannot challenge the constitutionality of such an ordinance c. Zoning ordinances are a valid exercise of the police power and thus do not violate the constitutional protection of property rights d. Notes on Euclid a. Court demonstrates extreme deference to the legislature (unless the legislature is doing something that lacks state interests, is arbitrary or unreasonable) 1. The legislature is in the best position to figure out how to plan a municipality i. Problem the minority is not on equal footing among the legislature because the interests of the majority will always prevail. The reason for going to court is that both parties are on equal before the judge and the minority interest will be heard b. Court doesnt hold that zoning is always unconstitutional just that no one can facially attack a zoning scheme 1. If a particular landowner is aggrieved, their claim can be grounded in the constitutional right of the taking of personal property without just compensation (grounded in the 5 th Amendment) The Structure of Authority Underlying Zoning (838) 1. Enabling Legislation a. Zoning is an exercise of the police power essentially, the power of government to protect health, safety, welfare and morals b. Generally speaking, the police power is held to reside in the state, but in the case of zoning all states have adopted enabling acts that delegate zoning authority to local governments a. Standard State Zoning Enabling Act was adopted at one time or another in all 50 states and is still in effect in many of them 1. empowers municipalities to regulate and restrict the height, number of stories, and size of buildings and other structures, the percentage of lot that may be occupied, the size of yards, courts, and other open spaces, the density of population, and the location and use of buildings, structures, and land for trade, industry, residence, or other purposes 2. Permits division of municipalities into zones of appropriate number, shape, and area, and provides that regulations may vary from one zone to the next i. Regulations must be made in accordance with a comprehensive plan and designed to lessen congestion in the streets; to secure safety from fire, panic, and other dangers; to promote health and the general welfare; to provide adequate light and air; to prevent overcrowding; etc. b. c.

69

3.

b.

The Comprehensive Plan a. The Standard Act says that zoning regulations shall be in accordance with a comprehensive plan a. A comprehensive plan is a statement of the local governments objectives and standards for development 1. Based on surveys and studies of the citys present situation and future needs, theidea being to anticipate change and promote development b. Only about the states require comprehensive plans c. As to those with requirements, judicial attitudes vary greatly 1. Some hold, absent very specific language in the enabling legislation, that the plan need not be written down in a document separate from the zoning ordinance itself 2. Some consider that the statement of purpose in the zoning ordinances preamble is evidence of an underlying plan to which the ordinance conforms 3. Some have founf the scheme of regulations in the ordinance to the the plan The Nonconforming Use i. PA Northwestern Distributors, Inc. v. Zoning Hearing Board Supreme Court PA, 1991 (841) 1. On May 4, 1985, opened an adult bookstore in PA. 4 days later, the local Board of Supervisors published a notice of its in tention to amend the town ordinance to regulate adult commercial enterprises. A public hearing followed, and on May 23, 1985, the Board adopted Ordinance No. 243, effective May 28, 1985. This ordinance imposes extensive restrictions on the location and operation of adult commercial enterprises. It also includes an amortization provision which gave all those who operated pre-existing businesses in conflict with this ordinance a period of 90 days to comply. s bookstore, by definition, is an adult commercial enterpri ce under Ordinance No. 243, and it does not fall into any of the areas designated for such business by the ordinance. The Zoning Officer of the town informed that the bookstore did not comply with the ordinance. then filed an appeal to the Zoning Hearing Board () , challenging the validity of this 90-day provision. After a hearing, upheld the validity of the ordinance as applied. then filed an appeal the the Court of Common Pleas. No further evidence was taken, and the appeal was dismissed. The decision was appealed and the Commonwealth Court affirmed, citing Sullivan (provisions for the amortization of nonconforming uses are constitutional exercises of the police power so long as they are reasonable). The Commonwealth Court held that the real and substantial be nefits to the town of elimination of the nonconforming use from this location more than offset the losses to the affected landowner, and ruled against . appealed to the PA SC which reversed 2. Sullivan is not a correct statement of the law regarding amortization provisions. Amortization is not a reasonable means of zoning regulation. IN PA, all property is held subject to the right of reasonable regulation by the government that is necessary to preserve the health, safety, morals, or general welfare of the people. A zoning ordinance is generally presumed to be valid. That presumption, however must be balanced by an individuals constitutionally guaranteed right to use property without government rest rictions, except when his use creates a nuisance or violates a covenant, restriction, or easement. It has long been the law in PA that local governments cannot compel a change in the nature of an existing lawful use of property. A lawful nonconforming use establishes a vested property right in its owner that cannot be infringed upon unless it is a nuisance, it is abandoned, or it is extinguished by eminent domain. If a zoning law or regulation has the effect of depriving a property owner of the lawful pre-existing nonconforming use of his property, it amounts to a taking for which the owner must be justly compensated. Here, the amortization provision deprives of the lawful use of its property by forcing it to cease its business operations there within 90 days. The PA Constitution protects the right of a property owner to use his property in any lawful way that he chooses. This ordinance restricts future yses and extinguishes a present lawful nonconforming use against the owners wishes. If municipalities were allowed to amortize nonconforming uses out of existence, economic development could be seriously compromised, with investors afraid of changes in the zoning laws. Further, and use could be subsequently amortized out of existence without just compensation to the property owner. No use and no property owner would be safe 3. If a zoning law or regulation has the effect of depriving a property owner of the lawful re-existing nonconforming use of his or her property, it amounts to a taking for which the owner must be justly compensated 4. Sullivan should be pheld, because a reasonable amortization privision is valid if it reflects the consideration of several factors. Any blanket rule against amortization provisions would prevent effective zoning, unnecessarily restrict a states police power, a nd prevent the elimination of nonconforming uses to further the public interest. Several factors have been used to determine the reasonableness of amortization, including the duration of the investment in the property use; the length of the amortization period in relation to the nonconforming use; and the degree of offensiveness of the nonconforming use to the character of the surrounding neighborhood. A community should have the right to change its character withoug being locked into outdated definitions of what is or is not offensive. Here, the amortization provision is not a reasonable one as it does not provide adequate time for the nonconforming use to be eliminated. 90 days is not enough time to allow a merchant to close a business and settle contractual obligations. Moreover, such a period is not enough for to find an alternative means of income or obtain a reasonable retur n on its investment 5. Notes on PA Northwestern a. The problem is not the amortization period if it had been 5 years and not 90 days, the majority would have come out the same a. Majority is concerned with the landowner who has a vested property right already invested in the store and had no notice b. The concurring opinion, on the other hand, would have come out differently if the amortization period was 5 years instead of 90 days 1. Concurrence was upset the landowner did have adequate time to get a return on his investment 0 if the period were long enough to get a return and then is phased out, the concurrence would say this was not a taking b. Duration of a Nonconforming Use a. Concurrence limits the duration of the nonconforming use whereas the majority says once a right to operate is established, the municipality can never kick you out unless theres compensation

2.

To enact a zoning ordinance a city must create a planning (or zoning) commission and a board of adjustment, both composed of citizens as appointed by the mayor i. The ordinance must be enacted by the city council and if it needs amending later, the commission recommends the amendment to the city council, which must adopt the amendment for it to become legally effective ii. May grant a variance when the zoning restrictions cause the owner practical difficulty or unnecessary hardship and may also grant a special exception when specific requirements set forth in the zoning ordinance are met

70

1.

After this case in PA, the only way to get rid of nonconforming uses is: i. With a valid nuisance claim against the nonconformist ii. Let the nonconformist stay forever iii. Buy out the nonconformist

XI.

Notes (846) 1. Changes in jurisdictions without amortization legislation, the right to maintain a nonconforming use runs with land; hence it survives a change of ownership as to change of use, some jurisdictions provide that nonconforming uses may expand, especially to meet natural changes such as increased demand. a. Some allow one nonconforming use to be changed to another nonconforming use, but usually only if the change reduces the impact of the use on the zone in question b. Destruction of a nonconforming use (by act of G-d or otherwise) usually terminates it, and so too for abandonment, which requires intent to abandon the nonconforming use 2. Amortization a. Sullivan held that the distinction between an ordinance restricting future uses and one requiring the termination of present uses within a reasonable period of time is merely one of degree b. Courts in the 24 or so jurisdictions approving the amortization technique claim to require a reasonable period for the particular nonconforming use in question but in practice, this isnt really the case a. Factors considered relevant to an assessment of the reasonableness of a particular amortization period are: 1. The nature of the use in question 2. The amount invested in it 3. The number of improvements 4. The public detriment cause by the use 5. The character of the surrounding neighborhood 6. The amount of time needed to amortize the investment c. Where theres amortization legislation, the nonconforming use right ends at the termination of the applicable period, whether or not the property has been sold 3. Vested Rights a. In the case of nonconforming uses, a pre-existing operation is protected; plans to engage in some particular use are insufficient b. In the case of the vested rights doctrine, a proposed use mught be protected if sufficient commitments have been made plans drawn, permits obtained, the site prepared, construction begun in reliance on existing zoning requirements that are subsequently changed in a way that invalidates the proposed use a. Estoppel is sometimes applied when developers rely reasonably and to their detriment on the issuance of a permit and proceed to make substantial expenditures but the developer cannot rely on a permit unless he proceeds in good faith Eminent Domain and the Problem of Regulatory Takings a. Unsatisfied with private arrangements (servitudes) and nuisance law and the means of land use control, the government might and often does embark on more activist courses leaving property in the hands of its owners but regulating its use, or taking property from its owners and reallocating it to governmentally preferred uses i. Regulating example of this is zoning ii. Taking eminent domain 1. The power of government to force transfers of property from owners to itself a. That the government has the power of eminent domain is beyond dispute, but there are constraints the 5th Amendment enjoins, not shall private property be taken for public use, without just compensation a. Virtually all state constitutions have similar language; in any event, the 5 th Amendment applies to the states through the Due Process Clause of the 14th Amendment b. The Power of Eminent Domain: Sources and Rationales i. Rationales for the taking power 1. Sovereign states had original possession derived from grants from the state and was held subject to an implied reservation that the state might resume its ownership 2. Eminent Domain is the natural consequence of royal prerogatives that inhered in the concept of feudalism (on this view, the taking power is remnant of feudal tenures) 3. Eminent Domain is an inherent attribute of sovereignty, necessary to the very existence of government most common approach ii. Notes (943) 1. Posner provides a functional justification for the taking power that stresses efficiency a. Eminent doing is necessary to prevent monopoly. Once a railroad or a pipeline has begun to build its line, the cost of abandoning it for an alternative route becomes very high. Knowing this, people owning land in the path of the advancing will be tempted to hold out for very high prices a price in excess of the opportunity cost of the land. Transaction costs will be high, land-acquisition costs high, and for both reasons the right-of-way company will have to raise the price of its services. The higher price will induce some customers to use substitute services. Right-of-way companies will have a smaller output; as a result they will need, and buy, less than they would have bought at prices equal to the opportunity costs of the land.as a result of this, land what would have been more valuable to a right -of-way company than to its present owners will remain in its existing, less valuable us es and this is inefficient In settings of high transaction costs people must be allowed to use the courts to shift resources to a move valuable use, because the market is by definition unable to perform this function in those settings b. Duty to Compensate Posner also suggests an economic rationale for the compensation obligation private investors would be inhibited by the thought that government will snatch away the fruits of their venture and too little capital will be invested in productive enterprises a. The compensation requirement serves the dual purpose of offering a substantial measure of protection to private entitlements, while disciplining the power of the state, which would otherwise over-expand unless made to pay for the resources that it consumes c. The Public-Use Puzzle i. 5th Amendments mention of public use is read to mean that property may be taken only for such uses the government may not condemn for private purposes, however willing it might be to pay compensation for the forced transfer (p.948) 1. The breadth of the eminent domain power hinges on the meaning attached to public use

ii.

71

ii.

iii.

iv.

Kelo v. City of New London USSC, 2005 (945) 1. In 2000, the City of New London () proposed a development plan in an effort to revitalize the local economy. In tota l, the plan involved 7 parcels of land, some of which was privately owned, intended to be used for a mixture of commercial, residential, and recreational purposes. appointed and authorized the New London Development Corporation, a private nonprofit, to negotiate and purchase the private property necessary to implement the plan. When negotiations with and 8 other property owners broke down, initiated condemnation proceedings under its eminent domain powers. made no claim that the properties were i n poor condition. The petitioners brought suit in superior court, challenging s actions, claiming that the taking of their proper ty was not for public use within the meaning of the 5th Amendment. The superior court held that some of the takings were invalid, while others were permissible. Both parties appealed to the CT SC, which held that all of s takings were valid as a part of a developme nt plan intended for public use and in the public interest. The USSC affirmed the decision of the CT SC 2. Here, the taking was effectuated pursuant to a carefully considered development plan, and neither court below found an illegitimate purpose. However, the taken land is not intended to be entirely opened to the public, nor are the lessees of the land compelled to entirely open the land to the public. Nonetheless, public use does not require that the property be intended only for use by the general public, but rather that the taking be for a public purpose. (Test #1: Public Use, p. 950). Therefore, the takings are valid if the development plan authorizing the takings is intended for a public purpose. When considering whether a development plan is for a public purpose, considerable deference is given to legislative judgments. Plans are not considered piecemeal, lot by lot, but rather in their entirety to determine the overall intentions of the plan. Here, the plan was carefully considered to promote the local economy and public welfare. Although the plan combines commercial, residential, and recreational purposes for the condemned land, transferring some private property from one individual to another , the plans overall character furthers the public purposes of job growth, increased tax revenue, and economic development. Economic development is a longstanding function of government. Just as with the conversion of blighted areas into respectable communities and the breaking up of land oligopolies, economic development constitutes a pub lic purpose, even if the land itself is not entirely used by the public 3. Takings of private property for use by other private citizens pursuant to an economic development plan intended for a public purpose are valid under the 5th Amendment 4. The deferential standard of review employed by the Court should not hide the fact a taking should not survive the public use test if there is a clear showing that its purpose is to favor a particular private party, with only incidental or pretextual public benefits That a taking for economic development is not presumptively invalid, as petitioners had urged, does not foreclose the possibi lity that a more stringent standard of review might be appropriate for a more narrowly drawn category of takings 5. There are 3 categories of takings that had been held to satisfy the public use requirement in the past: (1) transfers of private property to public ownership (as for a road); (2) transfers to private parties such as common carriers, railroads, and the like, who make the property available for public use; (3) transfers to private parties as part of a program to serve a public purpose. This last category is the most troubling, especially where, as here, the public purpose is economic development. Kelo takes this too far, leaving the door open to governmental taking of any private property capable of improvement and resale under any economic development plan. The beneficiaries are likely to be those citizens with disproportionate influence and power in the political process (large corporations, development firms). As for the victims, the government not has license to transfer property from those with fewer resources to those with more What we already know from the beginning of the class 1. Sometimes there are necessary projects that would benefit the whole. So if we require the government to act as a regular market participant, some individuals holding the property would win windfalls in the expense of everyone else through governmental taxes. Government is usually deemed as a deep pocket with infinite resources, so some people would try to take advantage of their situation. 2. Granted that sometimes governmental taking is necessary. 3. However, there is subjective value attached to property. Thus, the price you assign to your property may be much greater than what the market would pay for it. So should the law recognize or not recognize that subjective value? 4. Also think about injunction and damages. If you dont give injunction, essentially you have to buy out the pr operty, so it eventually forces individuals to realize their subjective value attached to the property 5. Of Course, government has to pay compensation. But thats what government assesses as the market value. So, that takes away t he power individuals own to determine the value of your property. Is that OK? Plus, the assessment of the value of your property is done by an expert hired by government, and if you want to challenge the $, you have to hire another expert and its expensive . 6. Hypo: Trader Joes coming into town under Eminent Domain. Now that we have Kelo as precedent, can the city do this? Yes No 1. Is it for public use (First question you have to ask)? Yes, 1.No public purpose. Scope (just one business v. in Kelo, the economic rejuvenation + deference to legislation (better whole development) + transfer from one owner to another evidence + elected body) (p.948) is explicitly prohibited in Kelo + just depending on more profitable use will give too much power to government [no limit] + democratic process doesnt always work to protect the minority when majority wants Traders Joe and some people have more power than others to influence legislature a. Local government is in better position to make the decision than court because they considered much more facts. So, if the municipality determines that there is need for economic rejuvenation, deference should be given. Furthermore, they were elected, so democratic process protects LO better than courts. They can be voted out. b. Some incidental benefit is not enough to support government to use ED power because thats giving too much power to government. Increase of tax alone isnt enough. Only more profitable use alone should be stopped by Constitutional right for private protection and limit the states power. Notes (952) 1. Public Use a. 2 basic opposing views of the meaning of public use a. That the term means advantage or benefit to the public (broad view) b. That it means actual use of right to use of the condemned property by the public (narrow view) b. What should the public use test be? a. Berger Focus on the contemplated ends of an act of condemnation if the ends are sufficiently public in one sense or another, the test is passed 1. Essentially permits an act of condemnation so long as the objective it serves is in the public interest, as an exercise of the police power must be

72

c.

Epstein public use must involve provision of public goods in the technical economic conception a limited category or at least provision of goods, like highways and parts, open to the public at large without discrimination Level of Scrutiny a. Kelo court gave tremendous deference to the legislatures affording them broad latitude in determining what public needs judtify the use of the takings power 1. Most state courts take the same approach 2. But legislature can make law that limit its own power for ED that b.

d.

Just Compensation 1. Has been held to be satisfied by payment of market valus Compensation in the constitutional sense is therefore not full compensation, for market value is not the value that every owner of property attached to his property but merely the value the marginal owner attaches to his property a. The taking confiscates the additional personal value that it obtained from the property 2. Fairness Concerns people put more value into the land than what a willing buyer would pay in cash to a willing seller 3. Holdout Concern people may say they attach a greater value to their land than they actually do, just to get more money because its the government theyre dealing with (idea that the person writing the check isnt spending his own money, that the government has deep pockets, etc.) 4. If a taking flunks the public use test, the government may go forward only by purchasing the property through voluntary transactions with owners, who set the price for themselves Physical Occupations and Regulatory Takings i. If the government wishes to condemn private property for public use, it must comply with procedures designed to assure owners due process of law 1. Step 1 filing of a petition in court 2. Step 2 notice to all persons with an interest in the property in question to condemn (government might have to show the taking is necessary at this stage) 3. The court can give the government permission to enter and inspect the subject property; it may require the government to make a security deposit for the eventual condemnation, in an amount based on the compensation estimated to be awarded at the end of the proceedings 4. If theres a jury trial, it is typically the jury that determines judt compensation; issues of public use and necessity are decided by the court 5. At the conclusion of a successful condemnation action, the government must pay the compensation awarded plus interest, if any, accrued from the time of the taking 6. Dissatisfied condemnees may appeal ii. This is a puzzle. Hard to predict how the court will come out and hard to figure out where the state power is legitimate as police power and where its too much and becomes taking. iii. Whether a Taking has Occurred in Consequence of some Government Activity 1. Loretto v. Teleprompter Manhattan CATV Corp. USSC, 1982 (960) a. June 1, 1970 Teleprompter () installed a cable on an apartment building in NYC. The owner gave the exclusive right to provide cable tv services to the tenants. The cable was about inch in diameter, and about 30 feet long. It ran along the roof of the building, and was attached to 2 small directional taps, which allowed for future connections. These and other components were attached directly to the masonry. The cable served as a crossover line, helping provide other buildings on the block with CATV. purchased the building in 1971. connected a direct noncrossover cable line into the building 2 years later. Earlier, the state enacted executive law 828, effective January 1, 1973, which provided that a landlord may not interfere with the installation of cable facilities upon his property or premises. The state commission on cable tv said that a landlord is only entitled to a one-time $1 fee for installation. was not aware of the cable until after she bought the building. brought a class action against on behalf of all owners of real property in the state on which placed its components. claimed it was a trespass and, insofar as it re lied on 828, a taking without just compensation. NYC intervened. The trial court granted summary judgment for and the City. The Court of Appeals, over dissent, upheld the statute. The USSC reversed and remanded on the issue of compensation b. A permanent occupation has long been viewed as arguably the most serious invasion of an owners property interests. A classic example of this is found in cases where a person constructs a dam which permanently floods another persons property. A permanent physical occupation prevents the owner from both possessing the occupied space and excluding the occupier from possession and use of it. This power to exclude is one of the most important elements in an owners bundle of property rights. The owner is denied any control over the use of the property. Though an owner may still have the right to sell the property that right is seriously devalued, as a purchaser would also be unable to use the occupied space. While the size of an occupation should be considered in determining compensation, it should make no difference in finding whether a taking has occurred. argues 828 only applies to rental property, and thus is a permissible regulation. An occupation of only one type of property, however, is nonetheless an occupation. This reasoning will not have adverse consequences on the states ability to regulate housing conditions, because such regulation imposes duties on landlords and tenants, not on outside 3rd parties. Landlords can still install fire extinguishers and smoke alarms, provided 3rd parties are not involved. The state can still regulate an owners use of property, but it cannot authorize a permanent physical occupation of the property c. A permanent physical occupation of an owners property authorized by the government constitutes a taking of property which requires just compensation, regardless of the public interest it may serve d. The permanent physical occupation formula is a very problematic standard. The exact meaning of permanent is unclear. 828 only requires that the cable equipment be allowed as long as the building remains residential and the CATV company wishes to retain the installation. If this law results in a permanent occupation, other NY statutes that compel a landlord to make physical attachments to his property must also constitute takings, regardless of the public interests they may serve. The majoritys 3rd party problem would still exist even if owned the cable herself. If continued to transmit its signal through the cable, a host of conceptual arguments on whether such signal is physical could result. The distinction between permanent occupation versus temporary invasion has no basis in Takings Clause precedent or economic logic. Takings claims should be evaluated under a multiple factor balancing test and not by a set formula. Also, would have had no other use for this space, and any determinat ion of compensation should be made with this in mind. Moreover, the majority assumes that the tenants have no countervailing interest in allowing to use the space. Finally,

v.

73

2.

3.

4.

this decision suggests that the legislature cannot exercise its police power to grant the rights to even 1/8 cubic foot of space Notes (971) a. Majoritys per se categorical rule: if a property owner can characterize the government interference as a permanent physical occupation, and can convince the court thats whats going on, the owner of the property is entitled to compensation (the only remaining question is how much) Hadacheck v. Sebastian USSC, 1915 (973) a. is the owner of a tract which is now within the city limits of LA. An ordinance is in effect in the city which makes it unlawful for any person to establish or operate a brick yard or any other facility for the manufacture or burning of bricks within city limits. At the time purchased the land, it was outside city limits. did not expect the city to annex the surrounding area. On this tract, there is a very valuable bed of clay, which can be used to make fine quality bricks. The entire tract, if used for brick-making purposes, is worth about $800,000; if the tract were used for any other purpose, it would be worth no more than $60,000. has already made significant excavations over much of the tract; it cannot be used for residential purposes. has also set up expensive machinery for the manufacturing of bricks. The city has since grown and annexed the land adjoining the brick yard. argues, among many things, that his tract contains particularly fine clay, so that if the ordinance is declared valid he will be deprived of the use of his property without just compensation. also argues the brickyard is not a nuisance, and does not pose a danger to public health and safety. claims that no one has registered a complaint about it during the 7 years has operated it. Affidavits were provided which counter s allegations that the brick yard was not offensive to public health, claiming that fumes and dust from the yard have caused discomfort to those living nearby. was convicted of a misdemeanor for his violation of the city ordinance. b. A regulation that deprives an owner of property for the purpose of prohibiting a nuisance is an exercise of the police power, and therefore does not result in a taking which requires compensation. This is one of the most essential powers of government, and has few limitations. Though its particular use here may seem harsh, the general need for such power precludes any limitation, provided that the power is not used arbitrarily. A police power cannot be challenged by a persons interest in property because of conditions that were in effect when the property was obtained. To all ow such opposition would hinder the growth and development of cities. To promote growth, private interests must yield to the good of the community. Granted, s business of brick-making is lawful in and of itself. It is clearly, however, within the power of the state to regulate such a business and to declare that, in certain circumstances and certain areas, the business would constitute a nuisance. The affidavits provided, claiming the brickyard led to discomfort on nearby residents, further demonstrates that the adjoining area is not being used for residential purposes. Because of this situation, the brickyard does constitute a nuisance, and thus can be prohibited. Though claims the enforcement of the ordinance will deprive him of the use of his property and his business, this is not entirely true. While could not operate a brickyard on his tract, is not prohibited from removing the clay to be used at a brickyard outside the area bound by the ordinance. Though such operation of his business would be difficult from a financial standpoint, its not impossible c. A regulation that deprives an owner of property for the purpose of prohibiting a nuisance is an exercise of the police power, and therefore does not result in a taking which requires compensation Lets compare Loretto and Hadacheck Loretto; Taking Hadacheck; No taking Cable line Existing Brickyard State prohibited brickyard Permanent Physical Occupation = Taking [the rule makes sense Loses business when there are only a few discrete LO and its not difficult to pay them, but here its not the case, so why apply here?] Why not apply PPO here? Why isnt this taking? Another justification for PPO is Protection of minority right, and property right argument- doesnt make sense: wont undermine the value of the property, wont undermine the right to the property Court says Rule is better than standard: clear benefits would be decrease of judicial economy by decrease of litigation (government must think through and should be willing to pay, or dont take it)- this must be the only reason with substance But is this really a bright line rule? What is permanent and what is occupation? (p.969)- cable line can be removed so it is not necessarily permanent; p.972 Cosby case had problem with physical occupation with airspace being part of his land Advice: Are you willing to pay for it? Litigator: if we can make the case by saying that government action as permanent physical invasion, we can get money Protection of minority right argument it should fly here because its only 2 brickyards and unlikely to be protected by democratic process. How come this isnt sufficient reason to take that reasoning? Court said its nuisance case so government has to protect the healthy and safety of the public. 2nd bright line rule: if government action can be categorized as protecting the public, then its not taking He picked the place first, and it was outside the city limit at that time. If there was a case brought by his neighbor on nuisance, brickyard would have had easy case because there is notice at the time of purchase and they made the choice Characterize issue here as health and safety of the public nuisance as long as its for public benefit, not just for some group of people it police power if it protects the public (Often, argument is whether its protecting another group of people or protecting the public)

5.

Notes a. b. c.

This taking isnt arbitrary because the city is expanding and s brickyard is not compatible The court doesnt concern itself with fairness here once its an expression of the legitimate power of the legislature and its not arbitrary, then theres nothing further City doesnt have to pay for the re-zoning (zoning out of the brickyard) a. If its a private nuisance, then the cheapest cost-avoider has to pay, but here, we dont see the same idea even though it wouldnt be difficult to compensate the brickmakers in the area (arent too many) b. Danger of zoning is the oppression of minority rights

74

d. e.

Obvious problem with this case if something can be characterized as a nuisance and the legislature can show that the re-zoning/taking of the nuisance is not arbitrary, the legislature can zone the nuisance out of existence and not have to compensate (marked differe nce from Spur) Preventing harm versus conferring a benefit a. The line between the 2 is very fuzzy but as soon as something is declared preventing harm, theres no compensation Is there a permanent physical occupation? a. If yes then there is a taking (Loretto) b. If no then ask if the regulation is a nuisance/prevention of harm to others 1. If yes then its not a taking and the owner is owed no compensation (Hadacheck) 2. If no, move to the Penn Central balancing test between the need for the regulation and the interest it serves balances against the harm to the landowner if the regulation is enforced i. once at the balancing test, the government wins

1.

iv.

v.

Rules Based on Measuring and Balancing 1. Penn Central Transportation Company v. City of New York USSC, 1978 (990) a. In 1967, NYC () Landlarks Preservation Commission designated Grand Central Terminal a landmark under the citys Landmarks Preservation Law. The terminal was owned by and its affiliates. For at least 65 years, the property had been used as a railroad terminal with office space and concessions. The landmark law did not interfere with the Terminals continued use in this capacity, but it restricted any changes in the Terminals exterior architectural features withoug the Commissions approval. In addition, landmark owners are allowed to transfer their development rights under zoning regulations to contiguous properties on the same block, or other properties they also own. At this time, owned several properties. In 1968, in order to increase its income from the terminal, entered into a long -term lease with UGP, a British corporation. UGP was to build a 55-story office building above the Terminal, paying an annual rent of $1 million during construction and at least $3 million thereafter. submitted 2 plans to the Commission. The Commission sternly rejected both plans. It also said, however, that construction would be allowed depending on whether a proposed addition would harmonize in scale, materials, and character with the Terminal. Rather than appeal, brought suit in state court, alleging the landmarks law affected a taking of s property. The trial court granted injunctive and declaratory relief (from enforcement) but the intermediate appellate court reversed. The NY Court of Appeals affirmed, finding no taking as the law only restricted, not transferred, control of the property. The USSC affirmed. b. There is no set formula for deciding these cases, and so this Court must look at the particular facts here. The extent to which the law has interfered with distinct, investment-backed expectations is particularly relevant. claim that the airspace above the Terminal is a valuable property interest that has been taken through the landmark law. This claim is rejected because the rights in the property as a whole, not those in individual estates, must be considered. Also, argues that the law effects a taking by significantly diminishing the economic value of an individual landmark site, the Terminal property, unlike other laws which impose restrictions on entire historic districts. This argument is rejected because this landmark law, like others throughout the country, is part of a comprehensive plan to preserve landmarks all over the city. The interference with s property rights is not severe enough to equal a taking. The law does not interfere with s primary expectation concerning the use of the Terminal for railroad service, office space, and concessions. Thus, i s permitted to obtain a reasonable return on its investment in the Terminal. Moreover, has not been barred from any construction over the terminal; only the 2 plans in question have been rejected. Because has not submitted plans for a smaller structure, it is not known whether will be denied permission to use any of the Terminal airspace. Finally, though claim their airspace rights have been taken, has always had the right to transfer those rights to nearby properties. The situation might not be ideal for them, but those rights are still valuable c. A law which does not interfere with an owners primary expectation concerning the use of the property, and allows the owners to receive reasonable return on his investment, does not effect a taking which demands just compensation d. This landmark law is unlike typical zoning restrictions that usually provide benefits for, as well as impose burdens on, restricted properties. There is no reciprocity of advantage here. Only a few buildings are singled out with considerable burdens and no comparable benefits. Because there was no nuisance-control justification in restricting s airspace rights, the action resulted in a taking. Though the value of the transferable development rights may possibly be valuable enough to serve as just compensation, there is not enough evidence to prove this conclusively. That decision should be remanded to see if these transferable rights amount to a full and perfect equivalent for the property taken e. Notes on Penn Central a. The issue before the court is only about whether this whole organization is constitutional, i.e., Can they tell individual corporation to do or not to do something? b. The Court employs a balancing test once its determined that the restriction is to further a legitimate objective, a balancing test is conducted between the need for the regulation and the interest is serves and the harm to the landowner 1. Factors the court says are designed to help this determination: (p.996) i. Extent of the economic interference ii. Distinct economic investment-backed expectations (landowner goes in and invests a certain amount with certain expectations) iii. Transferable Development Rights (TDRs) iv. Whether theres unique oppression (landmark legislation is comprehensive, so these landowners are not singled out) v. Character of the occupation vi. Reciprocal burdens and benefits (owners are also benefited by the regulation) A Third Categorical Rule 1. Lucas v. South Carolina Costal Council a. In 1977, the SC legislature enacted a Costal Zone Management Act, after the earlier passage of a similar law in Congress. The SC Act imposed restrictions on owners of costal zone land that qualified as critical area, defined in the Act to include beaches and adjacent sand dunes. Owners of such land had to obtain a permit from the newly formed SC Costal Council () before putting the land to a use other than the use the critical area was devoted to on September 28, 1977. This Act marked the beginning of SCs expressed interest in managing the development of costal areas. In the late 1970s, and others started extensive residential development on the Isle of Palms, a barrier island off the coast of SC. In 1986,

75

b.

c. d.

e.

f.

g.

bought 2 lots on this island for $975,000, planning to build single-family homes on them. These lots did not fall within the critical area described in the 1977 Act, and thus were unaffected by it. The land adjacent to s lots already contained similar structures. IN 1988, the legislature enacted the Beachfront Management Act. This Act included s property in its zone. filed suit in SC Court of Common Appeals, contending this ban on construction effected a taking by completely extinguishing the value of his property. The trial court found that the 1988 Act deprived of any reasonable economic use of his lots. Thus, the court concluded that s properties had been take n and ordered compensation. The SC of SC reversed, saying when a regulation on the use of property is meant to prevent serious public harm no compensation is required, regardless of its effect on the propertys value. USSC reversed and remanded In the past, the Court has recognized that if the regulation of property goes too far it will be recognized as a taking. Though too far has never been exactly defined, a taking has been found where a land -use regulation denies an owner the economically viable use of his land. Though this rule has never been formally justified, it has been accepted because such a loss, in the landowners point of view, is the equivalent of a physical taking. Though the government has been allowed, in certain cases, to affect property values without paying compensation, this should not occue when a landowner has been deprived of all economic beneficial uses of his land. Here, the SC SC felt the 1988 Act involved a valid exercise of the States police powers to mitigate the harm to the public interest that s proposed development would case. essentially conceded that the states beach areas were a valuable resource, that new construction contributed to the erosion of these areas, and that discouraging such construction was necessary to prevent great public harm. The distinction, however, between an act which prevents public harm and one which confers a public benefit often lies in the eye of the beholder. Such a distinction is almost impossible to make objectively. Thus, this sort of harmful-use logic should not be used to separate regulatory takings which require compensation from regulatory deprivations that do not. To adopt the SC SCs approach to this problem would essentially wipe out the limitation on regulation which goes too far. Therefore, a state should only be allowed to deprive an owner of all economically beneficial use of property, without needing to pay compensation, when the interest in the regulated use was not party of the title to being with. In other words, a property owner necessarily expects the uses of his property to be restricted, on occasion, by newly enacted legislation. That owner, however, should not have to expect that his property will be rendered economically worthless by similarly new legislation against certain uses of land. Such a limitation must inhere in the title itself, in the background principles of the states property and nuisance law. The source of such a sweeping limitation, then, must not be the legislative designation of something as a nuisance, but rather the common law doctrine A land-use regulation that deprives an owner of all economically valuable use of property by prohibiting uses that are permitted under background principles of property and nuisance law results in a taking, and thus requires compensation The majority opinion, though establishing a framework for remand, did not decide the question of whether a temporary taking had indeed occurred. Also, the determination that s property was deprived of all economic value by a mere development restriction is a curious one. In addition, the finding that a parcel of property is left with no value must be based under the Takings Clause on the owners reasonable, investment-backed expectations. Though this definition of value may seem circular in some aspects, it is not entirely so. The Constitution protects expectations that are based on objective rules and customs that are generally accepted as reasonable ones. Finally, reasonable expectations must be understood in light of the whole of our legal tradition. The common law doctrine of nuisance is too narrow to rely on when considering issues of a governments regulatory power, particularly in the context of todays complex, modern society BLACKMUN with this decision, the court launches a missile to kill a mouse. First, if the state legislature is correct that the construction ban in the zone outlined prevents serious harm, then the Act is constitutional. Earlier decisions have consistently upheld regulations meant to protect the common welfare. Here, never challenged the legislative findings as to the importance of the construction ban, so the SC SC correctly found that no taking had occurred. Second, the trial courts finding that s property had lost all its economic value is wrong. The property can still be used for swimming, picnicking, camping, etc; also retains the right to sell this valuable land. Third, the Court has based many decisions on the idea that the state has the fill power to restrict an owners use of property if it is harmful to the public. It would be ridiculous to suggest that, under this new theory, an owner can have the right to harm the public if the right amount of economic loss can be shown. Fourth, the Courts reliance on a background principles standard is questionable. Though the Court claims the harm versus benefit question cannot be objective answered by a legislative decision, state courts make exactly the same kind of decision that the Court refulses to accept when made by the state legislature. There is no reason to think that modern interpretations of historic common law doctrines will be any more objective than a legislative decision. Finally, there is no historic or common law evidence of any kind of limit on a states ability to regulate a harmful use of property to the point of eliminating its economic value STEVENS The new rule is entirely arbitrary. A court could define property very broadly so total takings would only be rarely found. Likelwise, an investor could purchase the right to build an apt. building on a particular lot, and later argue that a zoning restriction would deprive the owners interest of value. This rule cannot be justified merely by an owners perception of the regulationl a regulation which diminishes a lots value by even a fraction would almost always appear to the owner to be a condemnation. The Court belief in the relative rareness of total takings does nothing to explain why some regulations should be categorized as takings and some should not. Also, a total taking does not necessarily mean that a property owner is being arbitrarily singled out, even though such an occurrence is possible. Furthermore, the exception for uses of property that are permissible under background pr inciples basically freezes the states common law. This action denied the legislature its traditional power to revise and develop law. Also, with this rule and exception, the Court neglects to consider the character of the regulatory action. This has traditionally been the most important element to consider in taking analysis, yet it is disregarded by the Courts decision Notes on Lucas a. Wipe Out Rule if you can say that in a particular case theres been a complete wipe out of all value attached to the property, the court moves to another rule 1. If it was a wipe out and part of the property owners interest to begin with, compensation is required b. Doesnt overrule Hadacheck because that was not a total wipe out situation and the use was something the court could have enjoined c. Not sure how important this case will be may never come up again since total wipe out is nearly impossible to show 1. Here, the Court assumed total wipe out because it was a finding of fact by the lower court 2. The court is really leaving this up to the states if you authorize permanent invasions, its a taking if if you totally wipe out the value of someones land, its a taking

76

i.

Other than that, its up to the state courts to decide property law is state law and the Court will defer to the legislature as to how much protection to give against eminent domain

77

You might also like